VNMath

107 11 0
VNMath

Đang tải... (xem toàn văn)

Tài liệu hạn chế xem trước, để xem đầy đủ mời bạn chọn Tải xuống

Thông tin tài liệu

Ví dụ 8.. Một trong số đó là tích phân Eliptic với khá nhiều dạng biểu diễn khác nhau và cũng có không ít các ứng dựng liên quan đến lớp tích phân này.. Mức độ áp dụng của hai hàm này tr[r]

(1)

Số 03 - Năm 2009

(2)

Mục lục

Câu chuyện Toán học

• Tốn học điện ảnh Dương Tấn Vũ 03

Bài viết chuyên đề

• Phép Nghịch đảo giải chứng minh Hình học phẳng Nguyễn Lâm Minh 11 • Applying R,r,p - method in some hard problems Tran Quang Hung 26

• Các phương pháp tính tích phân Nguyễn Văn Vinh 34

• Bài toán Kakeya Phan Thành Nam,

Mạch Nguyệt Minh 43

Bài viết Chuyên đề Dịch thuật

• Phương trình bất phương trình hàm số Đinh Ngọc Vương 56

Bạn đọc Tìm tịi

• Bí ẩn tập đóng lồng Trần Bạt Phong 71

Cuộc thi giải Tốn MathVn

• Đề Tốn dành cho Học sinh 75

• Đề Tốn dành cho Sinh viên 76

• Các vấn đề mở 77

• Lời giải kì trước 78

Nhìn giới

• Kỳ thi Qualify cho nghiên cứu sinh Mỹ 89

Olympic Học sinh – Sinh viên

• Olympic Sinh viên Kiev 2009 93

• Olympic Xác suất Kolmogorov 2009 94

• Kì thi TST Việt Nam 2009 - Đề thi bình luận Trần Nam Dũng 96 Sai lầm đâu?

(3)

Câu chuyện Toán học Toán học Điện ảnh

Phỏng dịch theo Joan Lasenby, Maths goes to the movies, Plus Magazine, Tháng 03 - 2007 Dương Tấn Vũ, Học sinh trường THPT Quốc Học - Huế

Ăn hết bỏng ngô chưa? Chỗ ngồi bạn tốt chứ? Bạn ngồi có thoải mái khơng? Hãy bắt đầu xem Toán học hân hạnh giới thiệu

Tất điều ngạc nhiên hình ảnh vi tính giống thực đến mức khơng thể tin phim Nhưng hầu hết không nhận khủng long Cơng viên kỷ Jura kì quan Chúa tể nhẫn - đặc biệt nhân vật Gollum - khơng thể có khơng có Tốn học

Những hình ảnh đáng kinh ngạc làm nào? Đồ họa vi tính tầm nhìn máy tính vấn đề lớn Trong viết này, có nhìn đơn giản vào vài yếu tố toán học cần dùng để đến sản phẩm cuối Đầu tiên, xây dựng giới thấy phim, sau mang chúng đời thực

Dựng cảnh

Mơ hình chủ thể khung dây làm từ đa giác đơn giản ví dụ tam giác

(4)

cách để làm điều ngón tay phải phía tam giác - phía phía ngồi Nếu bạn thử với ví dụ, bạn thấy chiều hướng (pháp tuyến ngoài) tam giác(A, B, C)sẽ ngược chiều với tam giác(A, C, B)

Pháp tuyến (A, B, C)ngược hướng với (A, C, B) xác định theo quy tắc nắm tay phải (quy tắc đinh ốc)

Kẻ tia từ điểm nhìn đến bề mặt Nó có phản xạ giao với nguồn sáng không?

Bây bề mặt vật thể mạng lưới tam giác, sửa tơ thành phần Điều quan trọng phải bắt giữ ánh sáng thực tế khung làm mơ hình, điều thực quy trình gọi ray tracing Bắt đầu từ điểm nhìn, kẻ tia trở lại hướng vào vật thể để chúng phản xạ qua Nếu tia từ mắt phản xạ qua bề mặt (một mắt lưới tam giác) giao với nguồn sáng, tơ bề mặt màu sáng để xuất chúng bị chiếu sáng nguồn sáng Nếu tia không giao với nguồn sáng tô màu tối

Để vẻ tia trở lại bề mặt, cần mơ tả bề mặt cách tốn học bao gồm đường thẳng mặt phẳng mô tả bề mặt Điều thực việc sử dụng Vectơ Chúng ta đặt hệ tọa độ không gian chiều lên phông với điểm gốc(0,0,0)- đặt điểm nhìn Một vectơv= (a, b, c)bây biểu thị mũi tên từ gốc đến điểm có tọa độ(a, b, c) Chúng ta nhânvvới số, chẳng hạn, theo quy tắc2v= 2(a, b, c) = (2a,2b,2c) Vậy2v mũi tên vẽ hướng vớiv dài gấp đôi

(5)

biểu thức mô tả đường thẳng chứa vectơv Nó mơ tả đường thẳng - tia phát từ điểm gốc nhìn theo hướng cho vectơv

Mặt phẳng xác định bề mặt tam giác miêu tả mẫu thông tin: tọa độ đỉnh-gọi đỉnha1, hai vectơ thể đường thẳng từ đỉnha1 đến đỉnha2 từ đỉnha1 đến đỉnha3

Dưới cho thấy phương trình tia từ mắt phương trình mặt phẳng cho bề mặt Để tìm tia có cắt bề mặt khơng có cắt đâu để lập phương trình tia phản xạ, cần giải phương trình bao gồm biểu thức

Phương trình tia, với λlà số thực vàv vectơ: r=λv

Phương trình mặt phẳng xác định bề mặt với đỉnha1, a2 vàa3: r=a1+µ1(a2−a1) +µ2(a3−a1)

Ray tracing tạo khung cảnh thực tế chậm Nó chấp nhận phim vi tính, trở thành vấn đề bạn cần thay đổi ánh sáng thời gian thực, ví dụ trị chơi vi tính Những tượng phức tạp bóng, tụ quang, phản xạ phức tạp khó để làm mẫu sống động Nhiều phương tiện tốn học phức tạp, ví dụ nhưPrecomputed Radiance Transfer1 vàRadiosity2 sử dụng

Các game Doom Neverwinter nights đòi hỏi ánh sáng sống động

(6)

Tất phải cần chút tưởng tượng

Một khung cảnh thiết lập chiếu sáng, đợi đạo diễn nói “Action” nhân vật bắt đầu chuyển động Bây kiểm tra tốn học mang những hình ảnh đến với sống không

Một chuyển động mà vât thể trình diễn xoay tròn quanh trục cho trước qua góc cho trước Hình học tọa độ cho cơng cụ để tính vị trí điểm vật thể sau chúng xoay, điều quan trọng công cụ phải nhanh hiệu

Để tìm cơng cụ này, hày lùi bước trở lại lớp học môn Tốn Chúng ta biết có hai bậc hai 25 là:5 và−5 vì(±5)2= 25 Nhưng bậc hai -25 bao nhiêu? Để tìm bậc hai số âm, nhà toán học xây dựng số mới, gọi lài, vớii2=−1 Vậy vì(±5i)2= 25i2=−25nên tìm rằng√−25 =±5iSự đưa vào sốicó nghĩa phương trình x2 =−1 giải Và số có dạngz =x+iy, gọi số phức, trở thành công cụ quan trọng tốn học Nhưng nhiều người khơng vui với số ảoimới lạ Cuối vào năm 1806 nhà toán học nghiệp dư Jean-Robert Argand đưa giải thích hình học số phức sối Argand liên kết số phức với điểm trên mặt phẳng số thực nằm trục số ảoinằm trục khác Ví dụ số1 +itương ứng với điểm (1,1) Một cách tổng quát sốa+ibtương ứng với điểm(a, b)

Phép nhân với số phức có ý nghĩa hình học - phép quay

Argand nhận phép nhân với số phức mô tả ý niệm hình học: phép quay Hãy xem chuyện xãy ta nhân số1 +i, biểu diễn điểm(1,1), với i

i(i+ 1) =i−1 =−1 +i

(7)

chính điểm(−1,−1), phép quay90◦ Nhân vớiilà "lệnh" để quay90◦! Thực tế, quay nào, khơng chỉ90◦, đạt đạt phép nhân với số phức

Tiến tới 3D

Tấm bia tưởng niệm đặt cầu Broome (Dublin), Hamilton phát quaternion cầu

Nhà toán học Sir William Rowan Hamilton cống hiến 20 năm cuối đời cho việc tìm kiếm cách biểu diện phép quay ba chiều tương tự việc số phức biểu diễn phép quay không gian chiều

Đến cuối đời Hamilton khám phá câu trả lời, hình thức ơng gọi quaternion - số có dạng

q=a0+a1i+a2j+a3k Vớii2=j2=k2=ijk=−1 vàa

0, a1, a2, a3 số thực

Cũng làm với số phức, mơ tả quaternion cách hình học sử dụng chúng để mơ tả phép quay Nhưng lần phép quay không gian chiều

Để làm điều này, i, j vàk phải mô tả mặt phẳng không gian chiều: i mơ tả mặt phẳngyz, j cho mặt phẳng xz vàk cho mặt phẳng xy với pháp tuyến theo hướngx,−y vàz

(8)

Giả sử cần quay điểma= (a1, a2, a3)một gócβ qua trục qua gốc tọa độ cho vectơ(b1, b2, b3) Chúng ta xây dựng quaternionq1 vàq2sử dụng vectơ trụcb góc quayβ

q1cos(β/2) + sin(β/2)(b1i+b2j+b3k)

q2= cos(β/2)−sin(β/2)(b1i+b2j+b3k)

Sau nhâna(được biểu diễn kết hợp vectơ đơn vị theo hướngx, y vàz) với quaternion (tuân theo quy tắc đặc biệt nhân mặt phẳngi, j vàk với vectơ đơn vị), ta được:

a0=q1aq2

Thì điểma0cho phép nhân xác điểm có bạn quayaquay quanh trục cho trước góc β! Vậy số phức dùng để miêu tả quay mặt phẳng, quaternion sử dụng để mơ tả quay khơng gian chiều

Ánh sáng lóe lên Hamilton, ông cầu Dublin, hóa cách hiệu để quay vật thể không gian chiều Nhưng người vui với phương pháp nhân mẻ ông Lord Kelvin, nhà vật lí, nói quaternion:“ tài tình, dù nữa, hồn tồn tai họa cho đụng đến nó!”

Có điều đặc biệt đáng ngại với số người bạn nhân quaternion, kết phụ thuộc vào thứ tự bạn nhân chúng, đặc tính gọi khơng giao hốn Ví dụ, từ quy tắc nhân Hamilton, thấy rằngij =kvàji=−k Tuy nhiên người xemi, j vàknhư mặt phẳng bản, đặc tính, gây lo lắng cho Kelvin người thời với ông, sinh trực tiếp từ tốn học

Mang hình ảnh vào cuốc sống

Phát minh Halminton sử dụng nhiều ứng dụng đồ họa để di chuyển vật thể hay tạo vận động Hai cơng cụ quan trọng đồ họa vi tính biến hình phép nội suy Phép nội suy kĩ thuật keyframing bao gồm xác định hình dạng, vị trí ban đầu kết thúc vật thể máy tính thực cơng việc giữa, thấy hình sau

Hình dạng ấm trà thay đổi qua chuỗi ảnh

(9)

Một vải rơi xuống cầu trịn đựơc làm mơ hình việc sử dụng quy tắc vật lí

Và sau vận dụng để làm hình ảnh cầu méo

Làm cho Gollum thật!

(10)

động thực tế, thường thường phải đòi hỏi kỹ thuật bắt giữ chuyển động

Nhiều nhân vật, Gollum phim Chúa Tể Của Những Chiếc Nhẫn chẳng hạn, đựơc xây dựng sử dụng cách bắt giữ chuyển động Điều đựơc thực việc gắn gương phản xạ người thật điểm thể: đầu, vai, khuỷu tay, đầu gối Những cá thể quay phim những máy quay đa chiều thay đổi vị trí gương phản xạ lưu trữ máy tính Một xương đặt vào không gian chiều ảo Cuối cùng, tất kĩ thuật mô tả sử dụng để đặt thịt vào xương tạo nhân vật sống, thở chuyển động

Dữ liệu thu đựơc từ chuyển động gương phản xạ gắng vào phần khác thể

Một khung xương lắp cách toán học vào liệu

(11)

Bài viết Chuyên đề MathVn Phép Nghịch đảo - Ứng dụng giải và

chứng minh Hình học phẳng

Nguyễn Lâm Minh1, Học sinh trường THPT chuyên Lê Hồng Phong, Tp HCM

I - Định nghĩa - Tính chất

1.1 Định nghĩa

Hồi cịn học THCS, có tốn quen biết:

"Cho (O) Một điểmA nằm ngồi đường trịn (O) Vẽ tiếp tuyến AK đến (O) (K∈(O)) Một cát tuyến từAđến(O)cắt(O)lần lượt tại2điểmM, N Khi đó, ta ln cóAK2=AM.AN". Để ý với điểm M0 nằm đường trịn(O)thì ln tồn điểmN0 khác giao điểm cuả(O)vàKM0sao choAM0.AN0=AK2 Khi choM0−→K thìN0−→K

Phép nghịch đảo xây dựng dựa toán quen thuộc bên Tức là, với điểm O cố định nằm mặt phẳng số sốk6= Nếu ứng với điểmP cuả mặt phẳng khác với điểmO, ta tìm điểm P0 khác nằm trênOP sao choOP OP0=kthì phép biến hình biến P 7→P0 được gọi là phép nghịch đảo cực O, phương tích k Ta ký hiệu phép biến hình là I(O, k)hayf(O, k) Trong viết này, tác giải sử dụng ký hiệuf(O, k)vàf(P) =P0 ám

P0 ảnh cuảP quaphép nghịch đảo cực O, phương tíchk 1.2 Tính chất

a)Phép nghịch đảo có tính chất đối hợp VìOP OP0=k=OP0.OP Do đóP =f(P0)và ngược lạiP0 =f(P) Như vậyf◦f(P) =P hay f2là phép đồng nhất.

b) Nếuk >0 hai điểmP, P0 nằm phía O Đường tròn(O,√k)lúc gọi làđường tròn nghịch đảocuả phép nghịch đảof(O, k) Khi điểmM mà thoả mãnf(M) =M

được gọi điểm kép cuả phép nghịch đảof(O, k) Hơn nữa, tập hợp điểm là(O,√k)

Nếu k < hai điểm P, P0 nằm hai phía khác O Trong trường hợp không xuất điểm kép đối vớif(O, k)do đường trịn nghịch đảo cuảf(O, k)sẽ gọi đường trịnbán thực, tâm cuả đường trịn thực bán kính cuả đường tròn ảo

KhiM tiến lại gầnO cực nghịch đảo ảnh cuả thìf(M)sẽ tiến xaO, tức

M −→Othì f(M)−→ ∞

c) Phép nghịch đảof(O, k)có phương tíchk >0 vàP, P0 ảnh qua phép nghịch đảo

f(O, k) đường trịn qua2 điểmP, P0 đềutrực giao với (O,√k)(Hai đường tròn(O),(O0) gọi làtrực giaovới nếu2tiếp tuyến tại1giao điểm cuả(O)và(O0)vng góc với nhau) Hơn nữa, đường trịn(C)quaP, P0 biến thành quaf(O, k), vớik >0

(12)

d) Nếu(O1)và(O2)lần lượt trực giao với(O,√k),k >0 và(O1),(O2)lần lượt cắt ta hai điểm hai điểm ảnh cuả qua phép nghịch đảof(O, k)

e) Phép nghịch đảof(O, k),k6= Thì với hai điểmA, B khơng thẳng hàng với cực nghịch đảo, ta ln cóA, B, f(A), f(B)là điểm đồng viên

f) ĐặtA0=f(A)vàB0=f(B)khi đóA0B0 =|k| AB OA.OB

Tuy nhiên ta lưu ý khẳng địnhf(O, k) :AB7→A0B0 sai!

Tính chất ảnh cuả đường thẳng hay đường tròn qua phép nghịch đảo phát biểu sau đây:

•Từ định nghiã ban đầu, ta biết đường thẳng dbất kỳ qua cực nghịch đảoO

thì quaf(O, k),dbiến thành

• Một đường thẳng dbất kỳ khơng qua O- cực nghịch đảo qua f(O, k), dsẽ biến thành đường tròn(C)đi qua cực nghịch đảo

Thật vậy, ta gọi P hình chiếu cuả O lên d vàP0 ảnh cuả P quaf(O, k) Gọi A điểm nằm dvàA0 ảnh cuả Aquaf(O, k) Khi ấy, ta đượcOP.OP0 =OA.OA0 =k, từ suy ra4A0P0O đồng dạng 4P AO Suy ra∠AP O =∠A0P0O = 90◦, điều nói lênA nằm đường trịn đường kínhOP0 Hơn nữa, tâm cuả (C)sẽ ảnh cuả điểm đối xứng với O quadqua phép nghịch đảo cựcO, phương tíchk

• Đảo lại, đường trịn (C) qua cực nghịch đảoO Khi đó, qua f(O, k), (C) biến thành đường thẳngdkhông qua cực nghịch đảo

Gọi P điểm đối xứng cuả O qua tâm đường tròn (C) P0 ảnh cuả P qua f(O, k) Với A điểm nằm (C) (A 6= O), ta gọi A0 ảnh cuả A qua f(O, k) Cũng chứng minh cuả tính chất bên trên, đó, ta được4OP0A0 đồng dạng với4OAP Từ suy ra ∠OP0A0=∠OAP= 90◦ Do điểmAsẽ nằm đường thẳng quaP0và vng góc vớiOP • Với đường trịn(C)khơng qua cực nghịch đảoO quaf(O, k), (C)sẽ biến thành(C0) khơng qua cực nghịch đảo

Lấy điểm M nằm (C) M0 ảnh cuả M qua f(O, k) Khi đó, ta có

OM.OM0 = k Gọi N giao điểm thứ hai cuả OM (C) p phương tích cuả O (C), ta cóOM.ON =p Từ suy raOM0 =kpON

Hệ thức chứng tỏ rằngM0 ảnh cuảN qua phép vị tự tâmO, tỉ số k1 = kp KhiM chạy vạch nên (C) N chạy vạch nên đường tròn (C), cònM0 vạch nên(C0) ảnh cuả (C)quaH(O, k1) Do đó(C0)là ảnh cuả(C)quaf(O, k) Vì(C)khơng qua cựcO, hiển nhiên(C0) không qua cựcO

Tuy nhiên tâm cuả(C)sẽ không biến thành tâm cuả(C0)quaf(O, k)

g) Phép nghịch đảo bảo tồn góc đường tròn (hay đường tròn đường thẳng, hay hai đường thẳng)

(13)

Ta định nghĩa góc hai đường cong:

Định nghĩa Cho hai đường (C1) (C2) cắt điểm A đó, ta dựng tiếp tuyến cuả(C1) và(C2) Khi đó, ta định nghĩa góc hai đường cong(C1)và(C2) góc hai tiếp tuyến tạiA cuả chúng

Chứng minh Trước tiên, ta xét bổ đề sau:

Bổ đề Cho f(O, k) biến đường cong(C)thành đường cong (C0) Nếu A, A0 hai điểm tương ứng (C),(C0) chúng có tíếp tuyến tiếp tuyến đối xứng với qua đường trung trực cuả đoạnAA0

Thật vậy, ta gọiM điểm nằm trên(C)vàM0 ảnh cuảM quaf(O, k), suy raM0 nằm trên(C0) Ta lại cóOM.OM0 =OA.OA0 =k, suy M, M0, A0, Anội tiếp.Gọi(K)là đường tròn quaA, A0, M0, M Cho M −→A, M0 −→ A0 Do M A, M0A0 suy biến thành tiếp tuyếnt vàt0 A, A0 cuả đường cong (C), (C0)tương ứng và(K)suy biến thành đường tròn (K0)tiếp xúc với đường cong (C) và(C0)lần lượt A vàA0 Rõ ràng lúc này, t vàt0 tiếp tuyến Avà A0 cuả (K0) tương ứng Từ suy ra,t vàt0 đối xứng qua đường trung trực cuảAA0.

Chứng minh tính chất

Giả sử qua phép nghịch đảo f, hai đường cong (C)và(D)cắt điểmA biến thành đường cong(C0)và(D0)cắt tạiA0 =f(A)

Theo bổ đề tiếp tuyếnAtvàA0t0 cuả(C)và(C0)tạiAvàA0 đối xứng qua trung trực AA0 tiếp tuyếnAuvàA0u0 cuả (D)và(D0)tại A vàA0 đối xứng qua trung trực củaAA0 Từ suy ra(A0t0, A0u0) =−(At, Au)

II - Vẻ đẹp cuả phép nghịch đảo chứng minh tốn hình học phẳng

Ta khởi động với toán quen thuộc, xuất nhiều kỳ thi nước, gần kỳ thi tuyển sinh THPT năm học 2009-2010

Bài toán Cho 4ABC nội tiếp đường tròn tâm (O) Gọi B0, C0 hình chiếu cuả

B, C AC, AB Chứng minh tiếp tuyến tạiA cuả đường tròn (O) song song vớiB0C0, từ suy raAO⊥B0C0

(14)

Trước tiên dễ thấy rằngB, C0, B0, C đồng viên Do đóAB.AC0=AC.AB0=k Xét phép nghịch đảo cựcA, phương tíchk, ta đượcI(A, k) :B07→C,C07→B Vì vậyI(O, k) :B0C07→(O) Gọi ta tiếp tuyến A cuả (O) ta có I(A, k) : ta 7→ d Mặt khác ta tiếp xúc (O)

ta||B0C0(phép nghịch đảo bảo tồn góc) Khi ấy, ta có ngayOA⊥B0C0 (VìOA⊥ta)

Bài toán toán thuộc dạng kinh điển quen thuộc Nhiều bạn chí bạn THCS khơng gặp khó khăn chứng minh tốn Trên trang websitewww.mathlinks.ro có đến "hàng tá" cách giải cho tốn này, có cách túy biến đổi góc Riêng ý sau cuả tốn chứng minh mà không cần dùng đến ý đầu Thật vậy, ta biết qua phép nghịch đảo cựcA, phương tíchk,I(A, k) :B0C07→(O) Do dóO ảnh cuả điểm đối xứng vớiAquaB0C0 Rõ ràng ta có ngayAO⊥B0C0 Hơn nữa, từ ý này, ta cịn đường thẳng quaA, B, Cvng góc vớiC0B0,A0C0,A0B0thì đồng quy với tạiO Bài tốn bên có dạng tổng qt Chúng ta xét dạng tổng quát cuả toán qua toán

Bài toán 2.Cho 4ABC nội tiếp đường tròn tâm(O) Một đường tròn (O0)bất kỳ qua B, C thoả mãn cắt đoạnAB, AC tạiB0, C0 GọiA0 giao điểm cuả B0C0 Một đường trịn (K)có tâm nằm trênB0C0 tiếp xúc vớiAA0 tạiA Chứng minh rằngA0 vàO hai điểm liên hợp với qua(K)

Lời giải

(15)

Thật vậy, qua phép nghịch đảo cựcA, phương tíchk=PA/(O0), ta cóI(A, k) :B07→B,C07→C Do đóB0C0 7→(ABC) Từ suy AO⊥B0C0 Điều ày chứng tỏA, O, A0 thẳng hàng Từ suy điều phải chứng minh.

Bài toán (Định lý Ptolémée) Chứng minh điều kiện cần đủ để tứ giác lồi nội tiếp tích hai đường chéo cuả tổng cuả tích hai cạnh đối diện

Lời giải

Xét tứ giác ABCD Xét phép nghịch đảo cực D, phương tíchk Thì I(D, k) :A 7→A0,

B7→B0,C7→C0 Như vậyABCDlà tứ giác nội tiếp khiA0, B0, C0 thằng hàng Điều xảy khiA0C0=A0B0+B0C0 hay nói cách khác là:

|k| AC

DA.DC =|k| AB

DA.DB =|k| BC DB.DC

Nhân hai vế cho DA.DB.DC.|k|, ta thu được:AC.BD=AD.BC+AB.DC

Định lý Ptolémée toán quen thuộc em học chuyên sâu toán THCS cách giải phổ biến cuả định lý cách gọi thêm điểm D0 thoả mãn ∠D0DC = ∠BAC, ∠D0CD = ∠BCA để tạo cặp tam giác 4CD0D 4CBA đồng dạng cặp đồng dạng khác, xuất khâu biến đổi góc Rõ ràng quan điểm phép nghịch đảo, định lý Ptolémée trở nên khơng chút khó khăn việc suy nghĩ gọi thêm yếu tố phụ! Lưu ý phương pháp dùng phép nghịch đảo, tương tự ta chứng minh định lý mở rộng: "Điều kiện cần đủ để đa giác lồi mặt phẳng A1A2 An, n≥4 nội tiếp đường tròn là:Pn−1

i=2 AiAi+1(Qk6=1A1Ak) =A2An.A1A3 A1An−1"

Tiếp theo ứng dụng khác cuả phép nghịch đảo tốn cuả Nga (Liên Xơ truớc đây) đề nghị kỳ thi IMO 1985

Bài toán Cho tam giác ABC Một đường tròn tâm O qua điểm A, C cắt lại đoạn

AB, BC theo thứ tự tai hai điểm phân biệt K, N Giả sử đường tròn ngoại tiếp tam giác ABC

vàKBN cắt hai điểm phân biệtB, M Chứng minh rằng:∠OM B= 90◦

(16)

Gọi Rlà bán kính cuả đường trịn tâm(O)nói GọiP ≡KN∩AC,S ≡KC∩AN Theo kết quen thuộc thìB đố cực cuảP Squa(O)và ngược lạiP đối cực cuảBSqua (O) Do đóSsẽ đối cực cuảBP qua(O) GọiM0≡OS∩BP, ta có ngayOM0⊥BP Mặt khác, ta lại cóBS⊥OP (DoBSlà đường đối cực cuảP qua(O)), tương tựP S⊥OB Ta suy đượcS

là trực tâm4BOP Do gọiB0 ≡BS∩OP, ta có ngayB0là ảnh cuảP quaI(O, R2) Ta lại có I(O, R) :A7→A,C 7→C Do vậyAC7→(OAC), P ∈AC ⇒B0 ∈(OAC)⇒P O.P B0 =P A.P C Mặt khác, dễ thấy B, M0, B0, O đồng viên P M0.P B = P O.P B0 ⇒ P M0.P B = P A.P C

⇒ M0 ∈ (ABC) Để ý P A.P C = P K.P N =P M0.P B, M0 ∈ (BKN) Hay nói cách khácM0 ≡(BKN)∩(ABC)M ≡M0 Ta có điề phải chứng minh

Bài toán dạng kinh điển Có tới ba cách chứng minh cho tốn có cách biến đổi góc độ dài cạnh cầu kỳ Một lần nữa, với quan điểm phép nghịch đảo lại cho ta lời giải đẹp "thuần" tính lý thuyết, khơng chút tính tốn cho tốn cũ mà đẹp bên Cũng xin nói thêm, điểmM tốn có tên gọi làđiểm Miquel tứ giác toàn phần(BA, BC, P K, P A)có nhiều tính chất thú vị giới thiệu viết kỳ khác

Ta tiếp tục xem xét ứng dụng khác cuả phép nghịch đảo qua đề nghị IMO cuả Bulgaria năm 1995

Bài toán ChoA, B, C, D bốn điểm phân biệt nằm đường thẳng xếp theo thứ tự Các đường trịn đường kínhAC,BD cắt điểmX, Y Đường thẳngXY

cắtBC Z Cho P điểm đường thẳng XY khác Z Đường thẳngCP cắt đường trịn đường kính AC C vàM, đường thẳng BP cắt đường trịn đường kính BD B vàN Chứng minh rằng: AM, DN, XY đồng quy

(17)

Gọi (C1) đường trịn đường kính AC, (C2) đường trịn đường kính BD P nằm trên XY trục đẳng phương cuả (C1) (C2) PP /(C1) = PP /(C2) Nói cách khác ta có

P C.P M =P B.P N =k Xét phép nghịch đảo cựcP, phương tíchk, ta cóI(P, k) :M 7→C,A7→A0

⇒AM 7→(P A0C) Tương tự ta có đượcN D7→(P BD0), đóD0 ảnh cuảD qua phép nghịch đảo cực I(P, k) XY 7→ XY Do để chứng minh XY, AM, DN đồng quy, ta chứng minhXY trục đẳng phương cuả(P A0C)và(P BD0) Thật vậy, ta có∠P ZC=∠P A0C= 90◦ ⇒

Z ∈(P A0C) Tương tự ta có đượcZ ∈(P BD0) Do P Z ≡XY trục đẳng phương cuả (P A0C)và(P BD0) Từ ta có điều phải chứng minh

Một lần phép nghịch đảo lại cho ta thấy lợi hại cuả việc đồng quy Có thể thấy để ý rằng, phép nghịch đảo làm giảm tối thiệu lượng đường tròn xuất tốn mà thay vào đường thẳng, hay đường trịn "dễ nhìn hơn" Biến xa lại gần, biến khó kiểm sốt, khó nắm bắt thành dễ kiểm sốt, dễ nắm bắt đặc tính vơ lợi hại cuả phép biến hình đặt biệt Cũng lưu ý với bạn đọc rằng, toán giải trục đẳng phương cách gọiQvàQ0 giao điểm củaAM, DN

với XY chứng minh Q≡Q0 Phần chi tiết xin dành cho bạn đọc Tiếp theo lại ứng dụng khác phép nghịch đảo, ta tiếp tục xét tốn sau

Bài tốn Cho(O)đường kính BC Một điểmAnằm ngoài(O) GọiB0, C0 giao điểm củaAC,AB với(O) Gọi H giao điểm củaBB0,CC0 Gọi M, N tiếp điểm tiếp tuyến từ Ađến(O) Chứng minh rằngH, M, N

(18)

Gọi A0 hình chiếu A lên BC Dễ thấy H trực tâm tam giácABC Xét phép nghịch đảo cựcA, phương tíchAB0.AC =AC0.AB=AM2 =AN2=k, ta cóI(A, k) :M 7→M N 7→N,

H 7→A0 Dễ thấy∠OM A=∠ON A=∠OA0A= 90◦ Như ta đượcA0∈(AM N) Từ suy đượcM, H, N

Phép nghịch đảo tỏ hữu hiệu việc chứng toán thẳng hàng Bài tốn bên phát biểu cách tổng quát Việc chứng minh chi tiết xin dành cho bạn đọc:

"Cho(O), từ điểmK nằm ngồi (O)kẻ tiếp tuyếnKM, KN đến (O)trong đóM, N

là tiếp điểm Hai đường thẳng quaK cắt(O)tại điểm là(A, D),(B, C) Gọi

Glà giao điểm củaAC vàBD Chứng minh rằngM, N, S thẳng hàng."

Ta tiếp tục xét toán sau Đây tốn tính chất đẹp, khó thú vị cuả tác giả Hà Duy Hưng - giáo viên ĐHSP Hà Nội đăng tạp chí Tốn Học Tuổi Trẻ

Bài toán (THTT 360/6-2007)Cho tứ giácABCD có cặp cạnh đối diện khơng song song và hai đường chéo AC, BD cắt O Các đường tròn ngoại tiếp tam giácOAB vàOCD

cắt X vàO Các đường tròn ngoại tiếp tam giácOAD vàOCB cắt Y vàO Các đường trịn đường kínhAC vàBD cắt Z vàT Chứng minh bốn điểm X, Y, Z, T

cùng thuộc đường thẳng đồng viên

Lời giải

Trước tiên việc chứng minh toán, ta xét bổ đề sau đây:

Bổ đề.Cho tứ giác ABCD.E giao điểm cuảAB, CD;F giao điểm cuảAD, BC Khi đường trịn đường kínhAC, BD, EF có trục đẳng phương

Thật vậy, gọi H, K trực tâm cuả tam giácECB vàF CD GọiL, M, N hình chiếu cuảH lên EB, EC, CB vàP, Q, R hình chiếu cuả K lên DF, CF,CD Khi ta thu được:

HL.HC =HM HB=HN HE

KP KC=KQ.KD=KR.KF

Từ suy ra:

(19)

PK/(AC)=PK/(BD)=PK/(EF)

Điều chứng tỏHKlà trục đẳng phương chung cuả đường trịn đường kínhAC, BD, EF

Trở toán Xét phép nghịch đảo cựcO, phương tíchkbất kỳ Ta cóI(O, k) :A7→A0,B7→

B0,C7→C0,D7→D0,X7→X0,Y 7→Y0,Z 7→Z0,T 7→T0 Do đó(OAB)7→A0B0,(OBC)7→B0C0, (OAD)7→A0D0,(OCB)7→C0B0 Từ suy raX0≡A0B0∩C0D0,Y0≡A0D0∩B0C0.Z0, T0 giao cuả đường trịn đường kínhA0C0 vàB0D0 Áp dùng bổ đề bên ta đượcX0, Y0, Z0, T0 đồng viên Từ dẫn đếnX, Y, Z, T đồng viên hay thẳng hàng.

Các tốn mà có nội dung yêu cầu chứng minh điểm đồng viên hay thẳng hàng tốn ý tưởng tự nhiên ban đầu cuả tiếp cận toán sử dụng phép nghịch đảo Đây nhận định chủ quan cuả riêng tác giả dựa số kinh nghiệm giải toán, bạn đọc thấy việc dùng phép nghịch đảo tốn lại khơng cần thiết; cụ thể Bài tốn 7bên giải mà khơng dùng đến biến hình (Tham khảo lời giải số 366/thánh 12/2007 tạp chí THTT) biến đổi góc xét cặp tam giác đồng dạng -không dễ nghĩ! Song ý cuả tác giải muốn nói qua "lăng kính" cuả phép nghịch đảo cho ta lời giả đẹp, tự nhiên, đậm tính lý thuyết quan trọng khơng biến đổi tính tốn phức tạp Bài toán Cho tam giác ABC nhọn nội tiếp (O) Gọi A1, B1, C1 hình chiếu cuả

A, B, C lênBC, CA, AB GọiHlà trực tâm cuả4ABC Giả sửA2, B2, C2lần lượt giao điểm cuả

(20)

góc với B2C2, C2A2, A2B2 Chứng minh rằngda, db, dc đồng quy tâm đường tròn Euler cuả tam giácABC

Lời giải

Nhận xét trước tiên cuả tiếp cận tốn tính "đối xứng" cuả Do ta tập trung vào việc chứng minh đường thẳngda qua tâm Euler cuả tam giácABC sau lập luận tương tự chodb, dc

GọiOa, Ob, Oclần lượt tâm đường tròn ngoại tiếp tam giác4BHC,4CHA,4AHB Xét phép nghịch đảo cực A, phương tích k = AC1.AB = AH.AA1 = AB1.AC, ta có I(A, k) : B 7→ C1, B1 7→ C ⇒ BB1 7→ (ACC1), H 7→ A1 ⇒ C1A1 7→ (HBA) Từ suy I(A, k) :B27→B0, đóB0 giao cuả đường tròn(ACC1)và(HBA) Tương tự ta có

C27→C0 đóC0 giao cuả(ABB1)và(HAC) Do vậyB2C27→(AB0C0)

GọiIalà tâm cuả đường trịn ngoại tiếp4AB0C0 Khi ta cóIa∈da Mặt khácB2A1.B2C1=

B2B0.B2A=B2H.B2B; chứng tỏ rằngPB2/(Ia)=PB2/(BHC) Lập luận tương tự cho C2 Do vậy, gọi X, Y giao điểm cuả đường tròn ngoại tiếp tam giác AB0C0 BHC Ta có B2C2 ≡ XY Khi I(A, k) :X 7→X, Y 7→ Y Mặt khác H 7→ A, B 7→ C1, C 7→ B1; suy (HBC)7→ (A1B1C1) ⇒ X, Y ∈ (A1B1C1) Dẫn đến (A1B1C1),(Ia),(BHC) chùm đường tròn Để ý (A1B1C1) đường tròn Euler cuả tam giác ABC Do gọi E tâm cuả đường tròn Euler cuả 4ABC, ta thu E,Ia, Oa thằng hàng Hơn nữa,IaEOa ⊥

XY ≡B2C2 NhưngAIa ⊥B2C2 Suy raE ∈da

Lập luận tương tự chodb,dc Từ ta có đượcda, db, dc đồng quy tâm đường tròn Euler cuả 4ABC.

(21)

Bài tốn 9.(BMO 2007)Cho(O)là đường trịn vàAlà điểm nằm ngoài(O) GọiAB, AC

lần lượt là2 tiếp tuyến từA đếnBC Cho D giao điểm cuảOA và(O) GọiX hình chiếu từ

B lên CD Giả sử Y trung điểm cuả BX Z giao điểm thứ hai cuả DZ and (O) Chứng minh rằng: ∠AZC = 90◦

Lời giải

Gọi M trung điểm cuả BC Xét phép nghịch đảo cựcO, phương tích k=R2, đó Rlà bán kính cuả (O) Ta có I(O, k) : A7→M, D 7→D, Z 7→Z Vì vậyI(O, k) : (ADZ)7→(M DZ) Để ý rằngBD 7→(OBD)quaI(O, k) Ta dự đoán BD tiếp tuyến tạiD cuả(ADZ)

Thật vậy, vìD ≡OA∩(O)⇒∠DCO=∠CDO=∠DBO Do CDlà tiếp tuyến D cuả (OBD),(1)

Ta có ZDCB tứ giác nội tiếp⇒∠Y ZB=∠DCB≡∠XCB NhưngM, Y trung điểm cuả BC, BX ⇒M Y||CX ⇒ ∠Y M B = ∠XCB Vì ∠Y ZB =∠Y M B Suy ZY BM

nội tiếp⇒∠ZM B=∠XY D Mặt khác∠DM B=∠M Y X= 90◦⇒∠DM Z=∠DY M =∠XDY

(CX||M Y), điều nói lênDX tiếp tuyến tạiD cuả (DZM), (2)

Từ(1),(2), ta suy raCDX tiếp tyến chung tạiD cuả(DZM)và(OBD) Khi ấy(DZM)và (OBD)tiếp xúc chung tạiD Do đóBDlà tiếp tuyến(ADZ)⇒∠BDZ =∠DAZ ≡∠M AZ Mặt khác ∠BDZ =∠BCZ Suy ∠M AZ =∠BCZ ≡ ∠M CZ, điều chứng tỏ AZM C nội tiếp đường trịn Lại có∠AM C= 90◦ ⇒∠AZC= 90◦.

Thật ra, chứng minh đầy đủ cho toán bên phải bao gồm hai trường hợp: Trường hợp D nằm cung nhỏ cuả BC vàD nằm cung lớn cuảBC Lời giải cuả tác giả xét trường D nằm cung nhỏ cuả BC Còn lời giải cho trường hợp cịn lại lời giải cuả tác giả toán bên trên, phần chi tiết cuả chứng minh xin dành cho bạn đọc Bài toán 10.GọiO vàR tâm bán kính đường tròn ngoại tiếp4ABC Gọi Z

vàr tâm bán kính cuả đường trịn nội tiếp 4ABC Giả sử K trọng tâm cuả tam giác tạo bở điểm tiếp xúc cuả(Z)với cạnh cuả tam giác ABC Chứng minh rằngZ ∈OK

và OZ

ZK =

3R r

(22)

Gọi D, E, F điểm tiếp xúc cuả (Z) với cạnh BC, CA, AB Giải sử

Ma, Mb, Mc trung điểm cạnh EF, F D, DE Xét phép nghịch đảo cực Z, phương tích k = r2 Ta có I(Z, k) : D 7→ D, E 7→ E, F 7→ F A 7→ Ma, B 7→ Mb, C 7→ Mc Do I(Z, k) : (ABC)7→(MaMbMc) Để ý rằng(MaMcMc)là đường tròn9- điểm Euler cuả4DEF Do vậy, gọiO0 tâm cuả đường tròn (MaMbMc)⇒O0, Z, O thẳng hàng Mặt khác O0 nằm đường thẳng EulerZK cuả 4DEF Vì O, Z, K thẳng hàng

Vì (MaMbMc)là ảnh cuả(ABC)qua phép nghịch đảo cựcZ, phương tíchk, (MaMbMc) ảnh cuả(ABC)qua phép vị tự tâmZ, tỉ sốk1=

k

PZ/(O) = r

2

2Rr = r

2R ⇒ ZO0

ZO = r

2R Mặt

khác, xét trong4DEF, ta có4DEF ảnh cuả4MaMbMcqua phép vị tự tâmK, tỉ sốk2=−2, vậyH(K, k2) :O07→Z ⇒ KZ

KO0 = 2⇒

KZ ZO0 =

2 ⇒

KZ ZO =

r

3R

Bài toán cho ta thấy mối liên hệ phép vị tự phép nghịch đảo Bài tốn chắn có nhiều cách giải khác, song qua tư tưởng việc vận dụng mối liên hệ phép nghịch đảo vị tự cho ta cách nhìn sáng suả, tự nhiên tiếp cận vấn đề Bạn đọc gặp lại ý tưởng lời giải qua câu phần tập áp dụng

Ta kết thúc "chuyến đi" cuả toán qua điểm cố định Qua toán cuối này, bạn đọc thấy phép nghịch đảo tỏ hữu hiệu cho dạng tốn loại

Bài tốn 11.Cho(O)đường kínhAB ĐiểmItrên đoạn AB(khácA vàB) Một đường thẳng

dthay đổi quaI cắt(O)tạiP, Q(dkhông trùng vớAB) Đường thẳngAP, AQcắt tiếp tuyếnmtại

M, N, đóm tiếp tuyến B của(O) Chứng minh rằng(AM N) qua điểm cố định thứ hai, từ suy tâm của(AM N)ln nằm đường cố định

Lời giải

Xét phép nghịch đảoI(A, k), đók=AB2, ta cóI(A, k) : (O)7→m, đóP 7→M,

(23)

III - Đôi nét lịch sử

Trong cuốnTopics in Elementery Geometry Bottema kết phần phép nghịch đảo dịng đây, tơi xin trích nguyên văn sau:

"Inversion originated in the middle of the nineteenth century and was first researched exten-sively by Liouville (1847) Its great importance for elementary geometry is clear if we consider that it makes it possible to transform certain exercises in which circles are concerned, and in particu-lar many constructions, into less complicated ones where one or more circles have been replaced by a line For similar reasons, inversion was soon applied by physicists, for example by Thomson in the theory of electric fields The transformation is also important from a more theoretical point of view In analogy with what we have seen for affine and projective geometry, a conformal geometry or inversive geometry was developed, which only studies such notions and properties that are not only invariant for rigid motions and similarities, but also for inversions This geometry therefore includes the notions of circle and angle, but not that of line, radius, or center The figure of a triangle, that is, of three points, is not interesting in this geometry We can in fact prove that it is always possible to choose an inversion in such a way that three given points are mapped into three other given points, so that from the point of view of conformal geometry all triangles are “congruent” This is clearly not the case for quadrilaterals, since four points can either all lie on a circle, or not It is then no coincidence that we will use inversion to prove certain properties of quadrilaterals: these are in fact theorems from conformal geometry."

IV - Bài tập áp dụng

Bài a) Nếu(O, R), (I, r)thoả mãn hệ thức IO2=R2−2Rr thì chúng đường tròn ngoại tiếp, nội tiếp tương ứng tam giác

b) Nếu hai đường trịn(O, R),(I, r)thoả mãnIO2=R2+ 2Rr , hai đường trịn đường trịm ngoại tiếp bàng tiếp tam giác

Bài 2.(Định lý Feurebach) Chứng minh tam giác đường trịn chín điểm Euler tam giácABCtiếp xúc với đường tròn nội tiếp tam giác tiếp xúc với đường tròm bàng tiếp tam giácABC

Bài Cho tam giác ABC M điểm nằm tam giác, H trực tâm tam giác Các đường thẳng quaH vng góc vớiAM, BM, CM tạiBC, CA, ABlần lượt tạiA1, B1, C1 Chứng minh rằng:A1, B1, C1 thằng hàng

Bài 4.Cho tam giácABCvới điểmM điểm nằm tam giác Đường thẳng vng góc vớiM A, M B, M CtạiM cắtBC, CA, ABtại điểmA0, B0, C0 Chứng minh rằng:A0, B0, C0 thẳng hàng

Bài 5.Cho tam giác ABC có (I)là tâm đường trịn nội tiếp tam giác GọiA0, B0, C0 điểm tiếp xúc của(I)với BC, CA, AB Chứng minh tâm đường tròn(AIA0), (BIB0), (CIC0)thằng hàng

Bài 6.Cho tam giácABCcố định nội tiếp đường tròn(O).M, N hai điểm chạy trênAB, AC

sao cho khoảng cách hai hình chiếu củaM, NlênBCluôn

2 BC Chứng minh đường trịn ngoại tiếp tam giác4AM N ln qua điểm cố định khácA

Bài Cho 4ABC nhọn nội tiếp đường tròn (O) Gọi A1, B1, C1lần lượt hình chiếu

(24)

cắt đường trịn đường kínhAHtại Ia Chứng minh rằng:Ia, M0, N0, P0 đồng viên

Bài (CMO 2007) Cho4ABC nhọn nội tiếp(I) ngoại tiếp(O) Gọi A0, B0, C0 điểm tiếp xúc (I) với BC, CA, AB Gọi (Oa),(Ob),(Oc) đường tròn ngoại tiếp tiếp tam giácAB0C0,BC0A0,CA0B0 Giả sửA1là giao điểm thứ hai của(Oa)và(O),B1, C1 định nghĩa tương tự Chứng minh rằng:A0A1, B0B1, C0C1 đồng quy GọiN điểm đồng quy Chứng minhN nằm đường thẳng Euler tam giácA0B0C0

Bài (China TST 2009) Cho4ABC điểmD nằm cạnh BC thoả mãn∠CAD= ∠CBA Một đường tròn(O)đi quaB, C cắt cạnh AB, ADmột lần tạiE, F GọiGlà giao điểm củaBF vàDE Giả sửM trung điểm củaAG Chứng minh rằng:CM ⊥AG

Bài 10 (Serbia TST 2009) Choklà đường tròn nột tiếp tam giácABC không cân với tâm

S.ktiếp xúc vớiBC, CA, ABlần lượt tạiP, Q, R GọiM giao điểm củaQRvàBC Một đường tròn quaB, C tiếp xúc vớiktạiN Đường tròn ngoại tiếp tam giác4M N P cắtAP điểm thử hai làL Chứng minh rằngS, L, M thẳng hàng

Bài 11 (III AMP Olympiad, pro.2) Cho4ABC với trực tâmH GọiD chân đường cao từ

B xuống AC vàE điểm đối xứng củaAquaD Đường tròn ngoại tiếp4EBC cắt đường trung tuyến từAcủa4ABC F Chứng minh rằng:A, D, H, F đồng viên

Bài 12 (Iran Geometry exam 2004) Cho4ABC nội tiếp đường tròn tâm (O) GọiA1, B1, C1 giao điểm tiếp tuyến từ A, B, C đến (O) Gọi A3, B3, C3 trung điểm

BC, CA, AB Đường thẳng vng góc từA3 đếnAOcắt tiếp tuyến từ A1 của(O)tạiXa

Xb, Xc định nghĩa tương tự Chứng minh rằng:Xa, Xb, Xc thẳng hàng

Bài 13.(Chọn đội tuyển PTNK 2009) Cho đường thẳngdcố định,Alà điểm cố định nằm ngồid A0 hình chiếu củaA trênd B, C hai điểm thuộc dsao cho A0B.A0C =const (B, C khác phía với A) Gọi M, N hình chiếu củaA0 lên AB, AC Tiếp tuyến M, N đường trịn đường kínhAA0 cắt ởK Chứng minh rằngK nằm đường cố định

Bài 14 (Đề đề nghị Olympic truyền thống 30/4) Cho đường tròn(O, R)tiếp xúc vớidtại H

cố định.M, N hai điểm di động trêndsao choHM HN =−k <0,k=cosnt Từ M, N vẽ hai tiếp tuyếnM A, N B tới(O) Chứng minh rằng:ABluôn qua điểm cố định

Bài 15 (Đề đề nghị Olympic truyền thống 30/4- 2008) Cho tam giác ABC có đường trung tuyếnAM, đường caoBD, CE Giả sửP giao điểm củaDEvàAM Giả sửAM= BC

2 Chứng minh rằngP trung điểm củaAM

Bài 16Cho tam giácABC nhọn nội tiếp(O) GọiA0, B0, C0lần lượt hình chiếu cuảA, B, C trênBC, CA, AB tương ứng Giải sử A1, B1, C1 giao điểm thứ hai cuả đường tròn ngoại tiếp tam giác4AB0C0, 4BC0A0 và4CA0B0 với (O) A2, B2, C2 giao điểm thứ hai cuả trung tuyến kẻ tứA, B, C đến(O) Chứng minh A2A1, B2B1, C2C1 đồng quy

Tài liệu tham khảo

(25)(26)

Applying R, r, p − method in some hard problems

Tran Quang Hung, Hanoi National University1

Introduction

In this article we will useR, r, p- method to prove some inequalities in triangle

Given triangleABC, we denote bya, b, cthe sides, semiperimeterp, circumradiusR, inradiusr, centroidG, incenter I, orthocenterH, we have following famous formulas:

ab+bc+ca=p2+r2+ 4Rr a2+b2+c2= 2(p2−r2−4Rr)

OI2=R2−2Rr⇒R≥2r, this formula is well-known as Euler’s.

9IG2=p2−16Rr+ 5r2⇒p2≥16Rr−5r2, this inequality is well-known as Gerretsen’s IH2= 4R2+ 3r2+ 4Rr−p2⇒p2≤4R2+ 4Rr+ 3r2

OH2= 9OG2= 9R2−(a2+b2+c2)⇒a2+b2+c2≤9R2

I - Garfunkel’s lemmas

In this section we introduce the useful lemma which was proposed by Jack Garfunkel in Crux Math

Lemma LetABC be acute triangle prove that

p2≥2R2+ 8Rr+ 3r2

Proof

The inequality equivalent to

sin2A+ sin2B+ sin2C≥(cosA+ cosB+ cosC)2

LetH be orthocenter of triangleABC, we haveHA= 2RcosA, HB= 2RcosB, HC = 2RcosC

and by law of sinea= 2RsinA, b= 2RsinB, c= 2RsinCwe need to prove

a2+b2+c2≥(HA+HB+HC)2 LetA0B0C0 be Cevian triangle2 ofH we have

AA0.AH+BB0.BH+CC0.CH =a

2+b2+c2

2 and

HA AA0 +

HB BB0 +

HC CC0 = Now apply Cauchy-Schwartz inequality we get

a2+b2+c2= (AA0.AH+BB0.BH+CC0.CH)(HA

AA0 +

HB BB0 +

HC

CC0)≥(HA+HB+HC)

2

We are done

1Email:hung100486@yahoo.com

(27)

Lemma 2.Let ABCbe acute triangle such that π

4 ≤min{A, B, C} ≤max{A, B, C} ≤

π

2, prove that

p2≤3R2+ 7Rr+r2

Proof

Using the indentities of R, r, p

cosA+ cosB+ cosC= R+r

r

cosAcosBcosC=p

2−4R2−4Rr−r2

4R2

The inequality is equivalent to

3(cosA+ cosB+ cosC)≥4 + cosAcosBcosC

Because π

4 ≤min{A, B, C} ≤max{A, B, C} ≤

π

2 we can assume that

π

4 ≤A≤

π

3, therefore

(2 sin2A + sin

A

2 −1)(2 sin

A

2 −1)≥0 ⇔3 cosA+ sinA

2 −4 cosAsin

2A

2 ≥4 (1)

We have |B−C|< π

2 so

4 cosA(1 + cosB−C

2 )>2(1 + √

2)>6>6 sin

A

2 therefore

4 cosA(1 + cosB−C

2 )≥2(1 + √

2)(1−cos

B−C

2 )≥6 sin

A

2(1−cos

B−C

2 )

⇔3(cosB+ cosC)−6 sinA

2 + cosAsin

2A

2 ≥4 cosAcosBcosC (2) From (1), (2) we have

3(cosA+ cosB+ cosC)≥4 + cosAcosBcosC

We are done

Note that When we apply Garfunkel’s lemma in acute triangle then we obtain the inequality sinA

2 + sin

B

2 + sin

C

2 ≥

1 + sinA sin B sin C

This inequality was propesed by Jack Garfunkel in Crux Math., vol 10 1984, problem 987, there-fore we call it by Garfunkel’s lemma

II - Some problems

In this section, using standar notations in triangle, we will show somes problems solved byR, r, p

(28)

Problem 1.LetABC be a triangle with areaS, prove that 2(ab+bc+ca)−(a2+b2+c2)≥4

a2(p−a) b+c +

b2(p−b) c+a +

c2(p−c) a+b

≥4√3S

Proof

First at all, we will say something about these inequalities,

2(ab+bc+ca)−(a2+b2+c2)≥4√3S⇔a2+b2+c2≥4√3S+ (b−c)2+ (c−a)2+ (a−b)2 It is well-known under the name Finsler - Hadwiger inequality, and the inequality

a2(p−a) b+c +

b2(p−b) c+a +

c2(p−c) a+b ≥

√ 3S

⇔ a

ra+ha

+ b

rb+hb

+ c

rc+hc ≥√3

It makes Finsler-Hadwiger stronger, we now solve the first by the follow indentities of R, r, p

2(ab+bc+ca)−(a2+b2+c2) = 16Rr+ 4r2 a2(p−a)

b+c +

b2(p−b) c+a +

c2(p−c) a+b =

r((3r+ 2R)p2−r3−6r2R−8R2r) p2+r2+ 2Rr

The first inequality equivalent to

2(ab+bc+ca)−(a2+b2+c2)≥4

a2(p−a) b+c +

b2(p−b) c+a +

c2(p−c) a+b

⇔16Rr+ 4r2≥8r((3r+ 2R)p

2−r3−6r2R−8R2r) p2+r2+ 2Rr

⇔ r(18Rr+ 24R

2+ 3r2−5p2)

p2+r2+ 2Rr ≥0⇔5p

2≤24R2+ 18Rr+ 3r2

Now using p2≤4R2+ 4Rr+ 3r2we must to show

5(4R2+ 4Rr+ 3r2)≤24R2+ 18Rr+ 3r2⇔2(2R+ 3r)(R−2r)≥0 Which is true Now for the second, we easily seen

a2(p−a) b+c +

b2(p−b) c+a +

c2(p−c) a+b

2

≥3Xb

2c2(p−b)(p−c)

(a+b)(a+c) We will prove that

Xb2c2(p−b)(p−c) (a+b)(a+c) ≥S

2=p2r2

We have

Xb2c2(p−b)(p−c) (a+b)(a+c) −p

2r2= r3((r−8R)p2+r3+ 10r2R+ 32R2r+ 32R3) p2+r2+ 2Rr

Therefore, it is sufficient to show that

(r−8R)p2+r3+ 10r2R+ 32R2r+ 32R3≥0⇔p2≤ r

3+ 10r2R+ 32R2r+ 32R3

(29)

but using inequalityp2≤4R2+ 4Rr+ 3r2, we only must show

4R2+ 4Rr+ 3r2≤r

3+ 10r2R+ 32R2r+ 32R3

8R−r ⇔

2r(2R−r)(R−2r) 8R−r ≥0

Which is true

Problem 2.Let ABC be a triangle andAA0, BB0, CC0 are bisector of ABC prove that

p(A0B0C0)≤

4p(ABC) here p(XY Z)show perimeter of triangleXY Z

Proof

We will prove stronger form of this inequality

B0C02+C0A02+A0B02≤ p

2

3

Whenpis semi-perimeter of triangleABC, indeed, we can use the following indentity ofR, r, p, whenAA0, BB0, CC0 are bisector of triangleABC then

p2

3 −B

0C02+C0A02+A0B02= p2

3 −

8Rr2((7R+ 8r)p2−4R2r−r2R) p4+ 4R2r2+ 4p2Rr+ 4r3R+r4+ 2p2r2

= (96R

3r3+ 24R2r4) + (−188r3R−164R2r2+r4)p2+ (2r2+ 4Rr)p4+p6

3(p4+ 4R2r2+ 4p2Rr+ 4r3R+r4+ 2p2r2)

Therefore, it is sufficient to show that

(96R3r3+ 24R2r4) + (−188r3R−164R2r2+r4)p2+ (2r2+ 4Rr)p4+p6≥0 It is equivalent to

4r3(R−2r)(648R2−237Rr+ 10r2) + 4r2(183R2−161Rr+ 14r2)(p2−16Rr+ 5r2) +13r(4R−r)(p2−16Rr+ 5r2)2+ (p2−16Rr+ 5r2)3≥0

⇔4r3(R−2r)(2128r2+2355r(R−2r)+648(R−2r)2)+4r2(424r2+571r(R−2r)+183(R−2r)2)13r(4R−r) +(p2−16Rr+ 5r2)2+ (p2−16Rr+ 5r2)3≥0

Which is true Now from this inequality we easily seen

(A0B0+B0C0+C0A0)2≤3(B0C02+C0A02+A0B02)≤p2

⇔p(A0B0C0)≤

4p(ABC)

Problem 3.(Proposed by Ji chen) LetABC be a triangle,ra, rb, rc are exradius, prove that

ra2

a2 + r2b b2 +

rc2

c2 ≥

9 +

15(b−c)2(c−a)2(a−b)2 4a2b2c2

Proof

Use the indentities of R, r, pwe have

r2

a

a2 + r2

b

b2 + r2

c

c2 −

9 =

(256rR3+ 16r3R+ 96R2r2+ 256R4+r4) + (2r2−68R2)p2+p4

(30)

and

15(b−c)2(c−a)2(a−b)2

4a2b2c2 =−

15(64rR3+ 48R2r2+ 12r3R+r4) + (−4R2−20Rr+ 2r2)p2+p4

16p2R2

Therefore we need to prove that

(256rR3+ 16r3R+ 96R2r2+ 256R4+r4) + (2r2−68R2)p2+p4

≥ −15(64rR3+ 48R2r2+ 12r3R+r4) + (−4R2−20Rr+ 2r2)p2+p4

⇔(304rR3+ 49r3R+ 204R2r2+ 64R4+ 4r4) + (8r2−32R2−75Rr)p2+ 4p4≥0 Now using p2≤4R2+ 4Rr+ 3r2it is equivalent to

4r(4r+R)(R−2r)2+r(43R−32r)(4R2+ 4Rr+ 3r2−p2) + 4(p2−4R2−4Rr−3r2)2≥0 Which is true

Note that This inequality is form of famous inequality Iran 96 as following

Xr2a

a2 =

X p

2tan2A

2 16R2sin2A

2 cos 2A = 16 X sinA    X

cos4A    =

cos2A cos 2B cos 2C    X

cos4A   = Xcos 2B cos 2C cos2A

2 So it is equivalent to

Xcos

2B

2 cos

2C

2 cos2A

2

≥ +

15(b−c)2(c−a)2(a−b)2 4a2b2c2

Usingcos2A =

p(p−a)

bc , it is equivalent to

Xp(p−b)(p−c)

a2(p−a) ≥

9 4+

15(b−c)2(c−a)2(a−b)2

4a2b2c2

Now we can replace a=xy+xz, b=yz+yx, c=zx+zy,∀x, y, z >0it is equivalent to

(xy+yz+zx)

1

(x+y)2 +

1 (y+z)2 +

1 (z+x)2

4+

15(x−y)2(y−z)2(z−x)2

4(x+y)2(y+z)2(z+x)2

This is stronger than Iran 96 inequality

Problem (Proposed by Jack Garfunkel, problem 825 Crux Math.) Let ABC be a triangle, prove that

tan2A + tan

2B

2 + tan

2C

2 + sin

A

2 sin

B

2 sin

C

2 ≥2 Proof

Using the indentities of R, r, pwe have tan2A

2 + tan

2B

2 + tan

2C

2 =

(31)

sinA sin B sin C = r 4R

It is equivalent to

(4R+r)2−2p2

p2 +

2r

R ≥2⇔p

2≤ R(4R+r)

2(2R−r)

Using the result of Mittenpunkt3(middlespoint)M of triangleABC, note thatM(a(p−a), b(p−

b), c(p−c))in4 , withO is circumcenter and using formula of distance, we can compute

M O2=2(r−2R)p

2+R(4R+r)2

(4R+r)2

From M O2≥0 we easily seenp2

≤R(4R+r)

2

2(2R−r)

Note that p2≤ R(4R+r)

2

2(2R−r) ≤4R

2+ 4Rr+ 3r2, where the second inequality is equivalent to

3r2(R−2r) 2(2R−r) ≥0

Problem 5.Let ABC be a triangle prove that cosA

2 + cos

B

2 + cos

C ≥ √ sinA

2 + sin

B

2 + sin

C

2

Proof

First at all we will prove the following inequality for acute triangle sinA+ sinB+ sinC≥√2

3(cosA+ cosB+ cosC)

2

Actually, this problem was proposed by Jack Garfunkel in Crux Math 1990 and the solution of author in Curx Math 1991, here is our solution

The inequality is equivalent to

p R ≥

2 √

(R+r)2

R2 ⇔p

2≥ 4(R+r)

3R2

Now apply Garfunkel’s lemmap2≥2R2+ 8Rr+ 3r2 we must show

2R2+ 8Rr+ 3r2≥4(R+r)

4

3R2 ⇔

(R−2r)(2R3+ 12R2r+ 9Rr2+ 2r3)

3R2 ≥0

which is true, so we are done

After that we introduce here the solution by Jack Garfunkel himseft, We note that sinA

2 sin B sin C ≤ therefore

(cosA+ cosB+ cosC)2=

1 + 4YsinA

= + 8YsinA + 16

Y sin2A

2 ≤1 + 10 Y

sinA 3http://mathworld.wolfram.com/Mittenpunkt.html

(32)

So me must prove that

sinA+ sinB+ sinC≥ √2

1 + 10YsinA

It is equivalent to

3p≥2R+ 5r⇔3p2≥4R2+ 20Rr+ 25r2

But by Garfunkel’s lemmap2≥2R2+ 8Rr+ 3r2, we must show

3(2R2+ 8Rr+ 3r2)≥4R2+ 20Rr+ 25r2⇔(R−2r)(4R+r)≥0 which is true, so we are done

Now we apply

sinA+ sinB+ sinC≥√2

3(cosA+ cosB+ cosC)

2

in acute triangle with angle π−A ,

π−B

2 ,

π−C

2 , we obtain our problem

Problem (Proposed by Virgil Nicula on Mathlinks.ro) Let ABC be three side of an acute triangle prove that

r

b+c−a

a +

r

c+a−b

b +

r

a+b−c

c ≥3

Proof

Note that sin2A =

(p−b)(p−c)

bc , and r= 4Rsin A

2 sin

B

2 sin

C

2, therefore

a b+c−a =

a

2(p−a) = 2R

r sin 2A

2 Now we can write the inequality as

X r r 2R sinA

≥3⇔2 sinA sin B sin C    X

sin2A

+ 2X

sinA sin B   ≥9

⇔X sinB sin C sinA

+ 2XsinA ≥ ⇔ X sinB sin C    X sinA   ≥

Now need to prove the inequality X

cosBcosC

X cosA ≥

for triangleABC such that π

2 ≥max{A, B, C} ≥min{A, B, C} ≥

π

4, indeed, write it inR, r, p, note that

X

cosBcosC= p

2−4R2+r2

4R2 ,

X

cosA =

p2−4R2+r2 p2−4R2−4Rr−r2

We need to prove that

p2−4R2+r2

4R2

p2−4R2+r2 p2−4R2−4Rr−r2 −

(33)

⇔10R2r2+ 72R3r+r4+ 88R4+ (−26R2+ 2r2)p2+p4≥0 Note that in triangle ABC such that π

2 ≥ max{A, B, C} ≥ min{A, B, C} ≥

π

4 then we have Garfunkel’s Lemmap2≤3R2+ 7Rr+r2, the above inequality equivalent to

(r2+ 8Rr+ 19R2)(R−2r)2+ (48r2+ 66r(R−2r) + 20(R−2r)2)(3R2+ 7Rr+r2−p2) +(p2−3R2−7Rr−r2)2≥0

Which is true, so we are done

Note that There are some nice equivalent form of this problem, letI be incenter ofABC r

b+c−a

a +

r

c+a−b

b +

r

a+b−c

c ≥3

⇔X

sinA

≥6 r

R

2r ⇔IA+IB+IC ≥3

√ 2Rr

Problem 7.(Proposed by Jack Garfunkel, Crux Math.)Prove in acute triangle ABC we have cosA

2 + cos

B

2 + cos

C

2 ≥ √

1 + sinA sin

B

2 sin

C

2

Proof

First at all we will prove the inequality in triangle such ABC that π

2 ≥ max{A, B, C} ≥ min{A, B, C} ≥ π

4:

sinA+ sinB+ sinC≥ √4

3(1 + cosAcosBcosC) It is equivalent to

p R ≥

4 √

p2−4Rr−r2

4R2 ⇔3R

2p2≤(p2−4Rr−r2)2

⇔(16R2r2+r4+ 8r3R) + (−2r2−8Rr−3R2)p2+p4≥0 Which is true, so we are done

References

[1]Crux Mathematicorum, vol 10 - 1984, Canadian Mathematical Society [2]Crux Mathematicorum, vol 12 - 1986, Canadian Mathematical Society

(34)

Các phương pháp tính tích phân

Nguyễn Văn Vinh, Đại học Tổng hợp Quốc gia Belarus

Trong số trước làm quen với số kĩ thuật tính tích phân suy rộng, tích phân hàm phần nguyên, phần lẻ, phép biến đổi Laplace, Fourier hay khai triển tích phân thành chuỗi Trong viết tìm hiểu thêm vài phương pháp khác mà chủ yếu phương pháp tích phân tham số, phương pháp hiệu ta áp dụng giải tốn, bên cạnh ứng dụng hàm Gamma, Beta để tính tích phân

I - Phương pháp tích phân tham số

Về tính chất loại tích phân hầu hết giáo trình giải tích điều có, nên khơng xét lại Trong số tính chất có hai tính chất quen thuộc mà ta hay áp dụng vi phân tích phân tích phân chứa tham số Chúng ta chủ yếu xoay quanh hai tính chất số ý nhỏ khác để thu kết Điều kiện để tồn tích phân thí dụ khơng phức tạp xin bỏ qua, bạn tự kiểm tra

Trước tiên ta xem xét ứng dụng tính chất đạo hàm tích phân chứa tham số Chúng ta bắt đầu ví dụ đơn giản

Ví dụ Tính

I(m, n) =

1

Z

0

xm(lnx)ndx

Lời giải.Khi áp dụng tính chất đạo hàm ta cần ý có hai đường để áp dụng, xuất phát từ tích phân đơn giản biết sau vi phân liên tục theo biến mà ta cần để thu kết quả, đôi lúc ta lại làm theo chiều ngược lại đạo hàm tích phân cần tính theo biến thích hợp để thu tích phân đơn giản hơn, áp dụng kĩ thuật thứ

Ta có

1

Z

0

xmdx=

m+

Đạo hàm hai vế tích phân theo mta thu d

dm

Z

0

xmdx=

1

Z

0

xmlnxdx=− (m+ 1)2

Lặp lạinlần bước làm ta thu

Z

0

xm(lnx)ndx= (−1)n n! (m+ 1)n+1

Một ví dụ cho phép ứng dụng thứ hai

Ví dụ Tính

I(λ) =

λ

Z

0

(35)

Lời giải Ta đạo hàmI(λ)theoλthu

I0(λ) =

λ

Z

0

x

(1 +λx) (1 +x2)dx+

ln(1 +λ2) +λ2

Bằng kĩ thuật tính tích phân thường ta thu λ

Z

0

x

(1 +λx) (1 +x2)dx=−

ln(1 +λ)2

(1 +λ2) +

ln(1 +λ2)

2 (1 +λ2) + λ

1 +λ2arctanλ

Từ ta có

I0(λ) =ln(1 +λ

2)

2(1 +λ2) + λ

1 +λ2arctanλ Từ biểu thức cuối ta thu

I(λ) =

2arctanλln(1 +λ

2)

Trong số trường hợp ta áp dụng kĩ thuật thứ hai tích phân tham số ta cần tính thoả mãn phương trình tích phân

Ví dụ Tính

I(λ) =

Z

0

e−x2−λ2/x2dx

Lời giải Ta thấy sau đạo hàm tích phân ta cần tính thoả mãn phương trình tích phân tuyến tính

I0(λ) + 2I(λ) =

Nghiệm phương trình giá trị tích phân ta cần tính I(λ) =

π

2 e

−2λ

Tương tự ta dễ dàng kiểm tra

y(x, µ) =

π

Z

0

eµxcosξdξ

là nghiệm phương trình vi phân

xy00+y0−µ2xy=

Tuy nhiên ta làm tốn số tích phân khơng chứa tham số mà có nhiều hơn, địi hỏi phải tiến hành đạo hàm nhiều lần chọn biến cho thích hợp

Ví dụ Tính

I(a, b) =

+∞

Z

0

arctanax arctanbx

(36)

Lời giải.Ta thấy tích phân cần tính chứa hai tham số khác nên để tính tiến hành đạo hàm theo hai tham số thu

Iab00(a, b) = π (a+b)

Từ ta tìm

I(a, b) =π

2 (a+b) (ln(a+b)−1) +φ(a) +ψ(b)

Từ tính liên tục tích phân cần tìm ta thu φ(a) +ψ(b) = π

2 (b(1−lnb) +a(1−lna))

Do ta có

I(a, b) =π

2 sgn (ab) ln

(|a|+|b|)|a|+|b| |a||a||b||b|

Áp dụng kĩ thuật ta giải số tốn thú vị

Ví dụ (SIAM-08-001) Tính

X

n=1 Ci(an)

n2

Lời giải.Đây công thức hay, số phép biến đổi ta nhanh chóng chuyển tốn tính

I(a) =

∞ X n=1 n2   n Z

cosat - t dt

Để tính biểu thức cuối ta đạo hàm hai vế theo a thu

I0(a) =−

∞ X n=1 n Z sinat n2 dt=

−π + a

Từ ta có

I(a) =−πa

2 +

a2

8

Cuối ta thu công thức chuỗi cần tìm đẹp ∞

X

n=1 Ci(an)

n2 = π2lna

6 −ζ

0(2) +π2γ

6 −

πa

2 +

a2

8

Thêm ví dụ hay

Ví dụ (CMJ-C904) Tính

I= Z Z

(37)

Lời giải Bằng số phép đổi biến chia miền tích phân bội cho ta thu

I=

2

Z

1

ln Γ(u)du+

1

Z

0 uln1

udu=

Z

1

ln Γ(u)du+1

Ta cần tính tích phân

2

Z

1

ln Γ(u)du

Để tính tích phân ta chuyển qua tính tích phân chưa tham số có dạng

I(p) =

p+1

Z

p

ln Γ(u)du (p≥0)

Khip= 0ta dễ dàng tính

I(0) = ln 2π

Khip≥1ta có đạo hàmI(p)theopthì thu

I0(p) = ln Γ(p+ 1)−ln Γ(p) = lnp Từ ta rút

I(p) =

p

Z

0

lnxdx+I(0) =p(lnp−1) +ln 2π

Từ điều ta thu kết cuối

I=

1

Z

0

Z

0

ln Γ(x+y)dxdy =ln 2π

2 −

3

Trong số trường hợp tích phân cần tính khó mà lại khơng chứa tham số ta cần linh hoạt thêm tham số phù hợp đảm bảo tính hội tụ tích phân ví dụ giúp ta tính tốn đơn giản

Ví dụ Tính tích phân

I=

Z

0

sinx x dx

Lời giải Ta thấy để tính trực tiếp tích phân khó nên ta chuyển qua tính tích phân sau

I(a) =

Z

0

e−axsinx x dx

Chú ý tích phân hội tụ với mọia >0 Để tính tích phân đơn giản ta đạo hàm theo tham sốavà thu

(38)

Từ ta rút

I(a) = π

2 −arctana

Và tích phân ta cần tính

I=π

Ở xem xét số ví dụ ứng dụng phép đạo hàm tích phân tham số Tiếp theo ta xem qua số kĩ thuật phép tích phân tích phân tham số

Ví dụ Tính tích phân

I(a, b) =

1

Z

0

xb−xa

lnx sin ln

1

xdx (a >0, b >0)

Lời giải Để tính tích phân ta khơng áp dụng phép đạo hàm tích phân chứa tham số mà ta chuyển tích phân cần tính dạng tích phân kép dạng tham số

Ta viết lại tích phân cần tính dạng

I(a, b) =

1

Z

0 dx

b

Z

a

xpsin ln1

xdp

Ta viết lại tích phân dạng

I(a, b) =

b

Z

a dp

1

Z

0

xpsin ln1

xdx

Đặt

I1(p) =

1

Z

0

xpsin ln1

xdx Ta có

I1(p) =

+∞

Z

0

e−(p+1)tsintdt= (p+ 1)2+ 1 Từ tích phân ban đầu cần tìm

I(a, b) =

b

Z

a

I1(p)dp= arctan(b+ 1)−arctan(a+ 1) = arctan

b−a

1 + (a+ 1)(b+ 1)

Một ví dụ ta quen biết phép tích phân tích phân tham số giải phương trình tích phân Abel

Ví dụ Giải phương trình tích phân Abel có dạng x

Z

0

φ(t)

(39)

Lời giải.Ta thấy tích phân vế trái tích phân phụ thuộc hai tham số, để giải phương trình tích phân Abel ta cần tác động thêm tích phân theo tham số hàm vế trái

Nhân hai vế phương trình cho với ds

(x−s)1−α chuyển hai tham số tích phân vế trái thànhα, svà tích phân hai vế theostừ0→xta thu

x

Z

0

ds

(x−s)1−α s

Z

0 φ(t) (s−t)αdt=

x

Z

0

f(s) (x−s)1−αds Biến đổi vế trái ta thu

x

Z

0 φ(t)dt

s

Z

t

ds

(x−s)1−α(s−t)α = x

Z

0

f(s) (x−s)1−αds Từ ta dễ dàng thu

x

Z

0

φ(t)dt= sinαπ

π x

Z

0

f(s) (x−s)1−αds Từ ta có nghiệm phương trình Abel

φ(x) = sinαπ

π

f(0)

x1−α + x

Z

0

f0(s) (x−s)1−αds

Sử dụng kĩ thuật tích phân tích phân có chứa tham số ta tính nhiều tích phân quen thuộc Laplace, Lipchizt, Dirichlet,

Bên cạnh số tích phân thơng thường có nhiều tích phân đặc biệt hàm đặc biệt có dạng biểu diễn dạng tích phân tham số Một số tích phân Eliptic với nhiều dạng biểu diễn khác có khơng ứng dựng liên quan đến lớp tích phân Vì vấn đề lớn nên có dịp trở lại viết riêng “Tích phân Eliptic”

Bài tập áp dụng Tính tích phân sau Bài

I(α) =

+∞

Z

0

dx

(x2+α)n+1, n∈N, α >0 Bài

I(a, b) =

+∞

Z

0

e−ax2−e−bx2

x dx, a >0, b >0 Bài

I(m, a, b) =

+∞

Z

0

e−ax−e−bx

x sinmxdx, ∀m∈R, a >0, b >0 Bài

I(a) =

1

Z

0

ln(1−a2x2)

(40)

Bài

I(a) =

+∞

Z

1

arctanax

x2√x2−1dx, ∀a∈R II - Hàm Gamma, Beta

Như biết với phát triển toán học số lượng hàm tốn mở rộng, nhiều hàm đặc biệt đóng vai trị to lớn giải tích lĩnh vực khác mà nói hai hàm Gamma, Beta hàm với ứng dụng Mức độ áp dụng hai hàm vấn đề tính tốn vi tích phân lớn nhiên viết có tính giới hạn nên nêu số ví dụ nói hay để qua thấy cách áp dụng phép biến đổi

Ví dụ 10 (SSMJ-5073) Tính

1

Z

0

{lnx}xmdx

vớim >−1

Lời giải Bằng số biến đổi tích phân phần lẻ ta chuyển tích phân cần tính dạng

Z

0

{lnx}xmdx=

e1+m−1− +∞

Z

0

te−t(m+1)dt

Ta ý tích phân thứ hai dễ dàng biểu diễn qua hàm Gamma thu kết cuối

1

Z

0

{lnx}xmdx=

(e1+m−1) (1 +m)−

Γ(2) (m+ 1)2 =

1

(e1+m−1) (1 +m)−

1 (m+ 1)2

Tuy nhiên số tốn địi hỏi ta có biến đổi linh hoạt để thu biều diễn hàm Gamma

Ví dụ 11 (Crux-3386) Tính ∞ Z e−x   x Z

e−t−1

t dt

lnxdx

Lời giải Bằng kĩ thuật khai triển chuỗi ta dễ dàng thu ∞ Z e−x   x Z

e−t−1

t dt

lnxdx=

X

n=1

(−1)n

n!n ∞

Z

0

e−xxnlnxdx

Ta thấy tích phân tổng chuỗi biểu diễn thong qua đạo hàm hàm Gamma từ ta dễ dàng tính tổng

∞ Z e−x   x Z

e−t−1

t dt

lnxdx=

X

n=1

(−1)nΓ0(n+ 1)

(41)

Ứng dụng hai hàm ta thu nhiều cơng thức biểu diễn tích phân chuỗi đẹp ví dụ cơng thức

Ví dụ 12 Chứng minh

1 (k−1)!

1

Z

0

(1−t)k−1f(ta)dt=

+∞

X

n=0

cn

(na+ 1) (na+ 2) (na+k)

Trong đóa >0 vàf(x) =

P

n=0

cnxn,∀x∈(−1,1)

Lời giải Thay hàmf(x)biểu diễn dạng chuỗi vào tích phân vế trái ta có

1 (k−1)!

1

Z

0

(1−t)k−1f(ta)dt= (k−1)!

+∞ X n=0 cn Z

tan(1−t)k−1dt= (k−1)!

+∞

X

n=0

cnB(an+ 1, k)

Sử dụng tính chất hàm Beta ta thu điều phải chứng minh

Nói đến ứng dụng hàm Gamma, Beta ta bỏ qua tích phân Dirichlet dạng biểu diễn

Ví dụ 13 Cho Vk.kn,r

p=

n

(x1, , xn)∈Rn, xi≥0, ∀i= 1, n0≤ kx1, , xnkp≤r o

vàn∈N, p≥1, r >0, β≥1, αi>0

Tính tích phân sau

Z

Z

V

xα11−1 xαn−1n

1−x

p

1+ +x p n rp

β−1

dx1dx2 dxn

Lời giải Sử dụng tích phân Dirichlet ý B(x, y) =Γ(x)Γ(y)

Γ(x+y),Γ(1 +x) =xΓ(x)

Ta thu công thức tổng quát đẹp

Z

Z

V

xα11−1 xαn−1n

1−x

p

1+ +xpn rp

β−1

dx1dx2 dxn=

Γ(β)rα1+ +αn n

Q

i=1

Γ (1 +α1/p) Γ (β+ (α1+ +αn)/p)

n

Q

i=1 αi

Các bước biến đổi trung gian bạn đọc xem thêm coi tập vận dụng

Hàm Gamma Beta sử dụng làm định nghĩa cho lượng lớn hàm toán học Bessel, PolyGamma, HyperGeometric,

(42)

Bài tập áp dụng Tính tích phân sau Bài

π/2

Z

0

(sinφ+cosφ)3sin−1/2φcos−1/2φdφ

Bài

2

Z

0

dx

p

x2(2−x) Bài

I(p) =

+∞

Z

0

xp−1lnx

1 +x dx, ∀p∈(0,1) Bài

I(p) =

+∞

Z

0

xpe−qxlnxdx (q >0)

Bài 10

1

Z

0

ln1

x

p

dx

Trong viết tiếp đến xem xét ứng dụng hàm Gamma, Beta hàm đặc biệt khác với phép tính thặng dư

Tài liệu tham khảo

1 Gorbuzov V.N.;Giải tích Tốn học: Tích phân phụ thuộc tham số (Tiếng Nga), Grodno 2006 2.Tuyển tập MathVn - Các kĩ thuật biến đổi vi tích phân, 2009 (preprint)

3.SIAM, Problems and Solutions.Link:http://www.siam.org/journals/problems.php 4.Các tạp chí CMJ, CRUX, SSMJ,

(43)

Bài toán Kakeya

Mạch Nguyệt Minh1- University of Pisa, Italy Phan Thành Nam2- University of Copenhagen, Denmark

I - Nhốt voi vào tủ lạnh!

Có lần chúng tơi gặp câu đố vui rằng: "Làm để nhốt voi vào tủ lạnh?" Thú thật, đến đáp án câu đố Tuy nhiên, xem Toán học xem xét vấn đề

"Nhốt voi vào tủ lạnh!" Một cách Tốn học, phát biểu có nghĩa đặt vật "lớn" tùy ý vào bên thể tích cho trước? Tất nhiên, phép co, câu hỏi tương đương với: làm để đặt vật cho trước vào thể tích "bé" tùy ý Điều nghe khơng tưởng, bạn đừng vội phản đối trước định nghĩa lớn, bé, và, tất nhiên, thể tích

Để đơn giản, xét hình trịn đường kính đơn vị mặt phẳng hai chiều Một đặc trưng cho "lớn" hình trịn là: chứa đoạn thẳng độ dài đơn vị với phương tùy ý Một cách hình ảnh, tưởng tượng đoạn thẳng độ dài đơn vị kim Vậy hình trịn đường kính đơn vị cho phép kim quay đủ vịng3600 mà khơng ngồi hình trịn

Năm 1917, nhà Toán học Nhật Bản Soichi Kakeya đặt câu hỏi sau

Kakeya needle problem Diện tích bé mà cho phép quay đoạn thẳng độ dài đơn vị cách liên tục đủ 3600 (mà khơng ngồi diện tích đó)?

Trong ví dụ đường trịn đường kính đơn vị nói cần diện tích π/4 ≈0.785 Một ví dụ khác, khơng tầm thường, xét tam giác đềuABC có chiều cao (xem Hình 1), từ đoạn thẳngAA1 (độ dài 1) ta cố địnhAvà quay 600 để thành đoạn thẳng AA2, sau tịnh tiến đoạn thẳng để biến thànhCC1, quay 600 để đượcCC2 Như tồn diện tích sử dụng là|ABC|= 1/√3≈0.577 (ta ký hiệu|S|cho diện tích hìnhS)

Tuy nhiên, Kakeya nhận chí tiết kiệm diện tích quay đoạn thẳng hình tam giác cong deltoid (Hình 2): hình hypocycloid vạch điểm cố định đường trịn bán kính 1/2 ta lăn đường trịn bán kính 3/2 Diện tích hình bằngπ/8 ≈0.393 Chú ý hình tam giác cong khoảng cách từ đỉnh tới cạnh đối diện Các bạn thử hình dung quay kim bên diện tích nào? (có thể xem [11])

(44)

Kakeya giả thuyết hình có diện tích bé Năm 1925, G.D Birkhoff viết toán mở sách ông, trước hết đề cập tới tốn bốn màu, sau ơng thêm vào "Cùng đơn giản lôi câu hỏi nêu cách vài năm nhà Toán học Nhật Bản Kakeya." [1]

Năm 1928, Abram Samoilovitch Besicovitch, nhà Toán học Nga−Do Thái, giải tốn theo lối đáng kinh ngạc: ơng chứng minh đáp số cho toán Kakeya "zero" Chính xác hơn, ơng tìm hình có diện tích bé tùy ý mà cho phép quay đoạn thẳng đơn vị cách liên tục đủ vịng [2] Thậm chí bỏ yêu cầu "quay liên tục" tồn tập hợp có độ đo (độ đo độ đo Lebesgue, khơng có thích khác) mà chứa đoạn thẳng độ dài đơn vị theo hướng Một tập hợp ngày gọi tập Besicovitch Thật ra, Besicovitch quan tâm đến toán nguyên độc lập với Kakeya Trên báo tiếng Nga năm 1920 ông xem xét câu hỏi tích phân Riemann: nếuf hàm khả tích Riemann mặt phẳng liệu có tồn cặp tọa độ vng góc cho với y hàm sốx7→f(x, y)khả tích Riemann (theo biếnx) hàm số y7→R

f(x, y)dxcũng khả tích Riemann (theo biến y)? Besicovitch phát điều khơng ơng xây dựng tập hợp compact có độ đo chứa đoạn thẳng độ dài đơn vị theo hướng [1] Có chi tiết thú vị năm 1958, Hội Tốn Học Mỹ có làm series phim tập Toán nhiều bậc giáo dục Ở tập cuối Giáo sư A.S Besicovitch mời giảng lời giải ơng cho tốn Kakeya [1] Có lẽ tốn ví dụ cho tính đơn giản đẹp đẽ Toán học Trong viết này, tìm hiểu lời giải cho tốn Kakeya (có lẽ lời giải đủ đơn giản để học sinh phổ thơng theo dõi được) cách xây dựng tập Besicovitch (bạn đọc cần kiến thức giải tích hàm lý thuyết độ đo) Cuối liên hệ tới Giả thuyết Kakeya Các bạn đọc muốn tìm hiểu thêm xin xem [11, 13]

II- Lời giải cho toán Kakeya

Thật ra, kết Besicovitch có từ năm 1919, tình hình bất ổn nước Nga lúc nên chúng biết đến rộng rãi công bố sau tờ Mathematische Zeitschrift (1928) Năm 1929, ý tưởng Besicovitch Perron đơn giản hóa Chứng minh mà tìm hiểu sau dựa trình bày [1, 4] chúng tơi nghĩ hồn tồn thích hợp với học sinh phổ thông

Trước hết, để ý thay yêu cầu quay kim (một cách liên tục) đủ góc3600, ta cần xây dựng tập hợp cho phép quay kim đủ góc900, sau dùng phép đối xứng ta dễ dàng thu lời giải cho toán ban đầu

Trở lại với câu hỏi: Làm để nhốt voi vào tủ lạnh? Câu trả lời phổ biến mà chúng tơi tìm Internet là: chặt voi nhiều khúc, đặt khúc vào bên đóng cửa tủ lạnh Tất nhiêncâu trả lời có nghĩa hài hước, ý tưởng hồn toàn tương tự cộng với điểm khác biệt: thành phần khác voi khơng thể chiếm vị trí khơng gian, hình chồng lên mặt phẳng!

Cụ thể hơn, xét tam giácABC (không thiết cân tạiA) Tưởng tượng ta cắt tam giácABC dọc theo trung tuyếnAM tam giác A1BM1,A2CM2 (Hình 3) Tịnh tiến

A1BM1 bởiδ

−−→

BC (δ∈(0,1)) chồng lênA2CM2 ta hình có diện tích nhỏ diện tích tam giác ban đầu Điều quan trọng đoạn thẳngAN vớiN thuộc đoạn

(45)

Bổ đề Hình (Bước 2) tạo phận: tam giác XBC đồng dạng với tam giác ban đầu với tỉ lệ(1−δ); hai "tai" (hai tam giácA2XY vàA1XZ) có tổng diện tích 2δ2 diện tích tam giác ban đầu

Tuy nhiên bạn người "khó tính" (theo nghĩa Tốn học) cịn chút khơng thoải mái Đó cách tịnh tiến ta phá vỡ liên tục: Hình (Bước 2)

XB <1 ta khơng thể quay đoạn thẳng độ dài đơn vị đoạn A1B thành có phương A2C cách liên tục mà khơng ngồi hình vẽ

Tuy nhiên khó khăn xử lý sau Để ý rằngA1M1//A2M2, ta lấy điểm U thuộc đoạn M1M2 (nhưng khácM1) thìA1U vàA2M2 cắt điểm R (xem Hình 5) Lúc liên tục khôi phục sau: ta xuất phát từ đoạn thẳng đơn vịA1W với

W thuộcA1B; cố địnhA1 quayA1W tới tiaA1R(q trình khơng ngồi diện tích tam giác ABC đường cao từ A tam giác này≥1); trượt đoạn thẳng theo tiaA1R đầu mút làR; cố định R quay đoạn thẳng tới tiaRA2; trượt đoạn thẳng theo tia

RA2cho đến đầu mút làA2; cuối cố địnhA2 quay đoạn thẳng tới tiaA2C Trong q trình diện tích sử dụng thêm tam giác cânRP Qvới cạnhRP =RQ= 1; nhiên diện tích nhỏ diện tích tam giácA1M1U làm nhỏ tùy ý cách chọnU đủ gần

M1

(46)

Để diễn đạt ý tưởng cách chặt chẽ, ta ký hiệuT tam giácABC có đáyBC nằm đường thẳngdvà đường cao từAbằng Với số nguyên dươngN ta chia đoạn thẳngBC

thành2N đoạn từ thu được 2N tam giácT

i chung đỉnh Atrong đóTi−1 vàTi kề Đầu tiên ta tịnh tiến cặp tam giác(T1, T2),(T3, T4) trường hợp Bổ đề với tỉ lệδ∈(0,1)và thu được2N−1hình có dạng Hình (Bước 2) hình gồm tam giác mớiEi = (1−δ)(T2i−1∪T2i)và hai "tai" với diện tích2δ2|T2i−1∪T2i|(chú ý rằngT2i−1∪T2i tam giác, nhắc lại ta ký hiệu |S| cho diện tích hình S) Để ý tam giác Ei sau số phép tịnh tiến thích hợp ghép lại thành tam giác với (1−δ)T Tiếp theo ta xem tam giác Ei tam giácTi bước lại tịnh tiến chúng theo cặp (E2i−1, E2i)với tỉ lệδ >0để được2N−2hình (khi tịnh tiến tam giácEi hai "tai" tịnh tiến theo) Bằng cách sau N bước ta hình cuối cùng, gọi Perron

Định lý 1(Perron tree) Diện tích Perron khơng vượt q (1−δ)2N + 2δ|T| Nói riêng, chọnδvàN thích hợp ta làm cho diện tích bé tùy ý

Chứng minh Sau bước ta được2N−1tam giác mớiEivà2N "tai" với tổng diện tích khơng q 2δ2|T| Tương tự, sau bước thứ hai ta có 2N−2 tam giác "tai" với tổng diện tích khơng vượt q 2δ2(1−δ)2|T| Tới bước thứ N ta tam giác bằng(1−δ)NT với "tai" với diện tích khơng q2δ2(1−δ)2(N−1)|T| Vậy tổng diện tích Perron khơng vượt

(1−δ)N(T)+ 2δ2

1 + (1−δ)2+ (1−δ)4+ + (1−δ)2(N−1)|T|

= (1−δ)2N|T|+ 2δ21−(1−δ)

2N

1−(1−δ)2 |T| ≤ (1−δ)

2N+ 2δ

|T|

Rõ ràng chọn δ đủ nhỏ sau chọn N đủ lớn ta làm cho(1−δ)2N + 2δ bé tùy ý (Ta chọnδ= log(N)/(2N)và sử dụng bất đẳng thức1−δ≤e−δ để suy ra(1−δ)2N+ 2δ≤

2 log(N)/N.)

Áp dụng định lý cho tam giác vuông cânT =ABC vớiA= (0,0), B= (1,0), C = (0,1) ta xây dựng tập có độ đo nhỏ tùy ý (bao gồm Perron phần thêm vào Hình 5) mà cho phép quay đoạn thẳng đơn vị cách liên tục đủ góc 900 Sau sử dụng copy (qua phép đối xứng) hình này, ta ghép lại hình có diện tích nhỏ tùy ý mà cho phép quay đoạn thẳng đơn vị cách liên tục đủ3600 Đó câu trả lời cho tốn Kakeya

III - Tập Besicovitch

Trong mục trước ta thấy với mỗiε >0 ta xây dựng hình có diện tích khơng qεmà quay đoạn thẳng đơn vị cách liên tục Một câu hỏi tự nhiên liệu ta lấyε= 0? Đáng tiếc, khơng q khó khăn ta thấy câu trả lời phủ định

Mệnh đề 1(Terence Tao [12]) Không tồn tập có độ đo mà đoạn thẳng đơn vị quay cách liên tục trọn vòng3600.

Để cho chứng minh chặt chẽ kết này, cần phát biểu tốn dạng "giải tích" Một đoạn thẳng đơn vị tham số hóa `={(u+sv|s∈[0,1]} vớiu∈R2

vàv∈S1, đóS1={(x, y)∈R2|x2+y2= 1}là đường trịn đơn vị Như kết phát

biểu

`(t) :={u(t) +sv(t)|s∈[0,1]}

vớiu: [0, T]→R2 liên tục, v: [0, T]→S1 liên tục toàn ánh, tập hợp{`(t)|t∈[0, T]} khơng

(47)

Chứng minh Vì uvàv liên tục khoảng đóng[0, T] nên chúng liên tục Do với δ = 18 tồn tạiε >0sao cho|u(t1)−u(t2)|< δ,|v(t1)−v(t2)|< δ nếu|t1−t2| ≤ε

Bởi vìv: [0, T]→S1khơng số, tồn tạit

1, t2sao cho0<|t1−t2| ≤εvàv(t1)6=v(t2) Để đơn giản, phép tịnh tiến quay cần, ta giả sửu(t1) = (0,0)vàv(t1) = (1,0) Với

t∈[t1, t2]ta có|u(t)|=|u(t)−u(t1)|< δ= 1/8, đóu(t)nằm bên trái đường thẳngx=14 Tương tựu(t) +v(t)nằm bên phải đường thẳngx= 34 Vậy đoạn thẳng `(t) :={u(t) +sv(t)|s∈[0,1]}

cắt đường thẳng x=a vớia∈[14,43] điểm, ký hiệu (a, wa(t)) Bởi vìwa(t) liên tục vàwa(t1) = nên[0, ua(t2)]⊂wa([t1, t2])

Nói riêng, đoạn thẳng`(t1), `(t2)cắt đường thẳng x= 41,x= 34 P1, Q1 vàP2,

Q2 (xem Hình 6) Theo phân tích trên, với điểm(a, b)thuộc đoạn P2Q2 đoạn thẳng nối (a, b)và (a,0) hồn toàn nằm tập hợp {`(t)|t ∈ [t1, t2]} Vậy {`(t)|t ∈ [t1, t2]} chứa tứ giác

P1P2Q2Q1(đây tứ giác lồi nếuP Q khơng cắt trục hồnh−Trường hợp 1, tứ giác lõm

P Q cắt trục hồnh− Trường hợp 2) Hiển nhiên diện tích tứ giác P1P2Q2Q1 lớn hơn0, ta có điều phải chứng minh

Như với yêu cầu "cây kim" phải quay liên tục diện tích mà chiếm chỗ phải có độ đo dương (mặc dù nhỏ tùy ý) Tuy nhiên số vấn đề kiện "có độ đo0" trở nên quan trọng, u cầu "quay liên tục" khơng cịn cần thiết Chẳng hạn, tập hợp có độ đo xem "khơng đáng kể" lý thuyết tích phân: việc thay đổi giá trị hàm số tập có độ đo 0, phá vỡ cách nghiêm trọng liên tục, lại khơng ảnh hưởng tới tích phân hàm số Thật ra, nói mục đầu tiên, ban đầu Besicovitch quan tâm đến câu hỏi rằng: nếuf hàm khả tích Riemann mặt phẳng liệu có tồn cặp tọa độ vng góc cho với y hàm sốx7→f(x, y)khả tích Riemann (theo biến

x) hàm sốy 7→R

f(x, y)dx khả tích Riemann (theo biếny)? Besicovitch thấy để phủ định điều này, ơng cần tìm tập compact có độ đo0 chứa đoạn thẳng độ dài đơn vị theo hướng

Định lý 2(Besicovitch) Tồn tập compact mặt phẳng, có độ đo 0, chứa đoạn thẳng độ dài đơn vị với phương tùy ý

(48)

Chứng minh Ý tưởng ta dùng Perron làm "khung" để xây dựng dãy tập mở bị chặn{Vk}∞k=1 thỏa mãn:

(i)Vk+1 ⊂Vk với mọik= 1,2, vàlimk→∞|Vk|= 0(|Vk|là độ đo tậpVk) (ii)Vk chứa đoạn thẳng đơn vị tạo với trục hoành góc trong[0, π/2] Khi đóE=

∞ T k=1

Vk tập cần tìm Thật vậy, mỗiVk tập compact nên Ecompact Hơn điều kiện (i) đảm bảoEcó độ đo0 Bây xét`:={u+sv, s∈[0,1]}(u∈R2,v∈S1)

vector đơn vị tạo với trục hoành góc trong[0, π/2], ta chứng tỏ tồn phép tịnh tiến

t∈R2 cho`+t:={(u+t) +s(v+t), s∈[0,1]} ⊂E Thật vậy, điều kiện (ii) với mỗik∈N

tồn tạitk∈R2saocho `+tk ⊂Vk Bởi vìVk ⊂V1 bị chặn nên dãy{tk}∞k=1 bị chặn Do tồn dãy con{tkr}

r=1hội tụ vector ttrongR2 Với mỗim∈Nthì`+tkr ⊂Vkr ⊂Vmvớir

đủ lớn, chor→ ∞ta `+t⊂Vm Vì điều với mọimnên `+t⊂E

Trong phần lại chứng minh ta sử dụng Perron để xây dựng dãy tập hợp

{Vk}như Ta thống Perron đề cập tới bên hiểu tập đóng

Bước (k= 1).LấyT0 tam giác (đóng)ABC với đáyBC nằm đường thẳng dvà đường cao từA 1, lấyV0 tập mở bị chặn chứaT0 Ta xây dựng Perron

T1từ T0 Định lý cho|T1| ≤2−2, ta muốn Perron nằm trongV0 Trước hết, ta chọn ε > đủ nhỏ cho lân cận mở B(T0, ε) := {a+b|a ∈ T0,|b| < ε} thỏa mãnB(T0, ε)⊂V0(bạn đọc giải thích ta chọnεnhư vậy?) Lấyδ >0đủ nhỏ

N ∈N đủ lớn, ta xây dựng PerronT1 theo Định lý (là hợp của2N tam giác có đáy đường thẳngd chiều cao bằng1, tam giác chồng lên nhau) cho

|T1| ≤2−2

Ta chứng minh nếuN đủ lớn cho2−N < εthìT1⊂B(T0, ε) Thật vậy, trở lại với cách xây dựng Perron Định lý 1, ta thấy bước ta tịnh tiến tam giác đoạn không quáδ2−N (2−N cạnh đáy tam giác con), bước thứ ta tịnh tiến tam giác đoạn không quáδ(1−δ)2−N ((1−δ)2−N cạnh đáy tam giác mới) tổng cộng trình ta tịnh tiến tam giác đoạn không

δ2−N(1 + (1−δ) + + (1−δ)N−1) =δ2−N1−(1−δ) N

1−(1−δ) ≤2

−N < ε.

Vậy điểm trongT0 sau trình tịnh tiến đoạn nhỏ ε, khơng thể ngồi lân cậnB(T0, ε) VậyT1⊂B(T0, ε)

Cuối cùng, ta chọnV1là tập mở chứaT1 nằm trongB(T0, ε)sao cho|V1| ≤ |T1|+ 2−2≤2−1 (bạn đọc giải thích ta chọnV1như vậy? Gợi ý: sử dụng tính chất outer regularity độ đo)

Tóm lại kết thúc bước ta Perron T1 (đóng), tập mở V1 với |V1| ≤2−1,

T1⊂V1⊂V1⊂V0

Bước Giả sử ta bước thứ k ≥1, ta có Perron Tk (đóng), tập mởVk với|Vk| ≤2−k, vàTk ⊂Vk ⊂Vk ⊂Vk−1

Nhắc lại Tk hợp 2m (m∈N đó) tam giác conTki, tam giác

(49)

|Tfki| ≤ε:= 2−(m+k+2)vàTfki⊂Vk LấyTk+1:=S i

f

Tki, ta cóTk+1⊂Vkvà|Tk+1| ≤ε2m= 2−(k+2) Từ ta chọn Vk+1 tập mở cho Tk+1 ⊂ Vk+1 ⊂ Vk+1 ⊂ Vk |Vk+1| ≤

|Tk+1|+ 2−(k+2)≤2−(k+1)

Bằng quy nạp ta xây dựng dãy{Vk}∞k=1 mong muốn Điều kết thúc chứng minh

IV - Một cách xây dựng khác

Trong mục giới thiệu cách xây dựng khác cho tập Besicovitch Cách xây dựng độc đáo đưa gần (2003) Tom Korner (Cambridge, U.K) [9], điểm mấu chốt sử dụng Định lý Baire cho không gian Banach, qua có "rất nhiều" tập Besicovitch Ở dựa theo chứng minh trình bày lại Terence Tao [13]

Một khái niệm quan trọng xây dựng làessential range(miền ảnh chính)

Định nghĩa 1(essential range) Cho hàm sốf : [0,1]→Rđo được, ta định nghĩaR(f),

essential range củaf, tập hợp điểm y∈Rsao cho tập hợpf−1((y−, y+))có độ đo dương

với >0

Để ý f đo nênf−1((y−, y+))là đo Hơn ta thay đổif tập có độ đo0 độ đo củaf−1((y−, y+))khơng đổi Do đóR(f)được định nghĩa tốt phụ thuộc vào lớp tương đương củaf (gồm hàm sai khác tập có độ đo 0) Các tính chất khác essential range nêu Bổ đề

Bổ đề Chof : [0,1]→Rđo Khi

(i) f(x)∈R(f)với hầu hếtx∈[0,1]

(ii) R(f)là tập đóng

(iii) Nếu g∈L∞([0,1])

R(f+g)⊂R(f) + [−||g||∞,||g||∞] Nói riêng, f ∈L∞([0,1]) thìR(f)⊂[−||f||∞,||f||∞]bị chặn

Để chứng minh (i) cần dùng Bổ đề phủ Vitali Xin nhắc lại phiên đơn giản bổ đề hữu ích bỏ qua chứng minh (xin google bạn biết kết lần đầu)

Bổ đề 3(Vitali Covering Lemma) Cho{Bi}i∈U họ cầu mở không gianRN

Thì tồn tập đếm đượcJ ⊂U cho{Bi}i∈J cầu mở rời nhau, [

i∈U

Bi⊂ [

i∈J 5Bi

trong đó5Bi cầu mở tâm với Bi có bán kính gấp lần

Phần sau gồm có chứng minh cho (i) (iii), khẳng định (ii) xin dành lại tập đơn giản cho bạn đọc

(50)

Xét họ khoảng mở{Ax:= (f(x)−x/5, f(x) +x/5)}x∈B Theo Bổ đề phủ Vitali tồn tập đếm đượcJ ⊂B cho{Ax}x∈J khoảng mở rời nhau,

[

x∈B

Ax⊂ [

x∈J 5Ax

trong đó5Ax= (f(x)−x, f(x) +x)

Sử dụng tính chất trên, vớix∈f−1(A

x)(∀x∈B), ta có

B⊂ [

x∈B

f−1(Ax)⊂ [

x∈J

f−1(5Ax)

Nhắc lại f−1(5Ax) có độ đo VậyB chứa hợp họ đếm tập có độ đo 0, đó|B|=

(iii) Bằng cách thay đổi f vàg tập có độ đo cần (điều khơng làm thay đổi

R(f)vàR(f+g)), ta giả sử f(x)∈R(f)vàg(x)∈[−||g||∞,||g||∞]với mọix∈[0,1] Bây xéty∈R(f+g) Do định nghĩa, với mỗin∈Nthì tập hợp(f+g)−1((y−1/n, y+ 1/n))

có độ đo dương (nói riêng khác rỗng), đo ta chọn phần tử xn ∈ [0,1] Vậy

f(xn) +g(xn)→y

Do dãy g(xn)chứa tập compact [−||g||∞,||g||∞]nên chuyển qua dãy cần ta giả sửg(xn)→z∈[−||g||∞,||g||∞] Khi đóf(xn) =y−g(xn)→y−z Vì dãy f(xn)chứa tập đóngR(f)nêny−z∈R(f) Vậy

y= (y−z) +z∈R(f) + [−||g||∞,||g||∞] Điều với y∈R(f+g)nên suy

R(f+g)⊂R(f) + [−||g||∞,||g||∞]

Nói riêng chọn f = 0ta thu R(g)⊂[−||g||∞,||g||∞] với mọig∈L∞([0,1])

Ta quan tâm đến độ đo essential rangeR(f) Tổng quát hơn, ta có khái niệmFavard length

Định nghĩa (Favard length) Cho f ∈L∞([0,1]) vàα∈R Ta đặt Lα(f) =|R(f+αId)|

trong đóIdlà ánh xạ đồng Id(x) =x

Lưu ý f +αId∈L∞([0,1]) nên R(f +αId) tập compact, đo đóLα(f) số thực không âm

Kết sau liên kết khái niệm essential range Favard length với tập Besicovitch mà cần xây dựng Tất nhiên, tương tự mục trước, phép đối xứng ta cần chứng tỏ tồn tập compact mặt phẳng, có độ đo chứa đoạn thẳng đơn vị tạo với trục hồnh góc tùy ý trong[0, π/4] Một tập hợp gọi tậpquarter-Besicovitch

Định lý Giả sử f ∈L∞([0,1]) thỏa mãnLα(f) = với mọiα∈[0,1] Khi tập hợp

E:={(x, y)∈R×[0,1] :x∈R(f+yId)với mọiy∈[0,1]}

(51)

Chứng minh Dễ thấy E bị chặn Để chứng minh E đóng, giả sử (xn, yn)∈E và(xn, yn)→ (x, y)trongR2, ta chứng tỏ(x, y)∈E, tức làx∈R(f+yId) Sử dụng Bổ đề (iii) ta có

xn∈R(f +ynId)⊂R(f+yId) + [−|yn−y|,|yn−y|]

DoR(f+yId)là tập compact nênx∈R(f+yId) VậyE đóng Hơn để ý phần giao E với đường thẳng y =a ln có độ đo (doLα(f) = với mọiα∈[0,1]) Từ dùng định lý Fubini suy ra|E|=

Cuối ta chứng tỏE chứa đoạn thẳng đơn vị với hệ số góc tùy ý trong[0,1] Thật vậy, ta với mỗix∈[0,1], tồn tạiλ(x)∈Rsao cho

(λ(x) +xy, y)∈E với mọiy∈[0,1] (1) Khi {(λ(x) +xy, y)|y ∈[0,1]} ⊂E đoạn thẳng có hệ số góc x∈[0,1]tùy ý, điều phải chứng minh

Để chứng minh (1), trước hết Bổ đề (i) nên với y ∈ [0,1], f(x) +xy ∈R(f+yId), tức là(f(x) +xy, y)∈E, với hầu hết x∈[0,1] Xét riêng với cácy hữu tỉ suy tồn tập

A∈[0,1]vớiAc:= [0,1]\Acó độ đo cho

(f(x) +xy, y)∈Evới x∈A, y∈[0,1]∩Q

Bởi E đóng nên chuyển qua giới hạn ta có

(f(x) +xy, y)∈E với mọix∈A, y∈[0,1]

Vậy x∈A ta cần chọnλ(x) =f(x) Nếu x∈Ac thì doA trù mật (bởi vì|Ac|= 0) nên có dãyxn trongAhội tụ vềx Ta có

(f(xn) +xny, y)∈E với mọiy∈[0,1]

Vì E bị chặn nênf(xn)bị chặn Do chuyển qua dãy cần ta giả sửf(xn) hội tụ số thực, ký hiệu làλ(x) Với mỗiy∈[0,1]cố định, do(f(xn) +xny, y)∈E đóng (f(xn) +xny, y)→(λ(x) +xy, xy)nên suy ra(λ(x) +xy, xy)∈E Vậy ta thu (1) điều hồn tất chứng minh

Trong phần cịn lại ta chứng tỏ tồn tạif ∈L∞([0,1])thỏa mãnLα(f) = 0với mọiα∈[0,1] Ta cần hai kết chuẩn bị

Bổ đề Với mỗiα∈Rthì hàm số Lα:L∞([0,1])→[0,∞)là nửa liên tục (upper semicon-tinuous)

Nhắc lại tính nửa liên tục có nghĩa nếufn→f trongL∞([0,1])thìlim supLα(fn)≤

Lα(f) Một cách tương đương, với >0 tồn tạiδ=δ(f, )sao choLα(f+g)≤Lα(f)nếu

||g||∞≤δ

Chứng minh DoLα(f) =L0(f+αId)nên cách thayf +αIdthay chof, ta cần chứng minh choL0 DoR(f)compact nên

R(f) = ∞ \

n=1

Rn vớiRn =R(f) + [−

n,

1

n]

Suy ra|Rn| → |R(f)| Do với >0thì |Rn| ≤ |R(f)|+vớinđủ lớn

(52)

Bây xem thử có Lα(f) = 0? Ta bắt đầu ví dụ đơn giản Ký hiệuP C tập hàm khúc (piecewise constant), tức làf ∈P C tồn phân hoạch hữu hạn0 =t0< t1< < tN = 1sao chof số khoảng(ti−1, ti)) Dễ dàng thấy nếuf ∈P C thìf nhận có hữu hạn giá trị, đóL0(f) = Tuy nhiên điều khơng đảm bảoLα(f) = vớiα6= Dù sao, kết sau ta kiểm sốtLα(f) = chọn |α| nhỏ Chứng minh kết đơn dùng định nghĩa dành lại cho bạn đọc tập

Bổ đề Nếuf thuộc P C thìLα(f)≤ |α| với mọiα∈R

Bây sẵn sàng để chứng minh định lý chính: tồn f ∈ L∞([0,1]) cho

Lα(f) = 0với mọiα∈[0,1] Thực ra, ta thu hẹp xuống không gian đóngP C ⊂L∞([0,1]) Khơng gian chứa hầu hết hàm số "tốt", chẳng hạnId, thực nhỏ hơnL∞([0,1]), chẳng hạn khơng chứasin(1

x) (bạn đọc kiểm tra khẳng định này) Ta có

Định lý Tập hợp F ={f ∈P C|Lα(f) = với mọiα∈[0,1]}trù mật P C

Nói riêng F 6=∅ mong muốn Để chứng minh Định lý ta sử dụng Định lý Baire cho không gian đầy đủP C (xin google bạn gặp định lý lần đầu)

Baire Category Theorem Cho X không gian Banach {Fn}∞n=1 tập mở, trù mật trongX Khi T∞n=1Fn trù mật X

Chứng minh Định lý Ta có

F = ∞ \

n=1

{f ∈P C|Lα(f)≤

n với mọiα∈[0,1]}

= ∞ \

n=1 n \

m=1

{f ∈P C|Lα(f)≤

n với mọiα∈[ m−1

n , m

n]}

Do Định lý Baire, ta cần chứng tỏ tập hợp có dạng

By,ε:={f ∈P C|Lα(f)≤2εvới mọiα∈[y, y+ε]}

mở trù mật trongP C Thật vậy, vìBy,εchứaP C+yId(do Bổ đề 5) suy tính trù mật Phần cịn lại xin bạn đọc tự kiểm chứng phần bù

P C\By,ε={f ∈P C|∃α∈[y, y+ε] :Lα(f)>2ε} đóng (sử dụng tính compact của[y, y+ε]và tính nửa liên tục Bổ đề 2)

V - Giả thuyết Kakeya

Một tập Besicovitch trongRN là tập hợp có độ đo (trong

RN) chứa đoạn thẳng đơn

vị với phương tùy ý Từ tập Besicovitch mặt phẳng, ta xây dựng tập Besicovitch với số chiều lớn (bạn đọc giải thích sao?)

(53)

Trước hết làm quen với số chiều Hausdorff Nói nơm na, ta xem độ đo "ước lượng xác" độ lớn số chiều một"ước lượng thô" Hãy tưởng tượng kiến đời bò đường thẳngR Đối với kiến Rlà khơng gian vơ rộng lớn, tới biên giới Nhưng ngày nọ, kiến leo lên mặt phẳng chiều Đối với lúc khơng gianRlại trở nên nhỏ bé Nhưng băn khoăn khơng biết giải thích cảm giác rằngR"nhỏ", dù saoRvẫn tập vơ hạn

Với kiến thức độ đo Lebesgue, bạn trả lời kiến rằng: đơn giản thơi, khơng gian chiều R có độ đo Tuy nhiên, đừng quên kiến chưa học lý thuyết độ

đo, nên cần giải thích thêm cho nó: ta phủ Rbởi họ đếm hình trịn R2

mà tổng diện tích làm cho bé tùy ý Chẳng hạn ta phủ[0,∞)bởi dãy hình trịn liền (Hình 7) với bán kính m1,m+11 ,m+11 , mcó thể chọn lớn tùy ý (chú ý P∞

n=1

n =∞nhưng P∞

n=1 n2 <∞)

Tuy nhiên thực mặt phẳng chiều, Rcịn bé Thật vậy, khẳng định phía

trên phát biểu cụ thể là: ta phủRbởi họ đếm hình trịnB(xn, rn) trongR2màP∞

n=1r

ncó thể làm cho bé tùy ý Thật ra, sử dụng ví dụ phía ta làm mạnh phát biểu thành: nếud >1 làm choP∞n=1rd

n bé tùy ý Mặt khác, hiển nhiên chọnd= Như hợp lý ta tuyên bố: "số chiều" củaRlà1 Chú ý ta thay

R2bởiRn "số chiều" củaRvẫn là1 Như khái niệm thuộc chất

củaRhơn không gian mà nhúng vào

Tổng quát quan sát trên, ta định nghĩa số chiều Hausdorff tập U không gian metricX sau Ta nóiU khơng qdchiều nếuU phủ họ đếm cầu{B(xn, rn)}trongX choP∞n=1rnd chọn bé tùy ý Chặn sốdnày gọi số chiều Hausdorff U, ký hiệu dimH(U)(một số tác giả gọi Hausdorff−Besicovitch dimension) Một cách hình thức, với d ≥ ta đặt dung lượng Hausdorff (Hausdorff content) củaU

CHd(U) := inf (∞

X

n=1

rdn|U ⊂

∞ [

n=1

B(xn, rn) )

và định nghĩadimH(U) := inf{d≥0|Cd

H(U) = 0}

Số chiều Hausdorff tổng quát số chiều thông thường (dimH(RN) = N)

xem định nghĩa cho số chiều Tuy nhiên tính số chiều Hausdorff đối tượng cho trước nói chung khơng dễ Do có số định nghĩa khác số chiều sử dụng rộng rãi Một chúng số chiều Minkowski (Minkowski dimensions, đơi cịn gọi Minkowski−Bouligand dimension hay box−counting dimension)

ChoU tập bị chặn trongRn Với mọiδ >0 ta gọiN

δ(U)là số lượng nhỏ cầu (nchiều) bán kínhδchúng ta cần dùng để phủU (chú ý doNđược tốt, tức tập khác rỗng có phần tử bé nhất, nênNδ(U)là số nguyên dương hoàn toàn xác định) Ta hi vọng nếuU códchiều vớiδ >0nhỏ

(54)

(Để dễ hình dung, thay cầu hình lập phương (nchiều)! Thật sử dụng hình lập phương khơng ảnh hưởng đến định nghĩa ta dùng cầu chất khơng gian metric−trong hình lập phương điểm đặc biệt riêngRn.)

Từ cơng thức phía trên, lấylog hai vế ta

log(Nδ(U))≈dlog(1/δ) +const(U)

Do vớiδ→0ta hi vọng

d≈log(Nδ(U))

log(1/δ)

Từ ta định nghĩa lower upper box-counting dimensions dimlowerbox(U) = lim inf

δ→0

log(Nδ(U)) log(1/δ) , dimupperbox(U) = lim sup

δ→0

log(Nδ(U)) log(1/δ)

Nếu hai giới hạn ta gọi box−counting dimension U ký hiệu dimbox(U) Đây định nghĩa số chiều Minkowski (vậy nói chung ta có số chiều lower upper)

Điểm tiện lợi số chiều Minkowski dễ dàng để tính cách xấp xỉ (sau ta chọnδ >0 nhỏ thìNδ(U)hồn tồn xác định) Số chiều Minkowski với số chiều Hausdorff lớp tập hợp "đẹp" (chẳng hạn số chiều tập Cantor bằnglog(2)/log(3)), nhiên nói chung chúng không (chẳng hạndimH(Q) = nhưngboxH(Q) = 1, bạn

đọc giải thích sao?) Một cách tổng quát, ta có (xem [5], trang 46)

dimH ≤dimlowerbox≤dimupperbox (2) Trở lại với tập Besicovitch, giả thuyết Kakeya [11] phát biểu

Kakeya set conjecture Mọi tập Besicovitch Rn có số chiều Hausdorff số chiều

Minkowski bằngn

Điều có nghĩa có độ đo 0, tập Besicovitch khơng "thực nhỏ" (về số chiều không gian) Chú ý bất đẳng thức (2), chứng minh giả thuyết Kakeya cho số chiều Hausdorff hiển nhiên cho số chiều Minkowski Tuy nhiên phát biểu phía nhấn mạnh với số chiều Minkowski giả thuyết chưa chứng minh, với số chiều Hausdorff giả thuyết chưa bị bác bỏ

Trường hợpn= 1thì giả thuyết cách tầm thường Năm 1971 Roy Davies chứng minh khẳng định với n = 2(xem [3] [5], trang 178) Với trường hợp n≥ người cố gắng nâng dần chặn số chiều Hausdorff (nếu chứng minh chặn lànthì giả thuyết giải xong) Năm 1995 Thomas Wolff chứng minh chặn n/2 + 1, nói riêng tập Besicovitch trongR3có nhất5/2chiều tập Besicovitch trongR4có nhất3

chiều (số chiều Hausdorff) Năm 2002, Nets Hawk Katz Terence Tao cải thiện đánh giá thành (2−√2)(n−4) + 3cho trường hợpn≥5 Riêng với số chiều Minkowski upper (dimupperbox) năm 2000 Katz−Laba−Tao [7] chứng minh chặn dưới5/2 + 10−10chon= 3, năm 2001 Laba−Tao [8] chứng minh điều tương tự, tức chặn dưới3 + 10−10, chon=

Tài liệu

(55)

[2] A.S Besicovitch, On Kakeya’s problem and a similar one,Mathematische Zeitschrift(1928) 27, 312-320

[3] R Davies, Some remarks on the Kakeya problem,Proc Cambridge Philos Soc (69) 417–421, 1971 doi:10.1017/S0305004100046867

[4] K J Falconer,The Geometry of Fractal Sets, Cambridge Tracts in Mathematics, 85 Cambridge University Press, Cambridge, (1986)

[5] K Falconer,Fractal geometry- Mathematical foundations and applications, Second Editor, John Wiley & Sons, Inc., Hoboken, NJ, (2003)

[6] N Katz, T Tao, New bounds for Kakeya problems Dedicated to the memory of Thomas H Wolff.J Anal Math.87(2002), 231-263

[7] N Katz, I Laba, T Tao, An improved bound for the Minkowski dimension of Besicovitch sets inR3,Annals of Math 152 (2000), 383-446

[8] I Laba, T Tao, An improved bound for the Minkowski dimension of Besicovitch sets in medium dimension,Geom Funct Anal 11 (2001), no 4, 773-806

[9] T W Korner, Besicovitch via Baire,Studia Mathematica158 (1) (2003), 65-78

[10] T Wolff, An improved bound for Kakeya type maximal functions, Rev Mat Iberoamericana (11) 651–674, (1995)

[11] http://en.wikipedia.org/wiki/Kakeya_set

[12] http://terrytao.wordpress.com/2008/12/31/a-remark-on-the-kakeya-needle-problem/

(56)

Bài viết Chuyên đề Dịch thuật Phương trình Bất phương trình hàm số

Phỏng dịch theo G.Falin, A.Falin, Tạp chí Kvant số 05, 06 năm 2006

Đinh Ngọc Vương , Sinh viên Đại học Vật lý Kỹ thuật Moskva

Trong năm gần đề thi vào trường Đại học Tổng hợp Quốc gia Matxcơva mang tên M.V Lomonosov thường xuất tốn giải phương trình, bất phương trình hệ phương trình mà giá trị chưa biết cần tìm khơng phải số mà hàm số Các toán khác toán thường dạng lẫn phương pháp giải Vì vậy, báo tác giả xin giới thiệu dạng phương pháp giải dựa vào ví dụ tốn đề thi Những tốn giải phương trình hay bất phương trình hàm có ích việc phân loại học sinh vào lớp chuyên toán

I - Phương trình tham số hóa

Phương trình hàm dạng đơn giản phương trình mà hàm chưa biết mơ tả bới hay nhiều tham số (ví dụ đơn giản hàm số đa thức) Trong trường hợp tốn tìm hàm số đưa dạng xác định giá trị tham số, nghĩa đưa tốn phổ thơng thơng thường Chúng ta khảo sát tốn sau:

Ví dụ (Khoa Tốn học tính tốn Điều khiển, 2001) Tồn khơng hàm tuyến tính

y=f(x)thỏa mãn phương trình sau với mọix:

2f(x+ 2) +f(4−x) = 2x+ (1)

Lời giải

Theo định nghĩa, hàm số tuyến tính hàm số có dạng: f(x) =kx+b Các tham sốkvàb biểu diễn hàm số nhất, nghĩa làk1x+b1=k2x+b2 với mọix, k1=k2 ,b1=b2

Đây trường hợp riêng khẳng định quan trọng sau mà áp dụng nhiều lần:

Hai đa thức đồng hệ số ứng với bậc biến số tương ứng

Vì tốn ban đầu viết dạng sau:

Tồn hay không số k, bsao cho với xthì đẳng thức sau đúng:

2(k(x+ 2) +b) + (k(4−x) +b) = 2x+ (2) Biến đổi tương đương ta được: kx+ 8k+ 3b= 2x+ 7, với mọix

Vậy sốk, b thỏa mãn hệ:

k=

8k+ 3b= (3)

(57)

Mặc dù tất phép biến đổi tương đương việc kiểm tra lại không cần thiết, giới thiệu cho bạn đọc cách kiểm tra trực tiếp có tìm hàm số thực thỏa mãn phương trình (1) hay khơng Nhận xét nói tốn

Giống phương trình bình thường, mà đại lượng chưa biết số, phương trình hàm nói chung khơng có nghiệm vô số nghiệm Chúng ta minh họa ví dụ tiếp theo:

Ví dụ (Khoa Tốn học tính tốn Điều khiển, 1997/2001/2005)Tồn hay khơng hàm số tuyến tính y=f(x)thỏa mãn điều kiện sau với mọix

f(x+ 3)−f(2−x) = 3x+ (4)

Lời giải

Giống phương pháp giải phương trình (1), tốn cho đưa dạng sau:

Tồn hay không số k, bđể hệ sau đúng:

2k=

k=

Hệ vơ nghiệm Do phương trình (4) khơng có nghiệm dạng tuyến tính

Ví dụ 3(Khoa Tốn học tính tốn Điều khiển, 1997)Tìm hàm bậc hai y =f(x)thỏa mãn điều kiện sau với mọix

f(1−x)−f(2−x) =−2x+ (5)

Lời giải

Theo định nghĩa, hàm bậc hai hàm số có dạng

f(x) =ax2+bx+c, a6=

Đưa toán dạng:

Tìm sốa6= 0, b, cthỏa mãn hệ:

2a=−2

−3a−b= (6)

Hệ (6) có nghiệm (a, b, c) = (−1,−4, c), c ∈ R - hệ số tự Vậy phương trình cho có vơ số nghiệm thuộc lớp hàm số bậc Tất nghiệm viết dạng: f(x) =−x2−4x+c, đóc- hệ số tự do.

Bài toán yêu cầu giải phương trình hàm thuộc lớp xác định (đa thức bậc n), phương trình phức tạp phương trình cho

Ví dụ 4(Khoa Tốn học tính tốn Điều khiển 2002/Olympic Romania 1980)Tìm tất đa thức bậcn:

P(x) =anxn+an−1xn−1+ +a1x+a0

nghĩa hệ sốan6= 0, thỏa mãn đồng thức

P(x2) = (P(x))2, x∈(−∞,+∞) (7)

(58)

Để giải toán viếtP(x)dưới dạngQ(x) +anxn, đó:

Q(x) =an−1xn−1+ +a1x+a0

Bậc củaQ(x)chúng ta chưa biết nên loại trừ khả số hệ số (hay chí tất cả) Khi (7) có dạng:

Q(x2) +anx2n= (Q(x))2+ 2anxnQ(x) +a2nx

2n, x∈(−∞; +∞) (8)

Hai đa thức đồng với hệ số hạng tử bậc phải Bậc đa thức Q(x2), (Q(x))2 bằng2k, bậc đa thức 2a

nxnQ(x) n+k Các số

hạng đơn thức bậc2nchỉ cóanx2nvàa2nx2n, vậy:an =a2n Nhưng vìan6= 0nênan = Điều

này cho phép viết (8) dạng:Q(x2) = (Q(x))2+ 2xnQ(x), x∈(−∞; +∞)

Vì k < n, nên bậc đa thức vế phải lớn bậc đa thức vế trái, điều xảy tất các hệ số đa thứcQ(x)bằng 0, tức làQ(x)≡0 Khi đồng thức (8) có dạng: anx2n=a2nx2n,x∈(−∞; +∞)tương đương vớian=

Vậy nghiệm phương trình hàm (7) thuộc lớp đa thức bậc nlàP(x) =xn.

II - Phương trình hàm tổng qt

Từ ví dụ xuất câu hỏi tự nhiên là: "Tồn hay không hàm số dạng tổng quát (không thiết phải hàm tuyến tinh) thỏa mãn phương trình hàm ban đầu?" Để trả lời câu hỏi giải (1) với hàm cho dạng tổng quát

Đầu tiên, thay xbởix−2, (1) có dạng:

2f(x) +f(6−x) = 2x+ 3,∀x (9) Coi (9) phương trình bình thường với biến A=f(x), B =f(6−x), xtrong trường hợp đóng vai trị tham số:2A+B= 2x+

Ta thấy, phương trình mà chứa tới hai ẩn chưa biết, ta cần phương trình Từ (9) thay x 6−x (vì đẳng thức với x nên thay x giá trị nào):

2f(6−x) +f(x) =−2x+ 15, với xhay2B+A=−2x+ 15

Ta có hệ phương trình sau:

2A+B= 2x+ 2B+A=−2x+ 15

Dễ dàng giải được: A≡f(x) = 2x−3

Nếu đặtφ0(x) =xvàφ1(x) = 6−x Dễ dàng kiểm tra

φ0(φ0(x)) =φ0(x); φ0(φ1(x)) =φ1(x); φ1(φ0(x)) =φ1(x); φ1(φ1(x)) =φ0(x)

Nói theo ngơn ngữ đại số đại, hàm sốφ0 vàφ1cùng với phép hợp hàm tạo thành

nhóm Khái niệm nhóm khái niệm quan trọng toán học đại ứng dụng rộng rãi nhiều lĩnh vực

Cũng lập luận tương tự, phương trình (4) vơ nghiệm xét hàm cần tìm thuộc lớp Chúng ta có kết mạnh sau:

Khơng tồn hàmf(x)nào thỏa mãn đồng thời(4)tại hai điểmx1 vàx2 đối xứng

(59)

Thật vậy, ta có hệ:

f(x1+ 3)−f(2−x1) = 3x1+

f(x2+ 3)−f(2−x2) = 3x2+

Vì x1 vàx2 đối xứng qua điểm−

1

2 nênx1+x2=−1 hệ chuyển dạng:

f(x1+ 3)−f(2−x1) = 3x1+

f(2−x1)−f(x1+ 3) =−3x1−2

Cộng theo vế hai phương trình thì:0 =−1

Giải phương trình (5) khó thú vị không cần điều kiện f(x)là tam thức bậc hai Giải toán cần sử dụng phương pháp mới, chúng thuận lợi cho việc giải phương trình khác, nói kỹ vấn đề Giống việc giải (1) hàm thuộc lớp tổng quát, trước tiên thay1−xbởixkhi (5) có dạng:

f(x)−f(x+ 1) = 2x+ 5, ∀x∈R (10)

Phương trình (10) khơng nhất, để phù hợp với tư tưởng chung giải phương trình hàm ta cần đưa dạng Để giải vấn đề tìm nghiệm riêng Thực tế làm điều này, nghiệm riêng làf0(x) =−x2−4x

Bây đặt g(x) =f(x)−f0(x)ta suy rag(x) =g(x+ 1),∀x∈R

Điều hàmg(x)là hàm số tuần hoàn với chu kỳT= Vì vậy, nghiệm chung (5) có dạng sau:f(x) =−x2−4x+g(x), đóg(x)- hàm tuần hồn với chu kỳ 1, xác định trên

toàn trục số

Tổng quát hóa ví dụ (bỏ qua điều kiện P(x) đa thức) nói chung khơng có câu trả lời cụ thể Phương trình hàm cho thỏa mãn với nhiều hàm khác nhau, ví dụ như:|x|,

3 √

x,y=sgn(x)- hàm số dấu (sgn(x)bằng 1, hay -1 phụ thuộc vàoxlớn hơn, hay nhỏ 0) Các lập luận đưa yêu cầu cao giải phương trình mà thiếu thơng tin liên quan đến dạng hàm cần tìm Mặc dù phương pháp giải không thay đổi xuất khả đặc biệt, xem xét điều thơng qua ví dụ sau:

Ví dụ 5(Khoa Hóa học, 7/2000).Hàmf(x) xác định đoạn 1

6,6

thỏa mãn hệ:      cos2f(x)−1

2

−12 cos 2f 1 x = 10 x

0≤f(x)≤π Giải bất phương trình: f(x)≤ π

8

Lời giải

Đơn giản hóa phương trình đầu:

1

cos(2f(x))−6 cos 2f 1 x =

x, x∈

1 6,6

(11)

Nếu x∈ 1

6,6

thì x ∈

1 6,6

Trong (11) ta thayxbởi x cos 2f 1 x

−6 cos (2f(x)) = 5x, x∈ 1

6,6

(60)

Coi hệ gồm hai phương trình (11) (12) hệ hai ẩn số:A= cos(2f(x))vàB = cos 2f 1 x , xtrong trường hợp đóng vai trị tham số

    

A −6B=

5

x

1

B −6A= 5x Bằng phép biến đổi sơ cấp, ta đưa hệ dạng:

(

B= x−5A 6Ax

(A+x)(6A−x) =

Với x∈ 1

6,6

thì A=A(x)nhận hai giá trị: −xhay x

6 Điều kiện 0≤f(x)≤π

4 bảo đảm choA= cos (2f(x))≥0 Vì nênA+x≥0và dễ dàng cóA=A(x) =

x

6,

B=B(x) = 6x

Hệ phương trình đầu tương đương với hệ sau:

  

 

cos (2f(x)) = x 6;∀x∈

1 6,6

0≤2f(x)≤ π

Khi x∈ 1

6,6

thì x∈

1 36,1

, ý

1 36,1

⊆[−1,1] Sử dụng định nghĩa hàm số cosin thu kết quả:f(x) =

2arccos

x

6 Yêu cầu toán ban đầu đưa việc giải bất

phương trình:arccosx ≤

π

4, vớix∈ 1

6,6

Dễ dàng giải được:3√2≤x≤6

Ví dụ (Khoa Hóa học, 2000).Tìm xđể hàm f(x) có cực trị thỏa mãn điều kiện sau với x6= 0,1

f(x) +f

1−x

=x (13)

Tìm hàm số

Lời giải

Giống cách giải ví dụ trước, coi phương trình hàm cho phương trình bình thường với ẩn chưa biết A = f(x) vàB =f

1 1−x

, x coi tham số: A+B =x Để nhận thêm phương trình nữa, (13) thayxbởi

1−x: f

1 1−x

+f

x−1

x

=

1−x ⇔B+C=

1 1−x

trong đóC=f

x−1

x

Vì xuất thêm ẩn nên ta cần thêm phương trình Trong (13) thay xbởi x−1

x :

f

x−1

x

+f(x) =x−1

x ⇔C+A= x−1

(61)

Như khơng làm xuất thêm phương trình mới, có hệ sau:

   

  

A+B =x B+C=

1−x C+A= x−1

x Dễ dàng giải hệ thu kết quả:

A≡f(x) = x

3−x+ 1

2x(x−1), x6= 0,1 (14)

Ba hàmA, B, C phù hợp với nhận xét giải phương trình (9): hàm tạo thành nhóm bậc ba Bây tìm điểm cực trị hàm (14) Đạo hàm:

f0(x) = x

4−2x3+x2−2x+ 1

2x2(x−1)2 =

x2− +√2x+ x2− 1−√2x+ 2x2(x−1)2

Biến đổi nhờ xét dấu đạo hàm ta thu kết hàm số f(x)có điểm cực trị:

x1=

1 +√2−p2√2−1

2 ∈(0; 1) (điểm cực đại)

x2=

1 +√2 +p2√2−1

2 ∈(1; +∞)(điểm cực tiểu)

III - Sử dụng khái niệm kết giải tích

Ví dụ 7(Khoa Tốn Cơ, 2001/Olympic lớp 10)Hàm số f(x) thỏa mãn điều kiện sau

x+f(x) =f(f(x)), ∀x∈R (15)

Giải phương trình f(f(x)) =

Lời giải

Hàm số cần tìm song ánh, thật vậy: nếuf(x1) =f(x2)thì:

x1=f(f(x1))−f(x1) =f(f(x2))−f(x2) =x2

Nếu x1, x2 nghiệm phương trìnhf(f(x)) = 0thìf(f(x1)) =f(f(x2)) Vì hàm song

ánh nên ta suy được:f(x1) =f(x2)⇒x1=x2 Như phương trình f(f(x)) = 0khơng thể

có nghiệm số

Thay x= 0vào phương trình (15) dễ dàng thấy nghiệm phù hợp Vậy hàm cho có nghiệm nhất:x=

Sau dựa theo ví dụ hay tìm hàm f(x)thỏa mãn (15) Nếu f(x)có dạng tổng qt việc giải khó thành cơng, nếuf(x)có dạng đa thức chứng minh (15) có nghiệm:

f1(x) =

1 +√5

2 x, f2(x) = 1−√5

2 x

Thật vậy, f(x)là đa thức có bậcn≥1 vế trái (15) có bậcn cịn vế phải (15) có bậcn2 Điều phù hợp khin= Đặtf(x) =kx+b Khi (15) có dạng:

(62)

Hay

(k+ 1)x+b=k2x+ (kb+b), ∀x Tương đương với hệ:

k+ =k2

b=kb+b Hệ có nghiệm:(k, b) = +

√ ,0

!

,(k, b) = 1− √

5 ,0

!

Chúng tương đương với hàm số thỏa mãn điều kiện (15):

f1(x) =

1 +√5

2 x, f2(x) = 1−√5

2 x (16)

Hai hàm chưa đủ để lập thành tập nghiệm (15) Chẳng hạn, hàm số

f(x) =   

 

1 +√5

2 x, khix∈A={a+b √

5|a, b∈Q} 1−√5

2 x, khix∈R\A

(17)

cũng nghiệm phương trình

Ví dụ (MK-MGU, 2005, vịng 1)Tồn hay khơng hai hàmf vàg xác định trênR thỏa mãn điều kiện sau:

f(g(x)) =x2, g(f(x)) =x3 (18)

Lời giải

Giả sử tồn hàm thỏa mãn điều kiện toán Giống cách giải ví dụ 7, trước hết ta chứng minh hàmf(x)là song ánh Giả sử với sốx1, x2 chof(x1) =f(x2), từ (18) ta có:

g(f(x1)) =g(f(x2)) Do đóx1=x2

Xét hàm số: f(g(f(x))) Vì g(f(x)) =x3 nênf(g(f(x))) =f x3

Mặt khác: f(g(f(x))) = (f(x))2 Ta có đẳng thức đúng:f x3

= (f(x))2với x∈R

Thay giá trịx= 0, 1,−1vào đẳng thức trên, đặta=f(0), b=f(1), c=f(−1)thu đẳng thức sau:a=a2, b=b2, c=c2

Vì hàmf song ánh nên sốa, b, c phải khác nhau, nghĩa phương trình bậc 2:t2=t có

3 nghiệm phân biệt Điều khơng thể xảy nên điều giả sử hàmf, g tồn khơng xác Vậy khơng tồn hàmf, g thỏa mãn điều kiện

Ví dụ (Khoa Tốn Cơ, 2003).Tìm tất hàmf(x)xác định toàn trục số thỏa mãn bất đẳng thức sau

f(y).cos (x−y)≤f(x) (19)

trong đóx, y số

Lời giải

Trong (19) thay ybởi x−π 2:

fx−π

.cosπ

(63)

Trong (19) thay tiếpy x+t:

f(x+t).cost≤f(x)

Vì cost≥1−t

2

2 hàmf không âm nên bất đẳng thức sau đúng:

f(x+t)

1−t

2

2

≤f(x) (20)

Xétt∈ −√2;√2, đó1−t

2

2 >0, chia vế (20) cho1−

t2

2

f(x+t)≤ f(x) 1−t

2

2

(21)

Trong (20) thay xbởix−t, sau thayt bởi−tthì thu đươc: f(x)

1−t

2

2

≤f(x+t) (22)

Từ (21) (22) ta có:

−t

2

2f(x)≤f(x+t)−f(x)≤f(x)

t2

2−t2, t∈

−√2;√2

Bất đẳng thức sau đúng:

|f(x+t)−f(x)| ≤f(x) t

2

2−t2, t∈

−√2;√2

Giả sửt6=

f(x+t)−f(x)

t

≤f(x) |t| 2−t2, t∈

−√2;√2, t6= (23) Bây cố địnhxvà chottiến dần đến0, hàm số vế trái tiến dần đến0 Theo nguyên lý kẹp có:

lim

x→0

f(x+t)−f(x)

t =

Theo định nghĩa giới hạn f0(x) Như chứng minh đạo hàm hàm cần tìm bằng0 với giá trị củax, tức làf(x)là hàm giá trị phải khơng âm f(x)≥0(đã chứng minh trên):

f(x)≡c, c≥0 (24)

Kiểm tra dễ dàng hàm số f(x) ≡ c, c ≥ thỏa mãn điều kiện (19) hiển nhiên

cos (x−y)≤1

IV - Phương trình hàm số cổ điển

Mỗi hàm số tốn học có tính chất xác định mô tả đẳng thức hay bất đẳng thức, chí khẳng định phức tạp Ví dụ hàm số f(x) =ax, a >0, a6= thỏa mãn số tính chất sau (với mọixvày):ax+y=axay, ax−y= a

x

ay, a

x>0, a0= 1

(64)

f(x) =axtăng a >1 giảm nếu0< a <1; f(x) =axliên tục với mọix;

f(x) =axkhả vi với mọix, đóf0(x) =axlna.

Nếu từ quan hệ chưa biết rõ hàm f(x)như tính chất viết lại sau:

f(x+y) =f(x)f(y), f(x−y) = f(x)

f(y), f(x)>0, f(0) = 1;

f(x)tăng nếua >1 giảm nếu0< a <1; f(x)liên tục với mọix;

f(x)khả vi với mọix, đóf0(x) =f(x) lna.

Một số phương trình (ví dụ: f(x+y) = f(x)f(y), f0(x) = f(x) lna) xem phương trình hàm số xuất câu hỏi tự nhiên tập hợp nghiệm chúng Điều thú vị phương trình cho khơng có nghiệm hàm khác ngồi hàm mà tính chất dẫn đến phương trình Trong trường hợp phương trình hàm cho kết hàm có tính chất đặc trưng Nếu phương trình có chứa phép tốn vi phân phương trình gọi phương trình vi phân Lý thuyết phương trình vi phân lĩnh vực lớn toán học đại với nhiều toán tự nhiên

1 Tính chất hàm tỉ lệ thuận

Hàm tỉ lệ thuận có dạng: y=kx Dễ thấy tất hàm có dạng nghiệm phương trình:

f(x+y) =f(x) +f(y), ∀x, y∈R (25)

Dưới chứng minh: Nếu f(x) liên tục phương trình thỏa mãn nghiệm có dạngf(x) =kx Nếu bỏ điều kiện liên tục nghiệm với xhữu tỷ Chúng trình bày lý thuyết Cauchy để giải số tốn sau:

Ví dụ 10 (Khoa Sinh học, 7/2005) Cho hàm sốf thỏa mãn:f(x+y) =f(x) +f(y) với

xhữu tỷ Biết rằngf(10) =−π Tínhf

−2

7

?

Lời giải

Chúng ta giải phương trình hàm cho:

f(x+y) =f(x) +f(y), ∀x∈Q (26)

Trước hết nhận thấy hàm có dạng f(x) =kxthỏa mãn phương trình Ta chứng minh phương trình khơng cịn nghiệm khác

Thayy= vào phương trình cho:f(x) =f(x) +f(0), suy raf(0) = Thayy=−xthì (26) có dạng:f(0) =f(x) +f(−x), suy raf(−x) =−f(x)

(65)

Thay y =xthì (26) có dạng: f(2x) = 2f(x), ∀x∈Q Sử dụng phương trình từ (26)

thayy= 2xta có:f(3x) = 3f(x),∀x∈Q

Tương tự y = 3xthì ta có: f(4x) = 4f(x),∀x∈Q Lặp lại trình nhận

được: Với số nguyên dươngnthì đẳng thức sau

f(nx) =nf(x),∀x∈Q (27)

Chứng minh điều dễ dàng nhờ phương pháp quy nạp Hàm ta cần tìm hàm lẻ (như nhận xét trên), (27) với số nguyênn

Ta chứng mình:

f(rx) =rf(x), ∀x∈Q (28)

Trong (27) thay x= t

n, thì:f(t) =nf

t n

Suy ra:f

t n

=

nf(t) Như (28) với r=

n, dễ dàng suy ra: f

m

nx

=

nf(mx) =

1

nmf(x) = m

nf(x) Vậy (28) với số hữu tỉr

Nếu thay x= 1thì: f(r) =rf(1)

Nếu thay k=f(1)thì:

f(x) =kx (29)

Vậy hàm sốf(x)là nghiệm (26) có dạng cho cơng thức (29) Trở lại tốn ban đầu dễ dàng tính được:f

−2 = π 35

Nếu ta bổ sung thêm điều kiện hàmf(x)liên tục tồnRthìf(x) =kxln Để chứng minh điều ta xét dãy số hữu tỷ(xn)hội tụ x Khi đó: lim

n→∞f(xn) =f( limn→∞xn) = limn→∞kxn hayf(x) =kx

Ví dụ 11 (Khoa Tốn Cơ, 2003)Hàm số xác định với số thựcx, y thỏa mãn:

f(x+y) =f(x) +f(y) + 80xy

Tính f

4

nếu biếtf

1

=

Lời giải

Phương trình hàm cho khơng vế phải chứa hạng tử 80xy Cách thơng thường đưa dạng cách tìm nghiệm riêng Có dạng tương tự phương trình là:(x+y)2=x2+y2+ 2xy Khơng khó để chứng minh f0(x) = 40x2

nghiệm Đặtg(x) =f(x)−f0(x) Phương trình ban đầu trở thành:

g(x+y) =g(x) +g(y), ∀x, y∈R

Theo ví dụ 10 thìg(x) =kxvới mọix∈Q Vậy khẳng định rằngf(x) = 40x2+kx Từ

điều kiệnf

1

= 2suy k=−2,f(x) = 40x2−2x Như vậy:f

4 = 40 4 −2.4

(66)

2 Tính chất hàm tuyến tính

Ví dụ 12 (Khoa Hóa học, 1999)Hàm số f(x) thỏa mãn điều kiện sau với số thựca, b:

f

a+ 2b

=f(a) + 2f(b) Tìm giá trị f(1999)nếu biếtf(1) = 1, f(4) =

Lời giải

Đặtg(x) =f(x)−f(0) Ta có: g

a+ 2b

3

= g(a) + 2g(b)

3 , g(0) = (30)

Thayb= 0vào (30):

ga

3

=

3g(a), a∈R

Thaya= vào (30):

g

2b

=2

3g(b), b∈R

Khi đó:

g(2b) = 3.1

3g(2b) = 3g 2b

3

= 3.2

3.g(b) = 2g(b)

Nhờ tính chất hàm g, phương trình (30) tương đương với:

1

3g(a+ 2b) =

g(a) + 2g(b)

3 ⇔g(a+ 2b) =g(a) + 2g(b)⇔g(a+ 2b) =g(a) +g(2b)

Thay2bbởi cthì:

g(a+c) =g(a) +g(c) ; a, c∈R

Như chứng minh ví dụ 11 thìg(x) =kx Do đóf(x) = f(0) +kx, x∈R Dựa vào điều

kiện f(1) = 1, f(4) = dễ dàng xác định f(0) = −1, k = Vậy hàm cần tìm có dạng: f(x) = 2x−1, x∈Rvàf(1999) = 3997

3 Tính chất hàm mũ

Chúng ta biết tính chất sau hàm mũ: ax+y=axay Ở ví dụ phương

trình hàm số có tính chất tương tự:

f(x+y) =f(x)f(y) (31)

Để giải phương trình (31) chúng tơi giới thiệu cho bạn đọc lý thuyết Cauchy

Ví dụ 13 (Khoa Sinh học, 6/2005) Cho hàm số f thỏa mãn f(x+y) = f(x).f(y) với

x, y∈Q Biết rằngf(4) = 16 Tínhf

−3

2

?

(67)

Ý tưởng chung để giải phương trình (31) dựa vào phép lấy logarit Đặt g(x) = lnf(x) g(x+y) = g(x) +g(y) Theo ví dụ 10 g(x) = kx, ∀x ∈ Q tương đương với f(x) = ekx ≡

ax, ∀x∈

Q(ở đâya=ek) Từ điều kiện f(4) = 16rút raa= Do đóf

−3

=

Nhưng để lập luận chặt chẽ yêu cầu phải chứng minhf(x)>0, ∀ ∈Q Trong (31) thayy= 4−x:

f(x)f(4−x) =f(4) = 16

Điều chứng tỏf(x)6= 0, ∀x∈Q Mặt khácf(x) =f x

2 +

x

2

=fx

2

Vậy chứng minh xongf(x)>0, ∀ ∈Q

Bài tốn khơng trường hợp xlà số hữu tỷ mà khixlà số thực

4 Tính chất hàm logarit

Hàm số logarity= logaxcó tính chất:loga(xy) = logax+ logay.Chúng ta có khẳng định sau:

Nếu hàm số f(x) xác định với số thực dương x, liên tục tập thỏa mãn:

f(1)6= 0; f(xy) =f(x) +f(y), x, y∈R+thì tồn số dươnga6= 1sao chof(x) = logax

Để chứng minh khẳng định xét hàm: g(x) =f(ex)xác định liên tục với x∈R Ta có:g(x+y) =f(ex+y) =f(ex.ey) =f(ex) +f(ey) =g(x) +g(y), suy rag(x) =kx

Vớix >0 thìf(x) =f elnx=g(lnx) =klnx

Theo đề bài: x0 6= 1thì f(x0)6= cho ta k6= Khi a=e

1

k số dương khác Vậy hàm f(x)có thể viết dạng sau:f(x) =klnx= lnx

lna = logax

5 Tính chất hàm lượng giác

Xét hàmy= cosx Ta có tính chất:cos (x+y) = cos (x−y) = cosx.cosy Phương trình hàm tương ứng:

f(x+y) +f(x−y) = 2f(x)f(y), x, y∈R (32)

Trên thực tế khơng có hàm cosin mà cịn nhiều hàm khác có tính chất vậy, phải kể đến làf(x)≡0, nghiệm tầm thường Để loại bỏ nghiệm tầm thường bổ sung thêm điều kiện: số điểmx0thì f(x)6=

Một hàm thú vị thật không dễ thấy hàm cosin hypebolic: cosh (x) =e

x+e−x

2

Thật vậy:

cosh (x+y) + cosh (x−y) =e

x+y+ex−y

2 +

ex+y+e−x+y

2 = e

x(ey+e−y) +e−x(ey+e−y)

2 =

ex+e−x

2

ey+e−y

2 = cosh (x).cosh (y)

Ta có bất đẳng thức: e

x+e−x

2 ≥1 (dấu “=” xảy x= 0) Dựa vào ta có

thể loại bỏ nghiệm hàm cosin hypebolic cách thêm vào điều kiệnf(x)≤1với mọix Giống phương trình Cơ-si khác giả thiết f(x) liên tục với mọix Nhận thấy, nếuf(x)

(68)

tham số)

Chúng ta chứng minh phương trình (32) khơng cịn có nghiệm khác 1) Thay x=x0, y = 0vào (32):2f(x0) = 2f(x0)f(0) Vìf(x0)6= 0nên

f(0) = (33)

2) Thay x = vào (32): f(y) +f(−y) = 2f(0).f(y) Áp dụng (33) nhận f(y) =

f(−y), ∀y∈R Nói cách khácf(x)là hàm chẵn

3) Trong (32) thay xbởi nxvà sau thayy=x:

f((n+ 1)x) = 2f(nx).f(x)−f((n−1)x)

Đây công thức truy hồi Có thể xác định đượcf((n+ 1)x)nếu biếtf(nx), f((n−1)x), f(x) Vì biết f(x) (vì biết f(0) = 1) tính f(2x), f(3x), , f(−2x), f(−3x),

Kết luận: Nếu nghiệmf(x)vàg(x)bằng điểmtthì chúng điểm có dạngnt, n∈Z (Để chặt chẽ ta phải chứng minh phương pháp quy nạp toán học)

4) Trong (32) ta thayy=x:

f(2x) + = 2f2(x) (34)

Đẳng thức giống với:cos (2x) + = cos2x

Từ dễ thấy với xthì f(x)≥ −1 Trong (34) thayxbởi x

2n, đón∈Z:

f

x 2n

=

v u u t1 +f

x 2n−1

2 (35)

Nếu bổ sung giả thiếtf x

2n

≥0và giá trịf(x)đã biết phương trình (34) giúp ta tính tất giá trịf x

2n

, n∈Z

Kết luận: Nếu nghiệm f(x)vàg(x)trùng số điểmtvà giá trị chúng không âm điểm t

2n, n∈Z chúng trùng tất điểm (dùng quy nạp để

chứng minh)

5) Vì f(0) = vàf(x)liên tục x= nên giới hạn đoạn [−ε; +ε],(ε >0) hàm f(x)dương Thật trường hợp ngược lại tồn dãy sốxk hội tụ về0sao chof(xk)≤0

Khi tính liên tục củaf(x)tại điểm0 cho ta: f(0) = lim

k→∞f(xk)≤0 trái với điều kiệnf(0) =

6) Bất đẳng thức < f(ε) ≤1 cho thấy tồn số α = arccosf(ε)và α∈ h0;π

Điều tương đương vớif(ε) = cosα= cosaε, đóa= αε

Theo mục chứng minh chúng trùng điểm có dạng 2εn, n∈N, theo mục chúng trùng điểm có dạng mε

(69)

Ví dụ 14 (Khoa Tốn Cơ, 2005) Tìm giá trị nhỏ hàm số f xác định tập số tự nhiên thỏa mãn điều kiện sau:

f(1) = cos (36)

f(n+ 1) =f(n).cos 1− q

1−(f(n))2.sin 1, n∈N (37)

Lời giải

Phương trình (37) cho phép tìm giá trị f(n+ 1) biết giá trị củaf(n) Vì biếtf(1) = cos 2nên tính đượcf(2), f(3),

Chúng ta tính vài giá trị đầu với hy vọng tìm quy luật Ta có: f(2) = cos 2.cos 1− |sin 2|.sin = cos 2.cos 1−sin 2.sin = cos

f(3) = cos 3.cos 1− |sin 3|.sin = cos 3.cos 1−sin 3.sin = cos

f(4) = cos 4.cos 1− |sin 4|.sin = cos 4.cos + sin 4.sin = cos

f(5) = cos 3.cos 1− |sin 3|.sin = cos 3.cos 1−sin 3.sin = cos

Như vậyf(2k) = cos 3vàf(2k+ 1) = cos 4vớik∈N∗ Điều chứng minh đơn giản nhờ

phương pháp quy nạp:

Vớik= 1:f(2) = cos 3, f(3) = cos Từ (37) chon= 2k+ 1:

f(2 (k+ 1)) =f((2k+ 1) + 1) =f(2k+ 1).cos 1− q

1−(f(2k+ 1))2.sin = cos 4.cos 1−

q

1−(cos 4)2.sin = cos 4.cos + sin 4.sin = cos

Tương tự:

f(2 (k+ 1) + 1) =f(2 (k+ 1)).cos 1− q

1−(f(2 (k+ 1)))2.sin = cos 3.cos 1−

q

1−(cos 3)2.sin =cos3.cos1−sin 3.sin = cos

Như hàmf nhận giá trị là:cos 2,cos 3, cos Giá trị nhỏ số làcos

Bài tập áp dụng

Bài 1.(Khoa Tốn học tính tồn Điều khiển, 1997) Tồn khơng hàm tuyến tính y=f(x)

thỏa mãn điều kiện sau với số thựcx:

2f(x+ 2) +f(4−x) = 2x+

Bài (Khoa Tốn học tính tồn Điều khiển, 1997) Tồn không hàm bậc hai y =f(x)

thỏa mãn điều kiện sau với số thựcx:

(70)

Bài (Khoa Toán học tính tồn Điều khiển, 1996) Tìm hàm sốy =f(x) thỏa mãn điều kiện sau với mọix6= 0:

f(x) + 3xf

1

x

= 3x2

Bài (Khoa Hóa học, 6/2001) Cho hàm sốf(x)thỏa mãn điều kiện sau với mọix:

f(x+ 1) =f(x) + 2x+

Biết f(0) = 0, tínhf(2001)?

Bài (Khoa Tốn học tính tồn Điều khiển, 2002/Olympic Bungari 1968) Tìm tất hàm sốf(x)thỏa mãn:

xf(y) +yf(x) = (x+y)f(x)f(y), ∀x, y∈(−∞,+∞)

Bài (Khoa Sinh học, 6/2005) Cho hàm sốf thỏa mãn

f(x−y) =f(x)−f(y), ∀x, y∈Q

Biết f(6) =−√3, tínhf

−5

4

?

Bài (Khoa Sinh học, 6/2005) Cho hàm sốf thỏa mãn

f(x−y) = f(x)

f(y), ∀x, y∈Q

Biết f(3) = 27, tínhf

−5

2

?

Bài (Khoa Toán cơ, 2005) Hàm sốf xác định tậpZthỏa mãn:

f(1) =cos1, f(n+ 1) =f(n).cos1−sinn.sin 1, n∈Z

(71)

Bạn đọc tìm tịi

Bí ẩn tập đóng lồng nhau

Trần Bạt Phong, Đại học Khoa học Tự nhiên, ĐHQG Tp HCM

Giới thiệu Trong trình học tập, hẳn có "khám phá", tìm tịi đáng nhớ Đó chứng minh "lạ" cho định lý "quen", nhận xét đơn giản có nhiều ứng dụng, đơn giản thắc mắc "vì lại thế, mà không kia?" Những tìm tịi nhỏ, khơng mới, tin chúng phần quan trọng chu trình "học-hiểu-thích", chúng thật đáng q MụcBạn đọc tìm tịi hi vọng nhận chia từ bạn tìm tịi, suy nghĩ

Tơi xin bắt đầu với kết quen thuộc, dãy cầu đóng "thắt dần" khơng gian metric đầy đủ có giao khác rỗng Để phát biểu điều cách chặt chẽ,

chúng ta thống số ký hiệu Xét không gian metric(X, d) Với mỗix∈X vàr≥0 ta

ký hiệuB0(x, r)là cầu đóng tâmxbán kínhr, tức

B0(x, r) :={y∈X|d(x, y)≤r}

Bây kết phía phát biểu sau:

Mệnh đề Cho (X, d) không gian metric đầy đủ Giả sử ta có dãy cầu đóng lồng

B0(x1, r1)⊃B0(x2, r2)⊃ vàrn→0 Khi

\

n=1

B0(xn, rn)6=∅

Chứng minh Để ý với mỗim≥nthìxm∈B0(xn, rn), tức làd(xn, xm)≤rn Do rn →0 nên

xn dãy Cauchy, do(X, d)đầy đủ nênxn hội tụ phần tửxtrong X Bây giờ, với

mỗinthìxm∈B0(xn, rn)vớim≥nvàxm→x, suy rax∈B0(xn, rn)vì cầu đóng

tập đóng Vậyx∈

\

n=1

B0(xn, rn)và ta có đpcm

Bạn đọc tinh ý thấy thật ∞

\

n=1

B0(xn, rn)chỉ có phần tử (vì sao?)

Bây phân tích kỹ Mệnh đề ta thấy có giả thiết bản: 1) (X, d)đầy đủ

2) B0(xn, rn)là cầu đóng (lồng nhau)

3) rn →0(tính thắt dần)

(72)

Mệnh đề cho ta tính chất quan trọng không gian đầy đủ: "một dãy cầu đóng thắt dần có giao khác rỗng" Một cách tự nhiên ta thắc mắc: phải tính chất đặc trưng cho tính đầy đủ khơng gian? Nói cách khác, biết dãy cầu thắt

dần khơng gian metric (X, d)đều có giao khác rỗng, liêu ta kết luận (X, d)

đầy đủ hay khơng? Khơng q khó khăn, kiểm tra điều ngược lại

Mệnh đề Cho(X, d)là không gian metric Giả sử dãy cầu đóng lồng

B0(x1, r1)⊃B0(x2, r2)⊃

vớirn→0đều thỏa mãn

\

n=1

B0(xn, rn)6=∅ Khi đó(X, d) đầy đủ

Chứng minh Lấyxn dãy Cauchy trongX, ta cần chứng minhxn hội tụ Tất nhiên, ta

cần chứng minh có dãy củaxn hội tụ đủ (vì sao?) Từ đó, chuyển qua dãy

cần, ta giả sử (vì sao?)

d(xn, xn+1)≤2−n, ∀n∈N

(điều nhằm mục đích để P∞

n=1d(xn, xn+1)<∞) Đặt

rn=

X

k=n

d(xk, xk+1)

Khi dễ dàng kiểm tra rằngrn dãy giảm về0và

d(xn, xm)≤rn−rm, ∀n≥m

Từ tính chất thứ hai ta suy

B0(x1, r1)⊃B0(x2, r2)⊃

Vậy theo giả thiết, tồn x∈

\

n=1

B0(xn, rn) Cuối xn →xbởi d(xn, x)≤rn →0

điều kết thúc chứng minh Vấn đề 2:Quả cầu đóng

Một câu hỏi tự nhiên liệu ta thay cầu đóng Mệnh đề

cầu mở Đây câu hỏi dễ trongR(với metric thơng thường) ta tìm phản

ví dụ: xét khoảng mở(0,n1)(là "quả cầu mở" tâmxn=21n với bán kínhrn= 21n →0),

ta có

(0,1)⊃(0,1 2)⊃(0,

1 3)⊃

nhưng ∞

\

n=1

(0,

n) =∅ Bạn đọc tinh ý nhận tâmxn cầu ví dụ

vẫn hội tụ (về 0) giới hạn nằm cầu, từ bạn giải thích điều phá vỡ phần chứng minh Mệnh đề

(73)

Mệnh đề Cho(X, d)là không gian metric đầy đủ Giả sửAn dãy tập đóng, khác

rỗng, lồng

A1⊃A2⊃ vàdiam(An)→0 Khi

\

n=1

An6=∅ (và đó, có phần tử)

Ở ta ký hiệu diam(A)là đường kính tậpA, tức

diam(A) = sup{d(x, y)|x, y∈A}

Chứng minh Mệnh đề hoàn toàn tương tự Mệnh đề xin dành lại cho bạn đọc Vấn đề 3.Tính thắt dần

Bây có câu hỏi giả thiết cuối cùng, câu hỏi thú vị nhất, liệu giả

thiếtrn →0trong Mệnh đề bỏ bớt?

Trước đến với câu trả lời, mời bạn chứng minh nếuX không gian định chuẩn

(đầy đủ) giả thiếtrn→0thực làthừa

Mệnh đề ChoX không gian Banach Giả sử ta có dãy cầu đóng lồng

B0(x1, r1)⊃B0(x2, r2)⊃

Khi ∞

\

n=1

B0(xn, rn)6=∅

Hướng dẫn Trong không gian định chuẩn từB0(xm, rm)⊂B0(xn, rn)ta có bất đẳng thức

||xn−xm|| ≤rn−rm, ∀m≥n

Để chứng minh bất đẳng thức này, xét điểm a giao điểm tia nốixn, xmvà mặt cầu

∂B0(xm, rm), tức

a=xm+rm

xn−xm

||xn−xm||

Bây trở lại trường hợp không gian metric Thật bất ngờ, trái với cảm giác sau làm việc với không gian định chuẩn, trường hợp không gian metric tổng quát (đầy đủ)

điều kiệnrn→0làkhơng thể bỏ bớt Để thấy điều đó, xây dựng khơng gian metric

đầy đủ(X, d)mà dãy cầu đóng (khơng thắt dần) có thê có giao rỗng Từ Mệnh

đề ta thấy không gian X phải "bệnh hoạn" (ít khơng khơng gian định chuẩn)

Sự kiện bán kính khơng tiến gợi ý cho nhận xét đơn giản hữu ích sau

đây Giả sử metric trênX thỏa mãn: tồn >0sao cho

d(x, y)≥, ∀x6=y

Thì (X, d)là khơng gian đầy đủ (vì dãy Cauchyyn dãy nếunđủ lớn)

Từ phân tích ta có phản ví dụ sau Xét X tập đếm gồm điểm

(phân biệt){xn}∞n=1, trênX ta trang bị metricdđịnh nghĩa bởi: d(xn, xm) = nếun=m

d(xn, xm) = + min{

1 n,

1

(74)

Bạn đọc dễ dàng kiểm tradlà metric, theo nhận xét thì(X, d)đầy đủ Bây

giờ lấyrn= + 1/n ta thấyB0(x1, r1) =X

B0(xn, rn) =X\{x1, , xn−1} vớin >1

(75)

Cuộc thi giải toán MathVn Phần A - Đề toán dành cho Học sinh

A25 Cho tam giác ABC nội tiếp đường tròn (O) A1 điểm BC; A2 điểm đối xứng củaAquaO Đường thẳngA1A2 lại cắt(O)tạiA3 LấyA4 đối xứng vớiAquaOA1 Đường thẳng qua A2, vng góc vớiBC lại cắt (O)tại A5 Chứng minh rằng,A3A5 qua trực tâm

tam giácABC

Nguyễn Minh Hà, Khối THPT chuyên ĐHSP Hà Nội

A26 Cho điểm nằm bên (hoặc biên) hình trịn bán kính r > cho có điểm nằm tâm hình trịn khoảng cách hai điểm khơng nhỏ Hỏir chọn nhỏ bao nhiêu?

TheoPaul Bateman, Paul Erdos

A27.Cho{xn}n≥1là dãy số thực thỏa mãn

1 +x2

n

+ 2nxn−2 =

với số nguyên dươngn Tính lim

n→∞

nln(2−2nxn)

xn

Dương Việt Thông, Khoa Cơ bản, ĐHSP Kỹ thuật Nam Định

A28.Cho tam giácABC vàXY Z, chứng minh

cos2X cot

A

2 + cos

2Y

2 cot

B

2 + cos

2Z

2 cot

C

2 ≥cos

X

2 cos

Y

2 cos

Z

2 Trần Quang Hùng, Đại học Khoa học Tự nhiên - ĐHQG Hà Nội

A29 Cho tam giác không cân ABC nội tiếp đường tròn (O) Hai tiếp tuyến cuả (O) B

C cắt S Tiếp tuyến của(O) A cắt BC V Gọi M trung điểm cạnh AC Kí hiệu

P, Q, I tương ứng điểm chung cuả cặp đường thẳng (V M, AB),(AS, BC),(AQ, CP) Trên

M P, M Qlần lượt lấy hai điểmE, F choIE||M Q, IF||M P Giả sử M P cắtAQở H, M Qcắt

CP K.Chứng minh rằngF H, KE vàM I đồng quy

Hoàng Quốc Khánh, Học sinh THPT chuyên Vĩnh Phúc, Tỉnh Vĩnh Phúc

A30 Cho tứ giác nội tiếp ABCD Gọi M, N theo thứ tự trung điểm AC, BD Đặt E =

AB∩CDvàF =AB∩CD Chứng minh rằng:

M N EF =

1

AC BD −

BD AC

(76)

A31.Giải hệ phương trình sau vớix, y, z∈R  

2x3+ +y= 37x+ 70 y3+z= 18y+ 40

3z3+x= 56z+ 100

Lê Nguyễn - Sinh viên Lớp TC0662A1, Đại học Cần Thơ

A32.Tìm số nguyên dươngmbé cho tồn tạia >1để hệ phương trình đồng dư 

x≡a3x modm y≡a5y modm z≡a7z modm

thỏa mãn với mọix, y, z∈N

Vladimir Lesko, Khoa Đại số, Hình học Tin học, Đại học Sư phạm Volgograd, LB Nga

A33.Giả sử a,b,c số thực dương Ký hiệu

Sr=ar(a−b)(a−c) +br(b−c)(b−a) +cr(c−a)(c−b)

Vớirlà số thực dương Khi ta có bất đẳng thức

S22(a2+b2+c2) + (ab+bc+ca)(a2+b2+c2−ab−bc−ca)(a−b)2(b−c)2(c−a)2≥2abcS1S2

Kim Đình Sơn, Học sinh lớp 12A1, THPT Chuyên Vĩnh Phúc

Phần B - Đề toán dành cho Sinh viên

B7.Chof :R→Rvà Z ∞

−∞

|f(x)|dx <∞

a Tìm phản ví dụ cho phát biểu lim

|x|→∞f(x) = b Giả sửf0 liên tục và

Z ∞

−∞

|f0(x)|dx <∞, liệu lim

|x|→∞f(x) =

Phan Thành Nam - Khoa Toán, Đại học Copenhagen, Đan Mạch

B8 ChoA tập mở, bị chặn R Hỏi độ đo Lebesgue A A có hay khơng?

Lâm Bạt Phong - Đại học Khoa học Tự nhiên, ĐHQG Tp.HCM

B9.Chứng minh

|sin(At)| ≤e|A−DA|t

Trong đó, vớiA= (aij)n, kí hiệu|A|= (|aij|)n,DA=diag(a11, a22, , ann) Quan hệA≥B nghĩa

(77)

Phần C - Các vấn đề mở1

C4.Xét phương trình vi phân

x00(t) +a(t)x2n+1(t) = 0, 0≤t <∞

Trong đónnguyên dương, a(t)là hàm khả vi liên tục vàa(t)≥a0>0với t

a Nếua0(t)đổi dấu hữu hạn lần chứng tỏ nghiệmx(t)bất kì bị chặn b Nếu bỏ điều kiệna0(t)đổi dấu hữu hạn lần, liệu khẳng định đúng? TheoKhoa Toán, Trường Đại học Khoa học Nghệ thuật McMicken, ĐH Cincinnati, Hoa Kỳ

C5.Xét ma trận đối xứng cấp8m có dạng

A=

  

A11 A12 A18 A21 A22 A28 A81 A82 A88

  

Trong blockAij ma trận đối xứng cấpm Giả sửAcó đúngk giá trị riêng âm, chứng minh ma trậnA11+A22+ +A88có khơng quákgiá trị riêng âm, (số giá trị tính với bội

của đa thức đặc trưng) TheoProblems and Solutions, SIAM

C6.Tìm hàmf từ không gian SchwartzS(R)của hàm giảm nhanh với giá trênR+={x∈

R : x≥0}, với biến đổi Fourier-Laplace

e

f(z) =

Z ∞

0

eizxf(x)dx

khơng có khơng điểm trênC+={z∈

C : Imz≥0} TheoProblems and Solutions, SIAM

1Các toán phần chọn từ vấn đề khoa Toán, trường Đại học trên

(78)

Lời giải kì trước

A1.Cho tam giác khơng cânABC nội tiếp đường trịn tâmO.M điểm mặt phẳng

AM∩(O) =A1 BM∩(O) =B1 CM ∩(O) =C1

Chứng minh tồn hai điểmM cho tam giác A1B1C1

Solution by Francisco Javier García Capitán, Priego de Córdoba, Spain

Letkbe the power of the pointM with respect the circle(O) Then we have

M A·M A1=M B·M B1=M C·M C1=k2,

soA1,B1, C1 are the inverses ofA, B, C with respect circle with centerM and radiusk

We take into account that by such an inversion we have

A1B1=

k2·AB

M A·M B, C1A1=

k2·CA M C·M A

Since we wantA1B1C1 to be equilateral we must have

A1B1=C1A1⇒

k2·AB M A·M B =

k2·CA M C·M A ⇒

M B M C =

(79)

that is M must belongs to the A−Apolonian circle for ABC, and in the same way to the three Apollonian circles, soM must be one of the two isodyamic points of triangleABC (we exclude the trivial case in whichM belongs to the circumcircle ofABC andA1B1C1degenerates in a point)

A2 Cho tứ giác ABCD ngoại tiếp đường tròn (O) Tiếp điểm (O) AB, BC, CD, DA

lần lượt M, N, P, Q Đường thẳng qua M vng góc với M N cắt P Q I Đường thẳng qua M

vng góc với M QcắtP N ởJ Chứng minh rằngAI||BJ

Solution by Ercole Suppa, Teramo, Italy

First we prove the following lemma:

Lemma Let ABC be a triangle inscribed in the circle (O) Let X be an arbitrary point on AB

and Y =OX ∩AC Let t be the tangent at A to the circumcircle The lines through O parallel to

AB andAC intersectt atU,V respectively Prove that U X||V Y Proof.LetD=AB∩OU,E =AC∩OV andF =AB∩V Y

Taking into account the equality of the opposite sides of the parallelogramADOEand the similarity

4U DA∼ 4AEV, 4DXO∼ 4EOY, we have:

U D DX =

U D AD·

AD DX =

AE EV ·

EO DX =

AE EV ·

EY DO =

AE EV ·

EY AE =

EY EV

Therefore the triangles4DU X and4EY V are similar, hence

(80)

and the proof of the lemma is complete Coming back to the problem denoteR=M I∩(O)andS=M J∩(O)as shown in figure:

We can notice thatQS∩RN =Obecause∠QM S=∠RM N= 90◦, so the Pascal theorem applied to the hexagon QP N RM S yields that I, O, J are collinear Thus the result follows at once from the lemma applied to the triangle4M RS sinceAO||M S,BO||M R

Solution by Francisco Javier García Capitán, Priego de Córdoba, Spain

We observe from construction that line BJ is the reflection of line AI with respect to the angle bisectormof angleQM N

If we consider that M, Q, N are fixed points and P is a variable point on the circle, then, as P

(81)

The map I→J fromr tos is a homography For, we can see that the mapI→J can be defined as follows: given anyI∈r, we draw line IQintersecting the circle again at someP Then we draw

P N, andJ is the intersection of this lineP N with the reflection of line M I on the angle bisector

mof angleQM N

Therefore the problem will be solved if we can find three positions of point P on the circle for which the linesAI andBJ are parallel First we suppose that pointP is confounded withN,

and the result follows from applying the following lemma to triangleM N Q

Lemma 1.IfXY Z is the tangential triangle ofABC, and the perpendicular toCAatA intersects

BC atD, thenDZ andCY are parallel

Proof Let’s make an angle chase Suppose thatC690◦ We have the semiinscribed and inscribed angles ∠Y CA = ∠Y AC = ∠ABC = B ⇒ ∠Y CB = B+C We also have ∠ZBA = ∠ZAB =

∠ACB =C ⇒ ZBD = A Moreover ∠AZB = 180◦−2C = 2(90◦−C) = 2∠ADB, hence D is on the circle centered at Z with radius ZA = AB Therefore the triangle ZDB is isosceles with

(82)

If we have C>90◦, we get as before that∠Y CA=∠Y AC=∠ABC =B⇒∠Y CB=B+C and

∠Y CD =A We also have ∠ZBD=A and ∠ZBA=∠ZAB =∠ACB = 180◦−C In this case we have∠ADB = 90◦−(180◦−C) =C−90◦ and ∠AZB= 180◦−2(180◦−C) = 2C−180◦ = 2(C−90◦) = 2∠ADB, henceD is on the circle centered atZ with radiusZA=AB Therefore the triangle ZDB is isosceles with ZD = ZB and ∠ZDB = A =∠Y CD and lines AY and DZ are parallel

In the same way, when the point P reaches the position of point Q the we have that the result also is true by applying Lemma to triangleM QN

As a third case, whenP =M, then we haveP =M =I =J andAI ≡BJ ≡AB, soAI andBJ

are parallel

A3.Cho tam giác ABC vớila, lb, lc tương ứng phân giác góc A, B, C Chứng minh a+b

la+lb

+ b+c

lb+lc

+ c+a

lc+la ≥2√3

Lời giải Nguyễn Mạnh Dũng, 12A2 Toán, ĐHKHTN-ĐHQGHN

Đầu tiên ta chứng minh bổ dề:

Bổ đề.Cho tam giác ABC,

3la ≤b+c−a

2

Chứng minh.Áp dụng bất đẳng thức AM-GM ta có

a

2 +

3la =a +

2√bcp3p(p−a)

b+c ≤

a

2+

(b+c)3(p−2a)+p

b+c =

a

2 +

4p−3a

(83)

Ta có điều phải chứng minh Tương tự ta chứng minh

3lb≤c+a−b

2,

3lc≤a+b−c

2

Quay lại toán, áp dụng bổ đề ta

a+b la+lb +

b+c lb+lc +

c+a lc+la ≥

X2

3(a+b)

a+b+ 4c

Mặt khác, áp dụng bất đẳng thức Cauchy-Schwarz, ta có

X a+b

a+b+ 4c ≥

4(a+b+c)2

P(a+b)(a+b+ 4c) = +

(a−b)2+ (b−c)2+ (c−a)2 2(a2+b2+c2) + 10(ab+bc+ca) ≥1

Do

a+b la+lb

+ b+c

lb+lc

+ c+a

lc+la ≥2√3

Dấu đẳng thức xảy khia=b=chay tam giác ABC tam giác

Solution by Francisco Javier García Capitán, Priego de Córdoba, Spain

By using the AM-GM inequality, we have

l2a= 4bcs(s−a)

(b+c) 6s(s−a),

wheresstands for the semiperimeter of triangleABC Adding the three similar inequalities, we get, by using the Cauchy Schwarz inequality

la+lb+lc= 1·la+ 1·lb+ 1·lc6√3

q

l2

a+l2b +l2c

6√3ps(s−a+s−b+s−c) =√3s

Next we use first the Chebyshev inequality, then the HM-AM inequality

a+b la+lb

+ b+c

lb+lc

+ c+a

lc+la >

(2a+ 2b+ 2c)

1

la+lb

+

lb+lc

+

lc+la

3

>

4s

2(la+lb+lc)

3 =

6s la+lb+lc >

6

3 =

3

and the problem is solved

Lời giải Nguyễn Duy Khánh, Lớp Toán Tiên tiến, ĐHKHTN, ĐHQG Hà Nội

Ta có

la=

p

bcs(s−a)

b+c ≤

p

s(s−a)

Tương tựlb≤

p

s(s−b)suy

a+b la+lb

≥ p a+b

s(s−a) +ps(s−b) ≥

a+b

p

(84)

Hoàn toàn tương tự

b+c lb+lc

≥p b+c

s(s−b) +ps(s−c) ≥

b+c

p

a(a+b+c)

c+a lc+la ≥

c+a

p

s(s−c) +ps(s−a) ≥

c+a

p

b(a+b+c)

Nên

a+b la+lb +

b+c lb+lc +

c+a lc+la ≥

1

a+b+c( a+b

√ c +

b+c √

a + c+a

√ b )

Vì ta cần

a+b √

c + b+c

√ a +

c+a √

b ≥2

p

3(a+b+c)

hay dạng tương tự :

m2+n2

p +

n2+p2

m +

p2+m2

n ≥2

p

3(m2+n2+p2)

⇔X(m−n)2(1

m+

1

n−

2

p

3(m2+n2+p2) +m+n+p)≥0

Theo bất đẳng thức AM-GM ta cóp3(m2+n2+p2)≥m+n+p, suy ra

p

3(m2+n2+p2) +m+n+p≥2(m+n+p)

Do m+ n− p

3(m2+n2+p2) +m+n+p ≥

1

m+

1

n−

1

m+n+p =

(m+n)2+mp+np−mn mn(m+n+p) ≥0

Vai trò m, n, plà nên ta có điều phải chứng minh Đẳng thức xảy tam giác

Các bạn có lời giải tốt: Nguyễn Đình Thi (THPT Chuyên Lương Văn Chánh, Tuy Hòa, Phú Yên), Võ Quốc Bá Cẩn (ĐH Cần Thơ, Tp Cần Thơ), Ercole Suppa (Teramo, Italy)

A5.Cho số dương a, b, c Chứng minh bất đẳng thức sau với mọik≥1:

ak+bk+ck≥ a

2(bk+ck) a2+bc +

b2(ck+ak)

b2+ca +

c2(ak+bk)

c2+ab

Lời giải Nguyễn Đình Thi (THPT Chun Lương Văn Chánh, Tuy Hịa, Phú Yên)

Đặt bc

a2 =x; ca b2 =y;

ab

c2 =z⇒xyz= 1Ta có

ak+bk+ck> a

2 bk+ck

a2+bc +

b2 ck+ak b2+ca +

c2 ak+bk c2+ab

⇔ak+bk+ck >b k+ck

1 + bc

a2

+c

k+ak

1 +ca

b2

+a

k+bk

1 + ab

c2

= b

k+ck

1 +x +

ck+ak

1 +y +

ak+bk

1 +z

(85)

> bk+ck(y+ 1)(z+ 1) + ak+ck(x+ 1)(z+ 1) + ak+bk(y+ 1)(z+ 1)

⇔ ak+bk+ck(2 +x+y+z+xy+yz+zx)>

>ak(2 + 2x+y+z+xy+zx) +bk(2 +x+ 2y+z+xy+yz) +ck(2 +x+y+ 2z+yz+zx)

⇔akyz+bkzx+ckxy>akx+bky+ckz ⇔ak.ca

b2 ab c2 +b

k.bc a2

ab c2 +c

k.bc a2

ca b2 >a

k.bc a2 +b

k.ca b2 +c

k.ab c2

Nhân vế bất đẳng thức choabcta bất đẳng thức tương đương

ak+3+bk+3+ck+3>ak−1b2c2+bk−1c2a2+ck−1a2b2 ⇔ak−1 a4−b2c2

+bk−1 b4−c2a2

+ck−1 c4−a2b2 >0

Khơng tính tổng qt, giả sửa>b>c, dok>1nên ak

>bk

>ck

a4−b2c2>b4−c2a2>c4−a2b2

Do theo bất đẳng thức Chebysev ta có

ak−1 a4−b2c2+bk−1 b4−c2a2+ck−1 c4−a2b2> ak+bk+ck a4+b4+c4−a2b2−c2a2−b2c2>0

Vậy ta có ta có điều phải chứng minh Đẳng thức xảy khia=b=c Nhận xét.Bằng cách chứng minh tương tự ta thu bất đẳng thức

bc bk+ck

a2+bc +

ca ck+ak

b2+ca +

ab ak+bk

c2+ab >a k

+bk+ck

Bất đẳng thức chứng minh cuối

ak.bc a2 +b

k.ca b2 +c

k.ab c2 6a

k.ca b2

ab c2 +b

k.bc a2

ab c2 +c

k.bc a2

ca b2

Do từ bất đẳng thức ta thu chuỗi bất đẳng thức thú vị

bc bk+ck

a2+bc +

ca ck+ak

b2+ca +

ab ak+bk

c2+ab >a

k+bk+ck

>a

2 bk+ck

a2+bc +

b2 ck+ak

b2+ca +

c2 ak+bk

c2+ab

Lời giải Võ Quốc Bá Cẩn (ĐH Cần Thơ, Tp Cần Thơ)

Bất đẳng thức cần chứng minh viết lại sau

X

cyc ak

1− b b2+ca−

c2 c2+ab

≥0

Tương đương

X

cyc

akbc(a2−bc) (b2+ca)(c2+ab) ≥0

hay

ak−1(a4−b2c2) +bk−1(b4−c2a2) +ck−1(c4−a2b2)≥0

Bất đẳng thức viết lại thành

(86)

Áp dụng bất đẳng thứcAM−GM ta

(k−1)ak+3+ 2bk+3+ 2ck+3≥(k+ 3)ak−1b2c2,

(k−1)bk+3+ 2ck+3+ 2ak+3≥(k+ 3)bk−1c2a2,

(k−1)ck+3+ 2ak+3+ 2bk+3≥(k+ 3)ck−1a2b2

Cộng tương ứng vế theo vế bất đẳng thức chia hai vế chok+ ta bất đẳng thức cần phải chứng minh Đẳng thức xảy khia=b=c

A6 Vớia, b, c số thực dương choa+b+c= Chứng minh

a2+b2c b+c +

b2+c2a c+a +

c2+a2b a+b ≥

2

Lời giải Võ Quốc Bá Cẩn (ĐH Cần Thơ, Tp Cần Thơ)

Áp dụng bất đẳng thức AM-GM, ta có

b2c

b+c =bc− bc2

b+c ≥bc−

(b+c)2c

4(b+c) = 4bc−

1 4c

2

Từ dẫn đến

b2c b+c +

c2a c+a+

a2b a+b ≥

3

4(ab+bc+ca)− 4(a

2+b2+c2) =5

4(ab+bc+ca)−

Bây giờ, áp dụng bất đẳng thức Cauchy-Schwarz, ta

a2 b+c+

b2 c+a+

c2

a+b = (a+b+c)

a

b+c+ b c+a+

c a+b

−(a+b+c)

= a

b+c + b c+a+

c

a+b −1≥

(a+b+c)2

2(ab+bc+ca)−1 =

1

2(ab+bc+ca)−1

Bây ta cần chứng minh

5

4(ab+bc+ca)− 4+

1

2(ab+bc+ca)−1≥

Tương đương

5

4(ab+bc+ca) +

1

2(ab+bc+ca) ≥ 23 12

Lại áp dụng bất đẳng thức AM-GM

5

4(ab+bc+ca) +

1

2(ab+bc+ca) =

9

2(ab+bc+ca) +

1 2(ab+bc+ca)

−13

4 (ab+bc+ca)

≥3−13

4

(a+b+c)2

3 = 23 12

(87)

Lời giải Nguyễn Đình Thi (THPT Chuyên Lương Văn Chánh, Tuy Hòa, Phú Yên)

Theo bất đẳng thức Schur

(a+b+c) a2+b2+c2+ab+bc+ca

>3 a2(b+c) +b2(c+a) +c2(a+b)

⇒a2(b+c) +b2(c+a) +c2(a+b)6 a

2+b2+c2+ab+bc+ca

3

Do

a2 b+c +

b2 c+a+

c2 a+b =

a4 a2(b+c)+

b4 b2(c+a)+

c4 c2(a+b)

> a

2+b2+c22

a2(b+c) +b2(c+a) +c2(a+b) >

3 a2+b2+c22

a2+b2+c2+ab+bc+ca

Lại có

b2c b+c+

c2a c+a+

a2b a+b =

b2c2 bc+c2 +

c2a2 ca+a2 +

a2b2 ab+b2 >

(ab+bc+ac)2

a2+b2+c2+ab+bc+ca

Do theo bất đẳng thức với ý

a2+b2+c2>ab+bc+ca

Ta có

a2+b2c b+c +

b2+c2a c+a +

c2+a2b a+b >

3 a2+b2+c22

a2+b2+c2+ab+bc+ca+

(ab+bc+ac)2 a2+b2+c2+ab+bc+ca

= a

2+b2+c22

a2+b2+c2+ab+bc+ca+

a2+b2+c22

a2+b2+c2+ab+bc+ca +

(ab+bc+ac)2

a2+b2+c2+ab+bc+ca

>

2.a2+b2+c2+ab+bc+ca2

a2+b2+c2+ab+bc+ca+

a2+b2+c2+ab+bc+ca2

2 (a2+b2+c2+ab+bc+ca)

=a2+b2+c2+ab+bc+ca>2

3(a+b+c)

2

=

Vậy ta có điều phải chứng minh Đẳng thức xảy khia=b=c= 13

Lời giải Nguyễn Mạnh Dũng, 12A2 Toán, ĐHKHTN-ĐHQGHN

Theo bất đẳng thức Cauchy-Schwarz, ta có:

b2c b+c +

c2a c+a+

a2b a+b ≥

(ab+bc+ca)2

a2+b2+c2+ab+bc+ca ≥

3abc

a2+b2+c2+ab+bc+ca

Ta cần chứng tỏ

a2 b+c+

b2 c+a+

c2 a+b+

3abc

a2+b2+c2+ab+bc+ca ≥

2(a+b+c)

Coi bất đẳng thức nhưA≥B,

A= X

cyclic a2 b+c −

1

X

cyclic

a, B= a+b+c −

3abc

(88)

Biến đổi đơn giản, thu

A=a+b+c

X

cyclic

(b−c)2

(a+b)(a+c), B=

X

cyclic

(5a+b+c)(b−c)2

12(a2+b2+c2+ab+bc+ca)

Như vậy, viết bất đẳng thức sau

Sa(b−c)2+Sb(c−a)2+Sc(a−b)2≥0

Với kí hiệu

Sa = 6(a+b+c) (a+b)(a+c)−

5a+b+c

a2+b2+c2+ab+bc+ca

và đối xứng cho trường hợpSb, Sc

Do 6(a+b+c) > 5a+b+c (a+b)(a+c) < a2 +b2+c2 +ab +bc+ca, nên S a >

Tương tự có Sb >0, Sc >0 Như ta đến điều phải chứng minh Đẳng thức xảy

(89)

Nhìn giới

Kỳ thi Qualify cho nghiên cứu sinh Mỹ

Trong chương trình giáo dục Mỹ, sinh viên sau tốt nghiệp cử nhân xin học tiếp để lấy Tiến sĩ (Ph.D.) mà khơng cần có Thạc sĩ Chương trình học dành cho ngành Tốn thường năm, giai đoạn đầu (khoảng năm) học môn (có thể xem có ý nghĩa tương đương với chương trình Thạc sĩ, nội dung khó hơn), giai đoạn cịn lại tập trung nghiên cứu để hồn thành luận văn Thơng thường, để đánh dấu kết thúc giai đoạn đầu, sinh viên phải đậu kỳ thi kiểm tra chất lượng, gọi Qualify exam Nội dung mà mức độ khó dễ kỳ thi phụ thuộc vào trường, nhiên nhìn chung kiểm tra hiểu biết sinh viên môn học bản, định sinh viên tiếp tục vào nghiên cứu hay khơng Với mục tiêu đó, đề thi Qualify thường hỏi vào điều bản, không đánh đố, để trả lời sinh viên phải nắm vững kiến thức mức độ "hiểu" "thuộc" Thật ra, sinh viên đại học Việt Nam học vững kiến thức mơn học đó, hồn tồn trả lời câu hỏi rơi vào lĩnh vực Mặt khác, chúng tơi thấy nhiều sinh viên năm thứ ba, thứ tư đầu tư sức lực vào kỳ thi Olympic sinh viên Toàn quốc, vốn đề gồm tốn phần giải tích biến đại số tuyến tính ma trận (tương đương với mơn học học kỳ năm thứ nhất) Đây lãng phí thời gian bạn có ý định theo đuổi chun ngành Tốn Do đó, chúng tơi xin giới thiệu số đề thi Qualify, với hi vọng nguồn tài liệu tham khảo tốt để bạn sinh viên tự kiểm tra kiến thức mình, đồng thời có đường hướng học tập đắn, đặc biệt bạn có ước mơ học Tốn lâu dài

Dưới đây, xin giới thiệu đề thi Qualify Đại học Indiana Mỹ năm 2009 Các bạn tham khảo nguyên văn đề thi, đề thi năm trước trang web trường

http://www.math.indiana.edu/programs/graduate/tiers/

Tier I Analysis Exam: January 2009

Hướng dẫn: Cố gắng trả lời tất câu hỏi Mọi tốn có điểm số

Bài Giả sửf vàg hàm liên tục đềuR→R Nếuf vàg bị chặn, chứng minh

f g liên tục Cho ví dụ để chứng tỏ nếuf hoặcg khơng bị chặn tích f g khơng liên tục (chứng minh rõ ràng phản ví dụ khơng liên tục đều)

Bài Cho hàm nhận giá trị thựcf, g∈C2(

R)vàh∈C1(R2)sao cho

f(0) =g(0) = 0, f0(0) =g0(0) =h(0,0) =

Chứng minh hàm hàm số H:R2→

Rcho

H(x, y) =

f(x)

Z

0

g(y)

Z

0

h(s, t)dsdt+1 2x

2+by2

(90)

Bài ChoH ={(x, y, z)∈R3|z >0, x2+y2+z2 =R2}, tức nửa mặt cầu tâm bán kínhRtrongR3 CHoF :R3→R3là trường vector

F(x, y, z) ={x2(y2−z3), xzy4+e−x2y4+y, x2y(y2x3+ 3)z+e−x2−y2}

Tính R H

F·bndS bnlà vector đơn vị hướng mặt cầu dS độ đo (diện tích) bề mặt

Bài ChoD hình vng với đỉnh(2,2),(3,3),(2,4),(1,3).Tính tích phân

Z Z D

ln(y2−x2)dxdy

Bài Giả sử hàm sốf :R2→Rthuộc lớpC4có tính chất: đạo hàm riêng phần bậc bậc hai củaf (x0, y0)đều bằng0, có đạo hàm riêng phần bậc ba tai(x0, y0) khác0 Chứng minh rằngf khơng đạt cực đại địa phương cực tiểu địa phương điểm tới hạn

Bài Chứng minh chuỗi

X n=1

nx

1 +n2(log(n))2x2

hội tụ trên[ε,∞)với mỗiε >0

Bài Giả sử hàm sốf :R3→Rthuộc lớpC1 chof(0,0,0) = 0và

∂f ∂x2

(0,0,0)6= 0, ∂f ∂x3

(0,0,0)6= 0, ∂f ∂x2

(0,0,0) + ∂f

∂x3

(0,0,0)6=−1

Chứng minh hệ phương trình

f(x1, f(x1, x2, x3), x3) =

f(x1, x2, f(x1, x2, x3)) =

xác định hàmx2=ϕ(x1), x3=ψ(x1)thuộc lớpC1 vớix1 lân cận của0 cho

f(x1, f(x1, ϕ(x1), ψ(x1)), ψ(x1)) =

f(x1, ϕ(x1), f(x1, ϕ(x1), ψ(x1))) =

Bài Chob∈[1, e]và dãy truy hồi a0= b

b,an+1= (b

b)an, n= 0,1,2, ,, tức là b

√ b,√bb

b

b

,√bb

b √ b b √ b

,√bb

b √ b b √ b b √ b ,

Chứng minh dãy số hội tụ tìm giới hạn

Bài Với n=1,2, ta định nghĩaxn: [−1,1]→Rbởi

xn(t) =     

−1nếu −1≤t≤ −1/n, nt −1/n≤t≤1/n,

(91)

a) Chứng minh {xn}là dãy Cauchy không gian metric (C([−1,1]), d),

C([−1,1]) tập hợp hàm liên tục trên[−1,1]và hàm khoảng cáchdcho

d(x, y) =

1

Z

−1

|x(t)−y(t)|dt

b) Chứng minh rằng(C([−1,1]), d)không đầy đủ

Tier I Algebra Exam: January 2009

Hướng dẫn: Mỗi toán phần tốn tính điểm, ra, tổng cộng 90 điểm Làm toán tờ giấy riêng Trừ có ghi khác, nên trình bày làm chi tiết kiểm tra khẳng định bạn

Bài (10 điểm)

(1) Chứng minh nhóm nhóm cyclic cyclic

(2) Hãy xác định, sai khác đẳng cấu, nhóm abel hữu hạn sinh có tính chất nhóm thực cyclic

Bài (5 điểm) ChoGlà nhóm Định nghĩa nhóm conH ∈Glà đặc trưng (charac-teristic) với đẳng cấuϕ:G→Gta cóϕ(H)⊂H Bây giả sử rằngH ≤Glà nhóm chuẩn tắc vàK≤H nhóm đặc trưng H Chứng minh rằngKlà nhóm chuẩn tắc củaG

Bài (5 điểm) ChoGlà nhóm abel hữu hạn cấpn, tốn tử nhân hai phần tử viết theo kiểu nhân Giả sử ánh xạf :x→xmlà tự đẳng cấu củaG, với số

nguyên dươngmcho trước Chứng minh rằngmvànnguyên tố

Bài 4.(10 điểm) ChoV không gian vector phức hữu hạn chiều, L(V)là không gian phức ánh xạ tuyến tínhV →V Với mỗiA∈L(V), hàm sốTA:L(V)→L(V)định nghĩa

TA(X) =AX−XA, ∀X ∈L(V)

là ánh xạ tuyến tính

(1) Giả sử A, B ∈L(V) có dạng chuẩn tắc Jordan (Jordan canonical form) Chứng minh rằngTA vàTB có dạng chuẩn tắc

(2) Giả sử số chiều củaV Chứng minh với mỗiA∈L(V)thìrank(TA)là Bài 5.(5 điểm) ChoV không gian vector thực vàT :V →V ánh xạ tuyến tính Giả sử vector khác0 V vector riêng T Chứng minh T ánh xạ đồng nhân với số

Bài (10 điểm) Trong ta xét ma trận trường số thực (1) ChoAlà ma trận thực2×2sao cho

A2+

1 0

=

0 0

Chứng minh Ađồng dạng với ma trậnB =

0

−1

(92)

(2) ChoAlà ma trận thựcn×nsao choA2+In = 0trong đóIn ma trận vuông đơn vị

cấpn Chứng minh nếun= 2m thìAđồng dạng với ma trậnB =

0 Im

−Im

Bài (10 điểm) ChoRlà vành giao hoán với đơn vị1và chứa ideals (1) Chứng minh phần tử khác 0củaR khả nghịch ước của0 (2) Điều ngược lại có khơng? Chứng minh câu trả lời

Bài 8.(5 điểm) Cho số nguyên tốp, vàFplà trường củapphần tử Giả sử ước chung

lớn hai đa thức

f(x) = 6x3+ 10x2−110x+ 16vàg(x) = 6x2+ 10x−16

trongFp[x]là Tìmp

Bài 9.(10 điểm) ChoF3 trường hữu hạn với phần tử vàF3là bao đóng đại số Cho Klà trường phân rã (splitting field) củag(x) =x21−1.

(1) Tìm số nghiệm g(x)trênF3

(2) (a) Tìm số phần tử K (b) Số phần tử trường thực cực đại A bao nhiêu? (một trường củaK gọi trường thực khơng bằngK)

Bài 10 (10 điểm)

(1) Giả sửγ mội số phức choγ2 là số đại số trên

Q Chứng minh rằngγ số đại số trênQ

(2) Choα, βlà số phức choαlà số siêu việt trênQ Chứng minh hai sốα−β vàαβ số siêu việt

Bài 11(10 điểm) Cho D miền nguyên (domain) Hai ideals khác khôngI, J ⊂D gọi comaximal nếuI+J =D, hai ideals gọi coprime if I∩J =I·J

(1) Chứng minh hai ideals I, J⊂Dlà comaximal chúng coprime

(93)

Olympic Học sinh - Sinh viên

Olympic Sinh viên Kiev 2009

Bài 1.Tam giác ABC nội tiếp đường trịn Liệu có ln tồn điểmD đường tròn cho tứ giácABCD ngoại tiếp

Bài 2.F0= 0, F1= 1, Fk =Fk−1+Fk−2, k≥2là dãy Fibonaci Tìm tất số nguyên dương

nsao cho đa thứcFnxn+1+Fn+1xn−1 bất khả quy trênQ[x]

Bài ChoA, B, C góc tam giác nhọn Chứng minh bất đẳng thức

cosA

sinBsinC +

cosB

sinCsinA+

cosC

sinAsinB ≥2

cosA

sinBsinC +

cosB

sinCsinA+

cosC

sinAsinB ≤

3

Bài Tìm tất số nguyên dươngnsao cho tồn ma trậnA, B, C∈Mn(Z)sao cho

ABC+BCA+CAB=I

Bài Cho hàm x, y : R → R cho (x(t)−x(s))(y(t)−y(s)) ≥ với t, s ∈ R Chứng minh tồn hai hàm không giảm f, g : R → R hàm z : R → R cho

x(t) =f(z(t)), y(t) =g(z(t))với mọit∈R

Bài Với(xn)n≥1 dãy số thực cho tồn giới hạn hữu hạn

lim

n→∞

1

n

n

X

k=1

xk Chứng minh với mọip >1tồn hữu hạn

lim

n→∞

1

np n

X

k=1

kp−1xk

Bài ĐặtK(x) =xe−x, x∈R Với mọin≥3, tính

sup

x1, ,xn∈R

min

1≤i<j≤nK(|xi−xj|)

Bài 8.Liệu có tồn hàm sốf :Q→Qsao chof(x)f(y)≤ |x−y|với mọix, y∈Q, x6=y Và với mọix∈Q, tập hợp{y∈Q| f(x)f(y) =|x−y|}là vơ hạn?

Bài 9.Tìm mọin≥2sao cho đánh số tất hốn vị tập{1,2, , n}bằng số từ1 đếnn! với cặp hai hốn vịσ, τ với số liền kề nhau, cặp thứ 1và

n!thìσ(k)6=τ(k)thỏa mãn với 1≤k≤n

Bài 10 Cho µ độ đo σ−đại số Borel R cho

Z

R

eaxdµ(x) < ∞ với

(94)

cho

Z

R

xeaxdµ(x) =

Bài 11.Với(ξn)n≥0,(νn)n≥0là hai dãy biến ngẫu nhiên gồm phần tử đôi độc lập phân phối (phân phối hai dãy khác nhau) NếuE(ξ0) = 0, P{v1= 1}=p, P{v1= 0}=

1−p,p∈(0,1) Kí hiệux0= 0, xn= n

X

k=1

vk, n≥1 Chứng tỏ

1

n

n

X

k=0

ξxk →0theo xác suất

n→ ∞

Bài 12.VớiX1, X2, , X2n biến ngẫu nhiên độc lập phân phối choX16= 0hầu chắn Kí hiệu

Yk=

k

X

i=1

Xi

v u u t

k

X

i=1

Xi2

,1≤k≤2n

Chứng minh bất đẳng thứcE(Y2

2n)≤1 + 4(E(Yn))2

Olympic Xác suất Kolmogorov 2009

Bài 1.Đại lượng ngẫu nhiênX có phương sai hữu hạn khơng đồng Chứng minh rằngP(X= 0)≤(E(X2))−1D(X)

Bài 2.Các tập hợpA1, A2, , A2000⊂A, tập chứa tối thiểu phần tử khơng có tập trùng Chứng minh tồn 100 phân hoạch tậpA, phân hoạch tạo tập đôi không giao nhauE1, E2, E3, E4, E5 cho tập Ai chứa hai tậpEi

Bài Từ container A mà có 1000 táo xanh 3000 táo đỏ người ta lấy nửa số táo chuyển sang containerB mà có 3000 táo xanh 1000 táo đỏ Sau từ containerB người ta lấy táo Tính xác suất để táo táo xanh

Bài 4.Cư dân thành phốN sau tan sở yêu thích câu cá Giờ tan sở họ ngẫu nhiên Ở hồ có cá chép cá rô Tỷ lệ chép làp Trong thành phố có đạo luật cấm người không bắt cá chép ngày, cư dân thành phố tuân thủ pháp luật, sau câu cá chép họ Hãy tính tỷ lệ số cá chép mà cư dân thành phố câu

Bài 5.Dãy đại lượng ngẫu nhiên(Xn)n∈Nhội tụ theo xác suất đại lượng ngẫu nhiênX cho với mỗin Xn vàX độc lập Phải chẳngX số hầu khắp nơi?

Bài Cho dãy đại lượng ngẫu nhiên X1, X2, với phân phối Poisson ứng với tham số

λ= Chứng minh rằngE(max{X1, X2, , Xn}) =O(lnn)khi n→ ∞

Bài Cho (Sn)n∈N dãy đại lượng ngẫu nhiên cho S0 = 0, Sn = ξ1 + +ξn, (ξj)j∈N đại lượng ngẫu nhiên đạt giá trị -1 ứng với xác suất 1/2 Kí hiệu

τ= inf{n∈N : Sn = 0} Vớia∈NtìmE(Na)vớiNa =|{j < τ : Sj=a}|

(95)

Bài Thực nghiệm với đại lượng ngẫu nhiên X có phân phối chuẩn với trung bình µ chưa biết, phương sai Để ước lượng µ ta sử dụng biểu thức f(X) với f hàm liên tục

E(f(X)2)<+∞ với mọiµ Chứng tỏ giá trị nhỏ (xác định tất hàmf thỏa mãn điều vậy) củasupµ∈RE(f(X)−µ)2 đạt hàmf(x) =x

Bài 10 Với p > 0, vàX1, X2, , Xn đại lượng ngẫu nhiên không gian xác suất, cho với > dãy

X

n=1

n−1P( max

k=1,2, ,n|Xk| > n

1/p) hội tụ Chứng minh rằng

(96)

Kỳ thi TST Việt Nam 2009 – Đề thi Bình luận

Trần Nam Dũng - Trường Đại học KHTN, ĐHQG Tp.Hồ Chí Minh

Kỳ thi chọn đội tuyển Việt Nam tham dự IMO 2009 diễn hai ngày 18 19/4/2009 Hà Nội Như thường lệ, ngày thí sinh làm tốn vịng Đề thi năm đánh giá khó chịu kết điểm chuẩn 15 cho suất vào đội tuyển chứng tỏ điều Dưới đề thi bình luận chúng tơi

Bình luận chung

Đề thi có nhiều khó, khó (Số học) (Hình học) Bài khó tính độc đạo nó, có thí sinh nắm vững thuyết phương trình Pell nâng cao cơng phá Bài hình số có cấu hình khó chịu, chất tốn bị dấu kỹ khiến nhiều thí sinh khơng tìm phương hướng Bài số thực chất khơng khó cách phát biểu cồng kềnh, thừa kiện nên làm cho thí sinh sợ lạc đường Tương tự số khơng q khó đặt vị trí cuối nên số thí sinh dám công phá không nhiều, câu gợi ý (câu a) đơn giản

Hai dễ kỳ thi số (Hình học) số (Bất đẳng thức) Đáng tiếc nhiều thí sinh gặp khó khăn với hai Tuy nhiên, lý khiến điểm cận kề khu vực “phao cứu sinh” đơng: có đến 10 nằm mức điểm 13-14 Đây điểm yếu đề thi chọn đội tuyển năm nay: tính phân loại khơng cao

I - Các toán cho điểm

Bài Cho tam giác ABC nhọn nội tiếp đường tròn tâm O; gọi A1, B1, C1 chân đường vng góc A, B, C xuống cạnh dối diện; gọi A2, B2, C2 điểm đối xứng A1, B1, C1 qua trung điểm cạnh BC, CA, AB Các đường tròn ngoại tiếp tam giác AB2C2, BC2A2, CA2B2 cắt(O) điểm thứ hai A3, B3, C3.Chứng minh A1A3, B1B3, C1C3 đồng quy

Lời giải

Gọi H trực tâm tam giácABC Gọi S điểm đối xứng củaH quaO Ta có

SA2⊥BC, SB2⊥CA, SC2⊥AB

Do đó, đường trịn ngoại tiếp tam giác AB2C2 đường trịn đường kính SA Gọi D điểm

đối xứng Aqua O SD⊥BC Do∠AA3D= 90◦ nên A3 thuộc đường thẳng SD Gọi M

trung điểm củaBC DoM A1||AA3 vàM A1=AA3/2nên A1A3 qua trọng tâmGcủa tam giác ABC Chứng minh tương tự ta cóB1B3vàC1C3 quaG VậyA1A3, B1B3, C1C3đồng quy

tại điểmG

Bình luận.Bài đơn giản Mấu chốt lời giải chứng minhAA3||BC Đáng tiếc

nhiều thí sinh làm đến không tiếp sử dụng “định lý” tự chế “Nếu có đường thẳng đồng quy (ý nóiAA2, BB2, CC2) lấy đối xứng qua ba trục đồng quy (ý

nói ba đường trung trực tam giác) ba đường thẳng ảnh đồng quy” Các bạn lấy phản ví dụ để chứng tỏ “mệnh đề” khơng

Bài Tìm tất số thực rsao cho với số dươnga, b, c ta ln có bất đẳng thức sau

r+ a

b+c r+

b

c+a r+

c a+b

r+1

(97)

Lời giải

Do bất đẳng thức nhất, ta giả sửa+b+c= Ta biến đổi bất đẳng thức đề dạng

X

(b−c)2(r

2

2 (b+c) +

ra

4 −

a

8)≥0

(Từ r≥1/2thì bất đẳng thức với mọia, b, c >0)

ĐặtSa =

r2

2 (b+c) +

ra

4 −

a

8 =

r2

2 − 8(4r

2−2r+ 1)a, vàS

b, Sc tương tự Bất đẳng thức

đưa dạng

X

(b−c)2Sa≥0

+ Cho b=cthì bất đẳng thức tương đươngSb=Sc≥0⇔4r2−(4r2−2r+ 1)b≥0

+ Cho a→0, b→1/2 ta được4r2−1 2(4r

2

−2r+ 1)≥0⇔4r2+ 2r−1≥0.

Ta chứng minh điều kiện đủ để bất đẳng thức với mọia, b, c Thật vậy, giả sử a≥b≥c Vì4r2−2r+ 1>0nên S

a≤Sb≤Sc Hơn đób≤1/2nên

Sb=r2/2−(4r2−2r+ 1)b/8≥r2/2−(4r2−2r+ 1)/16 = (4r2 + 2r−1)/16≥0

Sb+Sa =r2−(4r2−2r+ 1)(a+b)/8≥(4r2+ 2r−1)/8≥0

Từ

Sa(b−c)2+Sb(c−a)2+Sc(a−b)2=Sa(b−c)2+Sb((a−b) + (b−c))2+Sc(a−b)2

= (Sa+Sb)(b−c)2+ (Sb+Sc)(a−b)2+ 2Sb(a−b)(b−c)≥0

Bình luận.Ngồi cách giải trên, cịn có số cách giải khác khai triển toàn dùng bất đẳng thức Schur Muirhead Có số thí sinh tiếp cận phương pháp dồn biến

pqr nắm lý thuyết không vững nên phạm sai lầm "không sửa chữa được" Đây học cho bạn học sinh: Khi học phương pháp mới, phải học thật kỹ phân tích rõ điểm mạnh yếu phương pháp, điểm dễ bị sơ hở, sai lầm

II - Hai tốn trung bình

Bài Cho đa thứcP(x) =rx3+qx2+px+ 1 trong đóp, q, rlà số thực với r >0 Xét dãy số(an), n = 0, 1, xác định sau

a0= 1, a1=−p, a2=p2−q

an+3=−pan+2−qan+1−ran (n≥0)

Chứng minh đa thứcP(x)chỉ có nghiệm thực khơng có nghiệm bội dãy(an)có vơ số số âm

Bình luận.Bài tốn lẽ khơng khó đến (số thí sinh làm đếm đầu ngón tay) "đề bài" Việc đề cho giá trị ban đầu a0, a1, a2

khá đặc thù khiến thí sinh lạc đề, khơng biết khai thác Lời giải trình bày cho thầy giá trị khơng quan trọng, khơng ảnh hưởng đến kết tốn

(98)

Từ điều kiện đề suy phương trình đặc trưng phương trình sai phânx3+px2+qx+r=

có nghiệm thực âm hai nghiệm phức liên hợp

Giả sử ba nghiệm là−a, R(cosα+isinα), R(cosα−isinα)vớia >0, R >0,0< α < π

an=C1(−a)n+C2Rn(cosα+isinα)n+C3Rn(cosα−isinα)n

trong đóC1, C2, C3 số vàC2, C3 số phức liên hợp

ĐặtC2=R∗(cosϕ+isinϕ)vớiϕ∈[0,2π), ta có

an =C1(−a)n+Rn(R∗(cosϕ+isinϕ)(cosnα+isinnϕ) +R∗(cosϕ−sinϕ)(cosnα−isinnϕ))

=C1(−a)n+ 2RnR∗(cos(nα+ϕ))

Giả sử ngược lại tồn tạinsao choan≥0với mọin≥n0 Khi ta có

0≤an+1+aan= 2Rn+1R∗(cos((n+ 1)α+ϕ)) +a2RnR∗(cos(nα+ϕ))

= 2RnR∗(Rcos((n+ 1)α+ϕ) +acos(nα+ϕ)) = 2RnR∗.C.cos(nα+ϕ∗) (C >0, ϕ∗∈[0,2π))

với mọin≥n0

Điều khơng xảy vì0< α < πnên tồn vơ sốnsao chonα+ϕ∗∈(π/2+k2π,3π/2+2kπ) Bài 6.Có 6n+ 4nhà tốn học tham dự hội nghị, có 2n+ 1buổi thảo luận Mỗi buổi thảo luận có bàn trịn cho người ngồi vànbàn trịn cho người ngồi Biết người không ngồi cạnh đối diện lần

1 Hỏi thực việc xếp chỗ ngồi vớin= 1?

2 Hỏi thực việc xếp chỗ ngồi vớin >1?

Bình luận Bài tốn thuộc dạng xây dựng ví dụ Đây dạng tốn khơng thực quen thuộc với học sinh (vốn quen với tìm chứng minh) Chính điều việc tốn xếp vị trí số khiến cho có thí sinh giải giải Về thực chất khơng khó Vớin= 1, ta dễ dàng xây dựng ví dụ cách xếp:

1 (1 4), (5 10)

2 (1 7), (2 10)

3 (1 10), (2 7)

Ở đây, ý tưởng tách riêng, số cịn lại phân thành nhóm(2 4), (5 7)và(8 10), sau ghép cặp

Lại ghép cặp cách khác: (1 8), (3 10)

2 (1 9), (2 10)

(99)

Tiếp theo, với n= 2, ta tách 15 số cịn lại chia thành nhóm (2 6) (7 10 11) (12 13 14 15 16) tiếp tục xếp

1 (1 12) (3 13 14) (5 10 11 15 16) (1 13) (2 11 12 16) (4 10 14 15) (1 14) (2 10 12 15) (3 11 13 16) (1 10 15) (2 12 13) (4 11 14 16) (1 11 16) (2 12 14) (3 10 13 15)

Từ nảy sinh ý tưởng cho lời giải tổng quát: Tách số riêng, cịn lại chia thành nhóm, nhóm2n+ 1người Sẽ cho người lẻ tham gia bàn người, 2nngười lại tách tháchncặp phối hợp với cặp nhóm cịn lại tạo thành bàn người Vấn đề phải ghép cặp để hai người nhóm khơng ghép cặp với hai lần Như vậy, dễ thấy toán chuyển toán quen thuộc: Hãy xếp lịch thi đấu cho giải đấu gồm 2n+

đội thành2n+1lượt, lượt cóntrận đấu Bài tốn giải quy nạp hệ thặng dư

Lời giải Tách số 6n+ riêng Các số cịn lại chia thành nhóm: {1,2, ,2n+ 1},{2n+ 2, ,4n+ 2},{4n+ 3, ,6n+ 3} Ở bước thứk, từ nhóm ta chọn sốk,2k+ +kvà4k+ +k

để ghép với6n+ 4vào bàn Các số lại thuộc {1,2, ,2n+ 1}\{k} ta phân cặp theo quy tắc

i, jcùng cặp khii+j≡2k mod 2n+ Ta chứng minh cách chia thoả mãn điều kiện: i Hai sối, j xuất tương ứng lầnk lầnk0, do2k≡2k0 mod 2n+ 1⇔

k≡k0 mod 2n+

ii Với i thuộc A ={1,2, ,2n+ 1}\{k}, tồn j thuộcA choi+j ≡2k

mod 2n+ (Bạn đọc chứng minh chi tiết điều này)

Các số thuộc {2n+ 2, ,4n+ 2}\{2n+ +k}và{4n+ 3, ,6n+ 3}\{4n+ +k}cũng ghép cặp tương tự Lấy cặp từ nhóm 1, nhóm 2, nhóm ta ghép thành bàn người

III - Hai tốn khó

Bài Cho đường trịn (O) đường kính AB, M điểm tùy ý (O) Đường phân giác từ M tam giác AM Bcắt(O)tạiN Phân giác góc AM BcắtN A, N B P, Q Đường thẳng AM cắt đường trịn đường kính N Q điểm thứ hai R; đường thẳng BM cắt đường trịn đường kínhN P điểm thứ haiS Chứng minh đường trung tuyến kẻ từN tam giácN SR qua điểm cố định

Bình luận Đây tốn khó chịu Và nhiều thí sinh sa lầy toán này, tốn nhiều thời gian cơng sức khơng tìm sợi chứng minh Được biết toán tạo thành từ hai toán quen thuộc:

1) Cho tứ giác ABCD nội tiếp đường tròn Giả sử hai đường chéo cắt O K, L, M, N chân đường vng góc hạ từ O xuống AB, BC, CD, DA KLM N tứ giác ngoại tiếp Ngoài ra, O tâm đường trịn nội tiếp tứ giác KLM N ba đường thẳng (KL, M N, AC),(KN, LM, BC) đồng quy

(100)

Đáp án thức ngược theo hướng cồng kềnh, bao gồm bước tìm lại cấu hình ban đầu, tức tứ giác ngoại tiếp đường tròn(O), sử dụng ý tưởng chứng minh đường thẳng Newton để chứng minh thẳng hàng

Chỉ có số thí sinh làm trọn vẹn này, đáng ý lời giải trình bày (rất ngắn gọn sáng sủa) lời giải phương pháp toạ độ Cũng sáng sủa (đó điều đặc biệt lời giải theo hướng thường cồng kềnh xấu xí) đặc biệt hơn, nhờ lời giải mà phát điều kiện ∠AN B = 90◦ không cần thiết Thực cầnN

nằm phân giác góc∠AM B Sẽ lý thú bạn đọc tự kiểm tra lại điều (bằng phương pháp hình học phương pháp toạ độ)

Lời giải.Giả sửI=N S∩P Q, J =N R∩P Q VìM, N, Q, Rđồng viên, ta có∠AM P =∠RM Q=

∠J BM =∠J N Q Từ suy raM, N, B, J đồng viên

Tương tự,M, N, A, I đồng viên Vì vậy∠J BN=∠IAN = 90◦

TừA vàB kẻ đường thẳng vng góc với M N cắtN J vàN I tương ứng tạiU vàV Ta có

AU

J P =

N A

N P ⇒AU =J P N A N P,

BV

IQ =

N B

N Q ⇒BV =IQ

N B N Q

Từ AU BV = J P IQ N A N B

N Q

N P =

N A N B

N Q N B

SN J P

SN IQ

= N A

N B N Q N P

N J.N P.sin∠P N J N I.N Q.sin∠QN I =

N A N B

N J N I =

Suy raAU =BV, nhưngAU||BV nênAU BV hình bình hành VậyOlà trung điểm củaU V Mặt khác, ∆N AI ∼∆N SP ∼∆N BJ ∼∆N RQ

⇔ N A

N I =

N S

N P =

N B

N J =

N R

N B ⇒

N A N I

N S

N P =

N B N J

N R

N Q ⇒

N U N J

N S

N I =

N V N I

N R

N J ⇒

N U

N R =

N V N S

Do đóU V||RS Từ suy trung tuyến kẻ từN tam giácN SRđi qua trung điểm

O củaU V Ta có điều phải chứng minh

Bài 3.Choa, blà số nguyên dương khơng phương choabcũng khơng phương Chứng minh hai phương trình ax2−by2 = 1 vàax2−by2 =−1 khơng có nghiệm ngun dương

Lời giải Trước hết ta chứng minh bổ đề sau Bổ đề.Cho phương trình

Ax2−By2= (1)

vớiAvàABkhơng phương Gọi(a, b)là nghiệm nhỏ phương trình Pell kết hợp

x2−ABy2= (2)

Giả sử phương trình(1)có nghiệm và(x0;y0)là nghiệm nhỏ thì(x0;y0)là nghiệm

duy hệ phương trình

a=Ax2+By2 b= 2xy

Chứng minh.Giả sử(x0;y0)là nghiệm nhỏ (1) Đặtu=Ax20+By

0, v= 2x0y0 ta

(101)

Chứng tỏ(u;v)là nghiệm phương trình (2) Mà(a;b)là nghiệm nhỏ phương trình nênu≥a, v≥b

Ta chứng minhu=a, v=b Thật vậy, giả sử trái lạiu > a, v > b

a−b√AB <(a−b√AB)(a+b√AB) =a2−ABb2= ⇒(a−b√AB)(√Ax0+

By0)<(

Ax0+

By0)

⇒(ax0−Bby0)

A+ (ay0−Abx0)

B <(√Ax0+

By0)

Lại có

(a+b√AB)<(u+v√AB) =√Ax0+

By0

2

⇒(ax0−Bby0)

A−(ay0−Abx0)

B= (a+b√AB)(√Ax0−

By0) <(√Ax0+

By0)2(

Ax0−

By0) = (

Ax0+

By0)

Đặts=ax0−Bby0, t=ay0−Abx0 bất đẳng thức viết lại thành s√A+t√B < x0

A+y0

B (3)

s√A−t√B < x0

A+y0

B (4)

Tiếp theo, ta có(As2−Bt2) =A(ax

0−Bby0)2−B(ay0−Abx0)2= (a2−ABb2)(Ax20−By02) =

Ta thấys >0

s >0⇔ax0> Bby0⇔a2x20> B2b2y20⇔a2x20> Bb2(Ax20−1)

⇔(a2−ABb2)x2

0>−Bb2⇔x20>−Bb2

Bất đẳng thức cuối đúng, đós >0

Ta thấyt6= 0vìt= 0⇔ay0=Abx0⇔a2y02=A2b2x20⇔(ABb2+1)y02=Ab2(By02+1)y02=Ab2

Điều khơng thể xảy doAkhơng phương

Nếut >0thì(s;t)là nghiệm ngun dương (1), mà(x0, y0)là nghiệm nhỏ (1) nên s≥x0, t≥y0 Do vậys

A+t√B≥√Ax0+

By0, điều mâu thuẫn với (3)

Tương tự, vớit <0thì(s,−t)là nghiệm nguyên dương (1) ta dẫn đến bất đẳng thức mâu thuẫn với (4)

Vậyu=a, v=bhay (x0, y0)là nghiệm hệ

Trở lại toán giả sử hai phương trình

ax2−by2= (3)

bx2−ay2= (4)

có nghiệm

Gọi(m, n)là nghiệm nhỏ phương trìnhx2−aby2= 1,(x

1, y1)là nghiệm nhỏ

(102)

Áp dụng bổ đề 2, ta cóm=ax21+by12;n= 2x1y1 vàm=bx22+ay22;n= 2x2y2

Doax2

1=by12+ 1vàay22=bx22−1nên từ ta suy ax2

1+by21=bx22+ay22

⇔2by2

1+ = 2bx22−1

⇔b(x2

2−y21) =

Điều xảy dob >1 Nhận xét

1 Bổ đề lời giải phần định lý 5.19 , trang 148 Một số vấn đề Số học chọn lọc Nguyễn Văn Mậu chủ biên, NXB Giáo Dục 2008 Trong định lý có điều kiệnABkhơng phương vàA >1 Điểm cần đếnAkhơng phương điểm mà ta chứng minht6= Trong chứng minh trên,t= 0khi y2

0 =Ab2 Thay vào

đẳng thứcAx2

0−By02= ta suy raA(x20−Bb2) = Điều xảy doA >1 Vậy bổ

đề với điều kiệnABkhơng phương vàA >1 Và có nghĩa tốn ta nếuab khơng phương vàa, b >1

2 Bài tốn có xuất xứ gần với tốn sau đăng tạp chí American Mathe-matical Monthly:

“Giả sửxvày số nguyên dương chox+xy vày+xy số phương a Chứng minh có hai sốxvày số phương

b Hãy mơ tả tất cặp số nguyên dương x, ynhư vậy”.(∗)

Thật vậy, trước hết, biết chứng minh toán (*), ta giải số VTST09 sau:

Giả sử tồn x, y, u, v choax2−by2= 1vàau2−bv2=−1 ĐặtX =by2 vàY =au2 thì X(Y + 1) =by2.bv2= (byv)2,(X+ 1)Y =ax2.au2= (axu)2 là số phương, suy raX hoặc Y phương (theo tốn (*)!) Điều mâu thuẫn vìavàbkhơng phương

Đó bề ngồi, cịn đọc kỹ lời giải tốn (*) thấy mấu chốt lời giải tính chất “cơ bản” phương trình Pell loại loại tương đương với bổ đề lời giải

(103)

Sai lầm đâu?

Độ đo Metric

Phan Thành Nam - Khoa Toán, Đại học Copenhagen, Denmark

Giới thiệu.Một đặc tính tốt người học toán cẩn thận rèn luyện để tránh sai lầm lập luận Tuy nhiên, từ học sinh phổ thông giáo sư đại học việc mắc "lỗi" điều khó tránh khỏi Chuyên mục "Sai lầm đâu?" mở để chia sẻ số lỗi mà gặp trình học tập, đọc sách, nghiên cứu Học hỏi từ sai lầm mình, người khác điều hay, phải không bạn?

Trong viết muốn trao đổi với bạn chứng minh sách "Hausdorff mea-sure" C.A Rogers (Cambridge University Press, 1970) độ đo metric

Trước hết, xin nhắc lại số định nghĩa Chúng giả sử bạn đọc quen thuộc với khái niệm nhưσ−đại số (σ−algebra), độ đo (measure) không gian metric ChoΩlà tập hợp khác rỗng và2Ωlà họ tất tập của Ω(ký hiệu 2Ω là từ nhận xét tập hữu hạn cónphần tử số lượng tập là2n ) Một hàm sốµ: 2Ω→[0,∞]gọi độ đo ngồi (outer measure) thỏa mãn:

(i)µ(∅) = 0,

(ii) NếuA⊂B thìµ(A)≤µ(B),

(iii) Nếu E1, E2, họ đếm tập củaΩthì

µ ∞

[

n=1

En !

X

n=1

µ(En)

Tính chất (iii) gọi cộng tính đếm (countably subaddtive)

Từ độ đo ngồiµnhư vậy, thu hẹp2Ωxuống lớpσ−đại số mà

µlà độ đo Cách xây dựng tổng quát ý tưởng Carathéodory Để làm điều đó, ta định nghĩa tậpE⊂Ωlàµ−measurable thỏa mãn tiêu chuẩn Carathéodory

µ(A) =µ(A∩E) +µ(A∩Ec), ∀A⊂Ω

trong đóEc= Ω\E Một cách tương đương,E làµ−measurable nếu

µ(A∪B) =µ(A) +µ(B) với mọiA⊂E,B ⊂Ec (xin bạn đọc kiểm tra điều này) Ta có:

Định lý.[Carathéodory]Choµlà độ đo ngồi trênΩvàPlà lớp tập hợpµ−measurable.

Khi P

là mộtσ−đại số thu hẹp µtrênP

thỏa mãn tính cộng tính đếm

(104)

của Carathéodory trên, ta làm điều ta có độ đo ngồiµtrênΩmà tập mở (hoặc cách tương đương tập đóng) làµ−measurable

Để có điều nói trên, giả sử µthỏa mãn thêm tính chất Ta định nghĩa khoảng cách hai tập hợpA, B ⊂Ωlà

d(A, B) = inf{ρ(x, y)|x∈A, y∈B}

Ta gọiA, B tách ngặt (positively separated) nếud(A, B)>0 Tính chất sau nói rằngµ

thỏa mãn tính cộng tính hai tập tách ngặt (iv) NếuA, B⊂Ω, d(A, B)>0thì

µ(A∪B) =µ(A) +µ(B)

Một độ đo ngồi µ thỏa mãn từ (i)-(ii)-(iii)-(iv) gọi độ đo ngồi metric (metric outer measure, hoặcmetric measure) Có thể chứng minh µlà độ đo ngồi metric tập đóng (và đó, tập mở) đềuµ-measurable, sử dụng xây dựng Carathéodory ta thấy thu hẹp củaµtrênσ−đại số Borel tạo thành độ đo

(105)

Ghi chú:Định lý 17 nhắc tới chứng minh phát biểu µlà độ đo ngồi metric thỏa mãn tính chất "tăng ngặt": nếuA1⊂A2⊂ dãy tăng tập củaΩsao choAn vàAcn tách ngặt (positively separated), tức làd(An, Acn+1)>0,

µ ∞

[

n=1

An !

= sup

m∈N

(106)

Thông báo vấn đề giải giải thưởng

MathVn tạp chí khoa học mang tính cộng đồng giáo dục Tốn học Đi kèm với viết chuyên đề chúng tơi khởi xướng thi giải tốn khơng hạn chế đối tượng Qua chúng tơi mong muốn bạn trẻ tham gia tìm hiểu giải nhiều vấn đề khác phân thành ba loại: phần cho học sinh, phần cho sinh viên, phần vấn đề mở Các đối tượng tham gia giải tất phần

Cơ cấu giải thưởng

• Giải nhất: triệu đồng (1 giải)

• Giải nhì: triệu đồng (1 giải)

• Giải ba: triệu đồng (2 giải)

• Giải khuyến khích: Bản in Tạp chí MathVn năm (10 giải)

Qui tắc tính điểm

Với mục đích phát triển chuyên đề, động viên bạn tham gia trao đổi giao lưu, chúng tơi khuyến khích gửi lời giải cho toán phần tạp chí

+ Lời giải cho vấn đề mở (Open Problems): 15 điểm (Các bạn khơng cần gửi cho tạp chí MathVn mà gửi trực tiếp đến địa gốc chứa vấn đề đó, bạn đăng tên lời giải tính điểm)

+ Lời giải cho đề Qualify cho nghiên cứu sinh: 15 điểm + Bài toán đề nghị gửi cho tạp chí: 10 điểm

+ Lời giải cho đề kì MathVn chia mức khác tuỳ thuộc vào tính thách thức tốn:

* Bài khó hay: điểm

* Bài bình thường hay: điểm * Bài dễ hay : điểm

+ Lời giải tập chuyên đề: điểm

Với chế tính điểm sau số chúng tơi có bảng tổng kết theo thứ tự công bố diễn đàn tạp chí danh sách bạn đạt giải Tuy nhiên để giải xứng đáng điểm tổng kết sau kì báo phải lớn 150 điểm Các bạn nhận giải thưởng thông báo chuyển tiền qua bưu điện

Về mức độ chia khó dễ qua số báo sau:

+ Loại khó: A2,A4,A8,A9,A10,A11,A12,A13, A14, A25, A26, A32, B1,B2,B3,B4,B5,B6,B9.(8 điểm)

(107)

+ Loại dễ: A18, A22 (4 điểm) Về hạn giải bài:

+ Số : đến hết ngày 25/07/2009 + Số 2: đến hết ngày 29/10/2009 + Số 3: đến hết ngày 29/01/2010

Một lần thay mặt nhóm biên tập tạp chí MathVn mong có tham gia nhiều thầy giáo cô giáo bạn trẻ để cộng đồng tạp chí ngày phát triển bền vững chất lượng Thư từ thắc mắc xin gửi địa thư điện tử mathvn2008@gmail.com

hoặc truy cập vào websitehttp://mathvn.org

Ngày đăng: 24/05/2021, 15:41

Tài liệu cùng người dùng

Tài liệu liên quan